You are on page 1of 303

1 / 1 point

On December 1, 2020, MV and CD agreed to invest equal amounts and share profits equally to form
a partnership. MV invested P3,120,000 cash and a piece of equipment. CD invested some assets
which are shown on the next page:

Book value
Accounts Receivable 400,000
Inventory 1,120,000
Machineries, net 2,240,000
Intangibles, net 920,000

The assets invested by CD are not properly valued, P32,000 of the accounts receivable are proven
uncollectible. Inventories are to be written down to P1,040,000. Included in the machineries is an
obsolete apparatus acquired for P384,000 with an accumulated depreciation balance of P336,000.
Part of the intangibles is a patent with a carrying value of P56,000 which was sued upon by a
competitor. CD unsuccessfully defended the case and the final decision of the court was released on
November 29, 2020.
What is the fair value of the equipment invested by MV?

P968,000

P1,344,000

P1,400,000

P1,560,000

Question 2 0 / 1 point
On December 1, 2020, MG and AN are combining their separate businesses to form a partnership.
Cash and noncash assets are to be contributed. The noncash assets to be contributed and the
liabilities to be assumed are as follows:

MG AN
Book value Fair value Book value Fair value
Accounts Receivable 250,000 262,500 200,000 195,000
Inventory 400,000 450,000 200,000 207,500
PPE 1,000,000 912,500 862,500 822,500
Accounts Payable 150,000 150,000 112,500 112,500

MG and AN are to invest equal amount of cash such that the contribution of MG would be 10% more
than the investment of AN.
What is the amount of cash presented on the partnership’s statement of Financial Position on
December 1, 2020?

P2,512,500

P2,762,500

P5,025,000

P5,525,000

Question 3 1 / 1 point
JJ and KK are joining their separate business to form a partnership. Cash and non-cash assets are to
be contributed for a total capital of P300,000. The non-cash assets to be contributed and liabilities to
be assumed are:

JJ KK
Book Value Fair Value Book Value Fair Value
Accounts receivable P22,500 P22,500
Inventories 22,500 33,750 P60,000 P67,500
Equipment 37,500 30,000 67,500 71,250
Accounts payable 11,275 11,250 7,500 7,500

The partners’ capital accounts are to be equal after all contributions of assets and assumptions of
liabilities.
The amount of cash that each partner must contribute:

JJ – P75,000; KK – P18,750

JJ – P75,000; KK – P11,250

JJ – P161,250; KK – P157,500

JJ – P127,500; KK – P11,250

Question 4 1 / 1 point
Partnership and Corporation are both created by a mutual agreement.

True
False
Question 5 1 / 1 point
A, B and C decided to form ABC Partnership. It was agreed that A will contribute an equipment with
assessed value of P100,000 with historical cost of P800,000 and accumulated depreciation of
P600,000. A day after the partnership formation, the equipment was sold for P 300,000.

B will contribute a land and building with carrying amount of P1,200,000 and fair value of
P1,500,000. The land and building are subject to a mortgage payable amounting to P300,000 to be
assumed by the partnership. The partners agreed that B will have 60% capital interest in the
partnership. The partners also agreed that C will contribute sufficient cash to the partnership.
What is the total agreed capitalization of the ABC Partnership

P2,000,000

P3,000,000

P1,500,000

P2,500,000

Question 6 1 / 1 point
A, B and C decided to form ABC Partnership. It was agreed that A will contribute an equipment with
assessed value of P100,000 with historical cost of P800,000 and accumulated depreciation of
P600,000. A day after the partnership formation, the equipment was sold for P 300,000.

B will contribute a land and building with carrying amount of P1,200,000 and fair value of
P1,500,000. The land and building are subject to a mortgage payable amounting to P300,000 to be
assumed by the partnership. The partners agreed that B will have 60% capital interest in the
partnership. The partners also agreed that C will contribute sufficient cash to the partnership.

What is the cash to be contributed by C in the ABC Partnership?

P500,000

P700,000

P600,000

P800,000

Question 7 1 / 1 point
Which of the following transactions will decrease the capital balance of a partner?
Additional investment by said partner

Share in partnership profit

Drawings by said partner

Receipt of bonus from other partner

Question 8 1 / 1 point
AK and BK decided to form a partnership on October 1, 2020. Their Statement of Financial Position
on this date were:

AK Bk
Cash 65,625.00 164,062.50
Accounts Receivable 1,487,500.00 896,875.00
Merchandise Inventory 875,000.00 885,937.50
Equipment 656,250.00 1,268,750.00
Total 3,084,375.00 3,215,625.00

Accounts Payable 459,375.00 1,159,375.00


AK, Capital 2,625,000.00
BK, Capital 2,056,250.00
Total 3,084,375.00 3,215,625.00

They agreed the following adjustments shall be made:


• Equipment of AK is underdepreciated by P87,500 and that BK is overdepreciated by
P131,250.
• Allowance for doubtful accounts is to be set up amounting to P297,500 for AK and P196,875
for BK.
• Inventories of P21,875 and P15,312.50 are worthless in the books of AK and BK respectively.
• The partnership agreement provides for a profit and loss ratio of 70% to AK and 30% to BK.

Assuming the use of transfer of capital method, how much is the agreed capital of AK to bring the
capital balances proportionate to their profit and loss ratio.

P1,024,687.50

P2,218,125.00

P2,935,406.25
P2,390,937.50

Question 9 0 / 1 point
JJ and KK are joining their separate business to form a partnership. Cash and non-cash assets are to
be contributed for a total capital of P300,000. The non-cash assets to be contributed and liabilities to
be assumed are:

JJ KK
Book Value Fair Value Book Value Fair Value
Accounts receivable P22,500 P22,500
Inventories 22,500 33,750 P60,000 P67,500
Equipment 37,500 30,000 67,500 71,250
Accounts payable 11,275 11,250 7,500 7,500

The partners’ capital accounts are to be equal after all contributions of assets and assumptions of
liabilities.
The total assets of the partnership

P300,000

P281,250

P225,000

P318,750

Question 10 1 / 1 point
Charlie and Delta formed a partnership. Charlie invested cash worth P85,000 and a machine. On the
other hand, Delta contributed cash worth P55,000 and an equipment which has a mortgage of
P35,000 which the partners agreed to assume. The total capital after formation was P360,000. They
also further agreed to reflect 55:45 ratio as to their capital balances respectively. Charlie invested an
additional P8,000 cash to be in conformity with their capital ratio agreement. On the other hand,
Delta withdrew certain amount of cash also to conform to their capital ratio agreement. No other
investment or withdrawal occurred other than mentioned to reflect their capital ratio agreement.
How much is the fair value of the machine?

P105,000

P107,000

P115,000

P113,000

Question 11 0 / 1 point
Jamby and Minam just formed a partnership. Jamby contributed cash of P2,205,000 and office
equipment that cost P945,000. The equipment had been used in her sole proprietorship and had
been 70% depreciated, the appraised value of the equipment is P630,000. Jamby also contributed a
note payable of P210,000 to be assumed by the partnership. Jamby is to have 60% interest in the
partnership. Miriam contributed only P1,575,000 merchandise inventory at fair market value.
Assume the use of bonus method, the partners’ capital must be in conformity with their profit and
loss ratio upon formation.
Solution: JAMBY(60%) MINAM TOTAL CAPITAL
CASH 2,205,000
EQUIPMENT 630,000
INVENTORY 1,575,000
NOTE PAYABLE (210,000)
TOTAL 2,625,000 1,575,000 4,200,000
4,200,000*60%=2520,000
4200,000*40%=1,680,000-1,575,000=105,000

In the formation of a partnership, which of the following is true?

The agreed capital of Jamby upon formation is P2,625,000

The total agreed capital of the partnership is P4,375,000

The capital of Miriam will increase by P105,000 as a result of the transfer of capital

There is either an investment or withdrawal of asset under the bonus method


Question 12 1 / 1 point
Alma and Becca have just formed a partnership. Alma contributed cash of P176,400 and office
equipment that cost P75,600. The equipment had been used in his sole proprietorship and had been
70% depreciated, the current value of the equipment is P50,400. Alma also contributed a note
payable of P16,800 to be assumed by the partnership. Alma is to have a 30% interest in the
partnership. Becca contributed P256,000 land at fair market value. Becca should make additional
investment of:
SOLUTION: ALMA(30%) BECCA
CASH 176,400
EQUIP. 50,400
LAND 256,000
N/P (16,800)
TOTAL 210,000 256,000
ALMA 210,000/30%=700,000
BECCA 700,000*70%=490,000-256,000=234,000

P210,000
P256,000

P490,000

P234,000

Question 13 1 / 1 point
Alley and Barvey established a partnership on December 1, 20x4. They agreed that Alley will
contribute cash of P20,000; Land of P15,000 and Building of P50,000. Alley’s accounts payable of
P10,000 is to be assumed by the partnership. Barvey will contribute cash of P30,000 and furniture
and fixtures of P25,000
Assume that each partner initially should have an equal interest in partnership capital with no
contribution of intangible asset (bonus method). How much are the capital balances of each partner?
SOLUTION:
ALLEY BARVEY TOTAL CAPITAL
CASH 20,000 30,000
LAND 15,000
BUILDING 50,000
F&F 25,000
A/P (10,000)
TOTAL 75,000 55,000 130,000
130,000*.5=65,000

P75,000 for Alley and P55,000 for Barvey


P65,000 for Alley and P65,000 for Barvey.

P75,000 for Alley and P75,000 for Barvey.

P85,000 for Alley and P55,000 for Barvey

Question 14 1 / 1 point
On December 1, 2020, DD and EE formed a partnership with each contributing the following assets at
fair market values:

DD EE
Cash P18,000 P36,000
Machinery and equipment 27,000
Land 180,000
Building 54,000
Office furniture 27,000

The land and building are subject to a mortgage loan of P108,000 that the partnership will assume.
The partnership agreement provides that DD and EE share profits and losses, 40% and 60%,
respectively ad partners agreed to bring their capital balances in proportion to the profit and loss
ratio, and using the capital balance of EE as the basis. The additional cash investment made by DD
should be:

SOLUTION:
DD(40%) EE(60%)
DD,CAPITAL 72,000
EE, CAPITAL 162,000
162,000/60%=270,000
270,000*40%=108,000-72,000=36,000

P36,000

P171,000

P332,500

P268,200

Question 15 0 / 1 point
When a partner withdraws cash or other assets, the drawing account is

Credited

Debited

Debited and credited

Not affected

Question 16 0 / 1 point
A stipulation exempting a capitalist partner from losses is valid.

True
False
Question 17 1 / 1 point
Charlie and Delta formed a partnership. Charlie invested cash worth P85,000 and a machine. On the
other hand, Delta contributed cash worth P55,000 and an equipment which has a mortgage of
P35,000 which the partners agreed to assume. The total capital after formation was P360,000. They
also further agreed to reflect 55:45 ratio as to their capital balances respectively. Charlie invested an
additional P8,000 cash to be in conformity with their capital ratio agreement. On the other hand,
Delta withdrew certain amount of cash also to conform to their capital ratio agreement. No other
investment or withdrawal occurred other than mentioned to reflect their capital ratio agreement.
How much is the fair value of the equipment?
SOLUTION:
MACHINE=360,000*55%=198,000-85,000-8,000=105,000
INTIAL CONTRIBUTION BY CHARLIE= 85,000+105,000=190,000
360,000* 45%=162,000-(55,000+8,000+35,000)=150,000
Add 8,000 because it’s the payment being paid by Charlie as additional contribution that is being
paid by Delta and is withdrawn to conform with their capital ratio agreement.

P142,000

P107,000

P115,000

P150,000

Question 18 1 / 1 point
In the partnership books, there are as many capital and drawing accounts as there are partners.

True
False
Question 19 1 / 1 point
At the date of partnership formation, the amount credited to a partner’s capital is less than the fair
value of the property contributed. Which of the following is the most valid reason?

Bonus has been given by the partner to the other partners.

Goodwill arising from partnership formation has been recognized

The property contributed by the partner is impaired.

The property contributed by the partner has been subjected to positive asset
revaluation
Question 20 1 / 1 point
Partners are personally liable for the liabilities of the partnership if the partnership is unable to pay.

True
False

The net contributions (assets and related liabilities assumed by the partnership) of the partners to
the partnership are measured at

Cost

Discretionary amount determined by partners

Any of these

Fair value

Question 2 1 / 1 point
If a partner’s capital balance is created for an amount greater than or less than the fair value of his
net contribution, the excess or deficiency is called a

Goodwill

Premium

Discount

Bonus

Question 3 1 / 1 point
Under the bonus method, any increase or decrease in the capital credit of a partner

Deducted from or added to the capital credits of the other partners.

Deferred and amortized to profit or loss

Recognized as expense
Recognize as goodwill

Question 4 1 / 1 point
Partnership capital and drawings accounts are similar to the corporate

Preferred and common stock accounts.

Paid in capital and retained earnings accounts.

Retained earnings account.

Paid in capital, retained earnings, and dividends accounts.

Question 5 0 / 1 point
Under the bonus method, the asset contribution of the partner receiving the bonus is

Debited at an amount greater than the fair value of the asset contributed.

Debited at an amount lesser than the fair value of the asset contributed.

Either a or b

Debited at an amount equal to the fair value of the asset contributed.

Question 6 1 / 1 point
Transactions between and among the partners are

Recorded in the partnership books

Either a or b

Neither a nor b

Not recorded in the partnership books

Question 7 1 / 1 point
A and B agreed to form a partnership. A contributed cash of P100,000 while B contributed cash of
P200,000. The partnership agreement stipulates that A and B will have equal interest on the initial
capital of the partnership and in subsequent partnership profits and losses.
Which of the following statements is correct?

The partnership’s capital after recording the contribution of A but before


recording the contribution of B is P150,000.

The partnership’s capital after recording the contributions of both A and B is


greater than P300,000.

None of these.

A’s contribution will be debited for P150,000.

Question 8 1 / 1 point
A and B agreed to form a partnership. A contributed cash of P100,000 while B contributed cash of
P200,000. The partnership agreement stipulates that A and B will have equal interest on the initial
capital of the partnership and in subsequent partnership profits and losses.
Which of the following statements is correct?

A and B shall have capital balances of P150,000 each after the partnership
formation.

The capital balances of the partners after the partnership formation are P100,000
and P200,000, respectively. Any adjustment on the partners’ capital balances to
reflect the effects of the contractual stipulations shall not be made through the
partnership books.

The partnership formation will result to a debit to an unidentifiable asset call


“good wheel”.

The contractual agreement is unfair because A’s contribution is way below the
agreed interest of A in the partnership. Therefore, the contractual agreement is
void.

Question 9 1 / 1 point
A and B agreed to form a partnership. A contributed cash of P100,000 while B contributed cash of
P200,000. The partnership agreement stipulates that A and B will have equal interest on the initial
capital of the partnership and in subsequent partnership profits and losses.
Which of the following statements is incorrect?

The effect of the contractual stipulation is a decrease in B’s capital balance and a
corresponding increase in A’s capital balance.

The contractual stipulation does not affect the debit recording of the partners’
respective asset contributions.

The total partnership assets after the partnership formation is P300,000.

B’s asset contribution will be debited at a decreased amount of P150,000.

Question 10 1 / 1 point
A and B formed a partnership. The partnership agreement stipulates the following:
• A shall contribute noncash assets with carrying amount of P60,000 and fair value of
P100,000.
• B shall contribute cash of P200,000
• A and B shall have interests of 80% and 20%, respectively, on both the initial partnership
capital and in subsequent partnership profits and losses.
• No outside cash settlements shall be made between and among the partners.

The total partnership capital after the formation is


SOLUTION:
100,000+200,000=300,000

300,000

360,000

420,000

260,000

Question 11 1 / 1 point
A and B formed a partnership. The partnership agreement stipulates the following:
• A shall contribute noncash assets with carrying amount of P60,000 and fair value of
P100,000.
• B shall contribute cash of P200,000
• A and B shall have interests of 80% and 20%, respectively, on both the initial partnership
capital and in subsequent partnership profits and losses.
• No outside cash settlements shall be made between and among the partners.

The adjusted capital account of B after the formation is


SOLUTION: 300,000 *.2= 60,000

200,000

100,000

60,000

None of these

Question 12 1 / 1 point
A and B formed a partnership. The partnership agreement stipulates the following:
• A shall contribute noncash assets with carrying amount of P60,000 and fair value of
P100,000.
• B shall contribute cash of P200,000
• A and B shall have interests of 80% and 20%, respectively, on both the initial partnership
capital and in subsequent partnership profits and losses.
• No outside cash settlements shall be made between and among the partners.

The entry to record the contribution of B (and bonus given to A) includes

All of these

A credit to A's capital account for P140,000

A credit to B's capital account for P200,000

A debit to cash for P60,000

Question 13 1 / 1 point
A, B and C formed a partnership. Their contributions are as follows:

A B C
Cash 50,000 40,000 140,000
Equipment 150,000
Totals 50,000 190,000 140,000

Additional information:
• Although C has contributed the most cash to the partnership, he did not have the full amount
of P140,000 available and was forced to borrow P40,000. The partners agreed that half of
the amount borrowed shall be assumed by the partnership.
• The equipment contributed by B has an unpaid mortgage of P20,000, the repayment of which
is not assumed by the partnership.
• The partners agreed to equalize their interest. Cash settlement among the partners are to be
made outside the partnership.
Which partner(s) shall receive cash payment from the other partner(s)?

SOLUTION:
A B C TOTAL CAPITAL
CASH 50,000 40,000 140,000
EQUIP 150,000
LOAN (20,000)
TOTAL 50,000 190,000 120,000 360,000
360,000/3= 120,000-190,00 = (70,000)
A shall pay 70,000 to B as a refund for capital contribution

A shall receive P70,000 from B

A shall pay P70,000 to B

B shall receive P70,000 from C

C shall pay P70,000 to A

Question 14 1 / 1 point
A and B agreed to form a partnership. The partnership agreement stipulates the following:
• Initial capital of P300,000.
• A 25:75 interest in the equity of the partnership.

A contributed P100,000 cash while B contributed P200,000 cash. Which partner should provide
additional investment (or withdraw part of his investment) in order to bring the partners’ capital
credits equal to their respective interests in the equity of the partnership?
SOLUTION:
A: (300,000*25%=75,000) NO ADDITIONAL INVESTMENT
B: (300,000*75%=225,000) AN ADDITIONAL INVESTMENT OF 25,000
No additional contribution or withdrawal shall be made.

B shall withdraw capital of P25,000.

B shall make an additional investment of P25,000.

A shall provide additional capital of P25,000.

Question 15 1 / 1 point
On July 1, 20X3, Monuz and Pardo form a partnership, agreeing to share profits and losses in the
ratio of 4:6, respectively. Monuz contributed a parcel of land that cost him P25,000. Pardo
contributed P50,000 cash. The land was sold for P50,000 on July 1, 20X3 four hours after formation
of the partnership. How much should be recorded in Monuz capital account on formation of the
partnership?

P10,000

P20,000

P50,000

P25,000

Question 16 1 / 1 point
The business assets and liabilities of John and Paul appear below:

John Paul
Cash P11,000 P22,354
Accounts receivable 234,536 567,890
Inventories 120,035 260,102
Land 603,000 -
Building - 428,267
Furniture and fixtures 50,345 34,789
Other Assets 2,000 3,600
Total P1,020,916 P1,317,002

Accounts payable 178,940 243,650


Notes payable 200,000 345,000
John, capital 641,976 -
Paul, capital - 728,352
Total P1,020,916 P1,317,002

John and Paul agreed to form a partnership contributing their respective assets and equities subject
to the following adjustments:
a. Accounts receivable of P20,000 in John’s books and P35,000 in Paul’s are uncollectible.
b. Inventories of P5,500 and P6,700 are worthless in John’s and Paul’s respective book.
c. Other assets of P2,000 and P3,600 in John’s and Paul’s respective books are to be written
off.
The capital account of the partners after the adjustments will be:

SOLUTION:

John Paul
Cash P11,000 P22,354
Accounts receivable 234,536- 567,890-
20,000 35,000
Inventories 120,035- 260,102-
5,500 6700
Land 603,000 -
Building - 428,267
Furniture and fixtures 50,345 34,789
Other Assets 2,000-2000 3,600-3,600
Accounts payable (178,940) (243,650)
Notes payable (200,000) (345,000)
John, capital 614,476 -
Paul, capital - 683,052
Total

John’s P613,576; Paul’s P683,350

John's P614,476; Paul's P683,052

John’s P615,942; Paul’s P717,894

John’s P649,876; Paul’s P712,345

Question 17 1 / 1 point
Red, White and Blue form a partnership on May 1, 20X3. They agree that Red will contribute office
equipment with a total fair value of P40,000; White will contribute delivery equipment with a fair
value of P80,000; and Blue will contribute cash. If Blue wants a one third interest in the capital and
profits, he should contribute cash of:
SOLUTION:
40,000+80,000=120,000/(2/3)=180,000
180,000*(1/3)=60,000

P120,000

P40,000

P180,000

P60,000

Question 18 1 / 1 point
Aldo, Bert, and Chris formed a partnership on April 30, with the following assets, measured at their
fair values, contributed by each partner:

Aldo Bert Chris


Cash P10,000 P12,000 P30,000
Delivery trucks 150,000 28,000 -
Computers 8,500 5,100 -
Office furniture 3,500 2,500
Totals P168,500 P48,600 P32,500

Although Chris has contributed the most cash to the partnership, he did not have the full amount of
P30,000 available and was forced to borrow P20,000. The delivery truck contributed by Aldo ha a
mortgage of P90,000 and the partnership is to assume responsibility for the loan. The partners
agreed to equalize their interest. Cash settlement among the partners are to be made outside the
partnership.
SOLUTION:

Aldo Bert Chris TOTAL


Cash P10,000 P12,000 P30,000
Delivery trucks 150,000 28,000 -
Computers 8,500 5,100 -
Office furniture 3,500 2,500
MORTGAGE PAYABLE (90,000)
Totals P78,500 P48,600 P32,500 159,600

159,600/3=53,200
CHRIS: 53,200-32,500=20,700
BERT: 53,200-48,600=4,600
78,500-53,200=25,300

Berth should pay Aldo, P25,300 and Chris, P20,700.

Chris should pay Aldo, P25,300 and Bert, P4,600.

Aldo should pay Bert and Chris, P25,300.

Bert and Chris should pay Aldo, P4,600 and P20,700 respectively.

Question 19 1 / 1 point
Cong and Dong have just formed a partnership. Cong contributed cash of P126,000 and computer
equipment that cost P54,000. The computer had been used in his sole proprietorship and had been
depreciated to P24,000. The fair value of the equipment is P36,000. Cong also contributed a note
payable of P12,000 to be assumed by the partnership. Cong is to have 60% interest in the
partnership. Dong contributed only P90,000 cash.
Cong should make an additional (withdrawal) of:
SOLUTION:
CONG: 126,000+36,000-12,000=150,000
90,000/40%=225,000
225,000*.6=135,000
150,000-135,000= 15,000

(P76,800)

P96,000

(P15,000)

P84,000

Question 20 1 / 1 point
On June 1, 20X3, May and Nora formed a partnership. May is to invest assets at fair value which are
yet to be agreed upon. She is to transfer her liabilities and is to contribute sufficient cash to bring her
capital to P210,000 which is 70% of the total capital of the partnership.
Details regarding the book values of May’s business assets and liabilities and their corresponding
valuations are:

Book values Agreed valuations


Accounts receivable P58,000 P58,000
Allowance for doubtful accounts 4,200 5,000
Merchandise inventory 98,400 107,000
Store equipment 32,000 32,000
Accumulated depreciation – Store equipment 19,000 16,400
Office equipment 27,000 27,000
Accumulated depreciation – Office equipment 14,200 8,600
Accounts payable 56,000 56,000

Nora agrees to invest cash of P42,000 and merchandise valued at current market price. The value of
the merchandise to be invested by Nora and the cash to be invested by May are:

SOLUTION:
210,000/70%= 300,000
NORA: 300,000* 30%= 90,000
90,000-42,000=48,000
MAY: 58,000-5,000+107,000+32,000-16,400+27,000-8,600-56,000= 138,000
210,000-138,000=72,000

P48,000 and P72,000, respectively

P48,000 and P138,000, respectively

P252,000 and P138,000, respectively

P90,000 and P72,000, respectively

PARTNERSHIP OPERATIONS
1 / 1 point
According to the Civil Code of the Philippines, the designation of losses and profits can be entrusted
to one of the partners if the other partners are silent.

True
False
Question 2 1 / 1 point
According to the Civil Code of the Philippines, a stipulation which excludes one or more partners
from any share in the profits or losses is void.

True
False
Question 3 1 / 1 point
No bonus is allocated to any partner when the partnership incurred loss during the period.

True
False
Question 4 1 / 1 point
Salaries are nevertheless provided to the partners, if stipulated in the partnership agreement, even if
the partnership incurs loss.

True
False
Question 5 1 / 1 point
How should the partners in a business partnership share the profits or losses of the partnership?

Equally

At whatever basis of allocation that the dominating partner deems reasonable.

In accordance with the partnership agreement

Based on “rock, paper, scissors;” winner takes all

Question 6 0 / 1 point
According to the Philippine Civil Code, if only the share of each partner in the profits has been agreed
upon, the share of each in the losses shall be

equally.

equally, but the industrial partner shall not share in the loss.

the same as the sharing in profits.


the same as the sharing in profits. However, the industrial partner shall not share
in the loss.

Question 7 1 / 1 point
According to the Philippine Civil Code, in the absence of stipulation on the sharing of profits or
losses, partnership profits and losses shall be shared by the partners

equally.

in accordance with the partnership agreement.

in proportion to what the partners may have contributed.

in proportion to what the partners may have contributed but the industrial partner
shall not be liable for the losses.

Question 8 1 / 1 point
Which of the following is not a component of the formula used to distribute partnership profits to
the partners?

Salary allocation to those partners working.

After all other allocation, the remainder divided according to the profit and loss sharing
ratio.

Interest on the average capital investments.

Interest on notes to partners

Question 9 1 / 1 point
Which of the following is not considered a legitimate expense of a partnership?

Supplies used in the partners’ offices.

Depreciation on assets contributed to the partnership by partners.

Salaries for management hired to run the business.

Interest paid to partners based on the amount of invested capital.

Question 10 1 / 1 point
An incentive normally given to the managing partner in recognition of managerial skill or ability he
contributes to the partnership refers to:

Bonus
Goodwill

Income

Interest

Question 11 1 / 1 point
Red and White formed a partnership in 20X3. The partnership agreement provides for annual salary
allowances of P55,000 for Red and P45,000 for White. The partners share profits equally and losses
in a 60/40 ratio. The partnership had earnings of P80,000 for 20X3 before any allowance to partners.
What amount of these earnings should be credited to each partner’s capital account?
SOLUTION:
RED WHITE TOTAL
ANNUAL SALARY ALLO. 55,000 45,000 100,000
LOSS(60:40) 20,000 (12,000) (8,000) (20,000)
TOTAL 43,000 37,000 80,000
EARNINGS:80,000-100,000=-20,000

Red P40,000; White P40,000

Red P43,000; White P37,000

Red P44,000; White P36,000

Red P45,000; White P35,000

Question 12 1 / 1 point
Fox, Greg, and Howe are partners with average capital balances during 20X2 of P120,000, P60,000,
and P40,000, respectively. Partners receive 10% interest on their average capital balances. After
deducting salaries of P30,000 to Fox and P20,000 to Howe, the residual profit or loss is divided
equally . In 20X3 the partnership sustained a P33,000 loss before interest and salaries to partners. By
what amount should Fox’s capital account change?
SOLUTION:
FOX GREG HOWE TOTAL
INTEREST 12,000 6,000 4,000 22,000
SALARIES 30,000 20,000 50,000
LOSS(105,000/3) (35,000) (35,000) (35,000) (105,000)
TOTAL 7,000 (29,000) (11,000) (33,000)
LOSS: -33,000-72,000=-105,000
P7,000 increase.

P11,000 decrease.

P35,000 decrease.

P42,000 increase.

Question 13 1 / 1 point
Garcia and Henson formed a partnership on January 2, 20X5 and agreed to share profits 90% and
10%, respectively. Garcia contributed capital of P25,000. Henson contributed no capital but has
specialized expertise and manages the firm full time. There were no withdrawals during the year. The
partnership agreement provides for the following:
• Capital accounts are to be credited annually with interest at 5% of beginning capital.
• Henson is to be paid a salary of P1,000 a month.
• Henson is to receive a bonus of 20% of income calculated before deducting his salary and
interest on both capital accounts.
• Bonus, interest, and Henson’s salary are to be considered partnership expenses.

The partnership 20X5 income statement as follows:

Revenues 96,450
Expenses (including salary, interest, and bonus) 49,700
Net income 46,750
SOLUTION:
B=.2(46,750+(.05*25,000)+(1,000*12)+B)
B=12,000+.2B
.8B=12,000
B=15,000

What is Henson’s 20X5 bonus?

P11,688

P12,000

P15,000

P15,738

Question 14 1 / 1 point
JJ and KK are partners who share profits and losses in the ratio of 60%:40%, respectively. JJ’s salary is
P60,000 and P30,000 for KK. The partners are also paid interest on their average capital balances. In
20X2, JJ received P30,000 of interest and KK, P12,000. The profit and loss allocation is determined
after deductions for the salary and interest payments. If KK’s share in the residual income income
after deducting salaries and interest) was P60,000 in 20X2, what was the total partnership income?
SOLUTION: JJ KK TOTAL
SALARY 60,000 30,000 90,000
INTEREST 30,000 12,000 42,000
RESID INCOME 90,000 60,000 150,000
TOTAL 180,000 102,000 282,000

60,000/40%=150,000+90,000+42,000=282,000
150,000*.6=90,000

P192,000

P345,000

P282,000

P387,000

Question 15 1 / 1 point
Cab and Jo are considering forming a partnership whereby profits will be allocate through the use of
salaries and bonuses. Bonuses will be 10% of net income after total salaries and bonuses. Cab will
receive a salary of P30,000 and a bonus. Jo has the option of receiving a salary of P40,000 and a 10%
bonus or simply receiving a salary of P52,000. Both partners will receive the same amount of bonus.
Determine the level of net income that would be necessary so that Jo would be indifferent to the
profit sharing option selected.
SOLUTION:
40,000+.10(X-70,000-B)=52,000+12,000
BONUS MUST BE 52,000-40,000=12,000*2=24,000
.10(X-70,000-24,000)=24,000
.10X-9,400=24,000
X=334,00O

P240,000

P300,000
P94,000

P334,000

Question 16 1 / 1 point
The partnership agreement of XX, YY and ZZ provides for the year-end allocation of net income in the
following order:
• First, XX is to receive 10% of net income up to P200,000 and 20% over P200,000.
• Second, YY and ZZ each are to receive 5% of the remaining income over P300,000.
• The balance of income is to be allocated equally among three partners.

The partnership’s 20X1 net income was P500,000 before any allocation to partners. What amount
should be allocated to XX?
SOLUTION:
XX YY ZZ TOTAL
(.1*200,000) 500,000
+(.2*3000,000)= 80,000 (80,000) 6,000 6,000 92,000
420,000
(300,000)
120,000
*.05
6,000
INCOME 136,000 136,000 136,000 408,000
TOTAL 216,000 142,000 142,000 500,000
NET INCOME: 500,000-92,000=408,000/3=136,000

P202,000

P216,000

P206,000

P220,000

Question 17 1 / 1 point
AA, BB, and CC are partners with average capital balances during 20X2 of P360,000, P180,000, and
P120,000, respectively. Partners receive 10% interest on their average capital balances. After
deducting salaries of P90,000 to A and P60,000 to CC the residual profit or loss is divided equally. In
20X2 the partnership sustained a P99,000 loss before interest and salaries to partners. By what
amount should AA’s capital account change?

SOLUTION:
AA BB CC TOTAL
INTEREST 36,000 18,000 12,000 66,000
SALARIES 90,000 60,000 150,000
RESID. (105,000) (105,000) (105,000) (315,000)
TOTAL 21,000 (87,000) (33,000) (99,000)

LOSS=(99,000)-216,000=-315,000/3=-105,000

P21,000 increase

P33,000 decrease

P105,000 decrease

P126,000 increase

Question 18 0 / 1 point
RR and PP share profits after the provision of annual salary allowances of P14,400 and P13,200,
respectively in the ratio of 6:4. However, if partnership’s net income is insufficient to provide for said
allowances in full amount, the net income shall be divided equally between the partners. In 20X1,
the following errors were discovered: Depreciation for 20X1 is understated by P2,100, and the
inventory on December 31, 20X1 is overstated by P11,400. The partnership net income for 20X1 was
reported to be P19,500.
The capital accounts of the partners should be increased (decreased) by:
SOLUTION:
ADJUSTED NET INCOME: 19,500-2,100-11,400= 6,000
DECREASE OF 19,500
19,500-6,000=13,500
13,500/2= DECREASE OF 6,750

RR, P(6,540); PP, P(6,540)

RR, P3,000; PP, P3,000


RR, P(6,960); PP, P(6,540)

RR, P(6,750); PP, P(6,750)

Question 19 1 / 1 point
HH, MM and AA formed a partnership on January 1, 20X1, and contributed P150,000, P200,000, and
P250,000, respectively. Their articles of co-partnership provide that the operating income be shared
among the partners as follows: as salary, P24,000 for HH, P18,000 for MM, and P12,000 for AA;
interest of 12% on the average capital during 20X1 of the three partners; and the remainder in the
ration of 2:4:4, respectively.
The operating income for the year ending December 31, 20X1 amounted to P176,000. HH
contributed additional capital of P30,000 on July 1 and made a drawing of P10,000 on October 1;
MM contributed additional capital of P20,000 on August 1 and made a drawing of P10,000 on
October 1; and, AA made a drawing of P30,000 on November 1.
The partners’ capital balances on December 31, 20X1 are:
SOLUTION:
HH MM AA TOTAL
SALARY 24,000 18,000 12,000 54,000
INTEREST 19,500 24,700 29,400 73,600
RESID(2:4:4) 9,680 19,360 19,360 48,400
TOTAL 53,180 62,060 60,760 176,000
CAPITAL 170,000 210,000 220,000
TOTAL 223,180 272,060 280,760

NET INCOME=176,000- 127,600=48,400

AVERAGE CAPITAL BALANCES:


HH MM AA
150,000*6/12= 75,000 200,000*7/12= 116,667 250,000*10/12= 208,333
180,000*3/12= 45,000 220,000*2/12= 36,667
170,000*3/12= 42,500 210,000*3/12= 52,500 220,000*2/12= 36,667
TOTAL 162,500 205,834 245,000

HH, P179,680; MM, P239,360 ; and, AA, P239,360


HH, P179,760; MM, P229,520 ; and, AA, P239,520

HH, P189,680; MM, P239,360 ; and, AA, P269,360

HH, P223,180; MM, P272,060 ; and, AA, P280,760

Question 20 1 / 1 point
AA and BB formed a partnership in 20X2 and made the following investments and capital
withdrawals during the year:

AA BB
Investments Draws Investments Draws
March 1 P30,000 P20,000
June 1 P10,000 P10,000
August 1 P20,000 P2,000
December 1 P5,000

The partnership’s profit and loss agreement provides for a salary of which P30,000 was paid to each
partner for 20X2. AA is to receive a bonus of 10% on net income after salaries and bonus. The
partners are also to receive interest of 8% on average annual capital balances affected by both
investments and drawings. Any remaining profits are to be allocated equally among the partners.
Assuming net income of P60,000 before salaries and bonus, determine how the income wo uld be
allocated among the partners:
SOLUTION:
AA BB TOTAL
SALARY 30,000 30,000 60,000
INTEREST 2,168 800 2,968
RESID. (1,484) (1,484) (2,968)
TOTAL 30,710

NET INCOME:60,000-62,968=(2,986)/2=1,484

AVERAGE CAPITAL BALANCES:


AA BB
30,000*3/12=7,500 20,000*3/12=5,000
20,000*2/12=4,000 10,000*2/12=1,667
40,000*4/12=13,333 8,000*5/12=3,333
35,000*30/31/12=2,267
TOTAL 27,100 10,000

AA, P31,138; BB, P28,862

AA, P33,537; BB, P26,463

AA, P30,633; BB, P29,367

AA, P30,684; BB, P29,316

1 / 1 point
Katherine, Landee, and Mariah are partners with average capital balances during 2013 of P472,500,
P238,650, and P162,350, respectively. The partners receive 10% interest on their average capital
balances; after deducting salaries of P122, 325 to Katherine and P82,625 to Mariah, the residual
profits or loss is divided equally.
In 2013, the partnership had a net loss of P125,624 before the interest and salaries to partners.
SOLUTION
K L M TOTAL
SALARIES 122,325 82,625 204,950
INTEREST 47,250 23,865 16,235 87,350
RESID (139,308) (139,308) (139,308) (417,924)
TOTAL 30,267 (115,443) (40,448) (125,624)

NET LOSS= (125,624)-292,300=-417,924/3=(139,308)


By what amount should Katherine’s and Mariah’s capital account change?

P40,844 decrease; P31,235 decrease


P28,358 decrease; P32,458 decrease

P30,267 decrease; P40,448 decrease

P29,476 decrease; P17,536 decrease

Question 2 1 / 1 point
On January 1, 2020, Ferd and Erik decided to form a partnership. At the end of the year, the
partnership made a net income of P60,000. The capital accounts of the partnership shows the
following transactions.

Ferd, Capital Erik, Capital


Dr. Cr. Dr. Cr.
January 1 - P20,000 - P12,500
April 1 P2,500 - - -
June 1 - - - 5,000
August 1 - 5,000 -
September 1 - - P1,500
October 1 - 2,500 500
December 1 - 2,000 -

Assuming that an interest of 20% per annum is given on average capital and the balance of the
profits is divided equally, the sharing of the profits shall be:

SOLUTION:
FERD ERIK TOTAL
INTEREST 4,200 2,958.2 7,158.2
RESID 26,420.9 26,420.9 52,841.8
TOTAL 30,620.9 29,379.1 60,000

NET INCOME:60,000-7,158.2=52,841.8

AVERAGE CAPITAL BALANCES


FERD ERIK
20,000*3/12=5,000 12,500*5/12=5,208
17,500*4/12=5,833
17,500*3/12=4,375
22,500*2/12=3,750
16,000*1/12=1,333
25,000*2/12=4,167 15,500*3/12=3,875
27,000*1/12=2,250
TOTAL 21,000 14,791

Ferd, P33,600; Erik, P26,400

Ferd, P34,400; Erik, P25,600

Ferd, P30,600; Erik, P29,400

Ferd, P30,000; Erik, P29,700

Question 3 1 / 1 point
Amelville, Brian and Crimson are partners in the accounting firm. Their capital account balances at
year-end were: Amelville, P90,000; Brian, P110,000; Crimson, P50,000. They share profits and losses
in a 4:4:2 ratio, after the following special terms:
1. Partner Crimson is to receive a bonus of 10% of the net income after bonus.
2. Interest of 10% shall be paid on that portion of a partner’s capital in excess of P100,000.
3. Salaries of P10,000 and P12,000 shall be paid to partners Amelville and C, respectively.
Assuming a net income of P44,000 for the year, the total profit share of partner Crimson would be:
SOLUTION:
BONUS OF CRIMSON
.10(44,000-B)=B
B=4,000

A B C TOTAL
SALARIES 10,000 12,000 22,000
INTEREST 1,000 1,000
BONUS 4,000 4,000
RESID(4:4:2) 3,400
TOTAL 19,400

NET INCOME=44,000-27,000=17,000
P16,800

P19,800

P19,400

P7,800

Question 4 1 / 1 point
On January 1, 2020, Krizzia and Hebban have capital balances of P40,000 and P32,000 respectively.
On July 1, 2020 Krizzia invests an additional P8,000 and Hebban withdraws P3,200. Profits and losses
are divided as follow: Hebban is the managing partner and as such shall receive P32,000 salary and
Krizzia shall receive P14,400; both partners shall receive interest of 10% on their beginning capital
balances to offset whatever difference in capital investments they have and any remainder shall be
divided equally.
Income of the Krizzia-Hebban partnership for the year 2020 is P19,200. Krizzia’s share in the net
income is:

SOLUTION:
KRIZZIA HEBBAN TOTAL
SALARY 14,400 32,000 46,400
INTEREST 4,000 3,200 7,200
RESID (17,200)
TOTAL 1,200

NET INCOME=19,200-53,600=(34,400)

P9,600

P1,200

P1,760

P18,400

Question 5 0 / 1 point
Rothe and Sophia was organized and began operations on March 1, 2020. On that date, Rothe
invested P150,000 and Sophia invested computer equipment with current fair value of P180,000.
Because of shortage of cash, on November 1, 2020, Sophia invested additional cash of P60,000 in the
partnership. The partnership contract includes the following remuneration plan:

Rothe Sophia
Monthly salary (recognized as expense) P10,000 P20,000
Annual interest on beginning capital 12% 12%
Bonus on the net profit before salaries 20% -
and interest but after bonus
Balance equally.

The salary was to be withdrawn by each partner in monthly installments. The partnership’s net profit
for 2020 is P120,000.
What are the capital balances of the partners Rothe and Sophia on December, 31 2020?

SOLUTION:
Capital R S
3/1/20x1 150,000 180,000 Profit after sal but before Int. and B 120,000
10/1/20x1 60,000 Salaries 300,000
Profit before sal, int. and B 420,000
B after B .2(420,000-B)=B 70,000
Interest on BEG. Capital 15,000 18,000 Salaries 300,000
Salaries 100,000 200,000 Interest 33,000
Bonus 70,000 Profit after Sal., Int., and B 17,000
Equally 8,500 8,500
Drawings (100,000) (200,000)
243,500 266,500

P86,000; P154,000

P136,000; P350,000

P243,500; P266,500

P87,000; P155,000

Question 6 1 / 1 point
TM Partnership begins its first year of operation with the following capital balances:

Tresha capital P200,000


Myrel capital P100,000
According to the partnership agreement, all profits will be distributed as follows:
a. Tresha will be allowed a MONTHLY salary of P20,000 with P10,000 assigned to Myrel.
b. The partners will be allowed interest equal to 10 percent of the capital balance as of the
first day of the year.
c. Tresha will be allowed a bonus of 10 percent of the net profit after bonus.
d. The remainder will be divided on the basis of the beginning capital for the first year and
equally for the second year.
e. Each partner is allowed to withdraw up to P10,000 a year.
Assume that the net loss for the first year of operations is P15,000 with net income of P55,0 00 in the
subsequent year. Assume further that each partner withdraws the maximum amount from the
business each period.
What is the balance of Tresha’s capital account at the end of the second year?
SOLUTION:
You need to allocate first the net loss/net income to their respective shares before
computing their capital balances.
NOTES
*We will multiply their salary allocated to them by 12 months because the given salary is for
monthly only so we must find their annual salary each year . TAN = 20,000 x 12 ; MAY =
10,000 x 12.
* and their interest allocation is based on their beginning balance each year. Therefore you
will just multiply their beginning balance each year to know the interest allocated to them.
For Example in the First Year, tan has a 200,000 beg balance multiply it to 10% to know the
20,000 interest assigned to him for the first year.
* For the first year operation Tan will not have a bonus because the partnership has a NET
LOSS FOR THE YEAR. YOU WILL NOT ALLOCATE A BONUS IF YOU HAVE A LOSS.
FOR THE SECOND YEAR TAN WILL NOW RECEIVE A BONUS BECAUSE they have
already a net income.
* While the remainder, you will just WORK BACK to find the remainder balance then
allocate it to their respective shares. FOR the 1st yr it will be 2:1 while for the 2nd yr it will
allocate equally.
* I Give you the allocation table of the income(loss) below. Hope it will help Good luck.

1ST YR OF OPERATION TAN. MAY


BEG BALANCE 200,000. 100,000
Less Net loss. (10,000) (5,000)
Less Drawings. (10,000). (10,000)
Capital end. 180,000. 85,000

2ND YR of operation TAN. MAY


Beg bal 180,000. 85,000
Net income(loss). 94,750. (39,750)
Less drawings. (10,000). (10,000)
Capital end. 264,750. 35,250
TAN MAY TOTAL

Salary 240,000 120,000 360,000


Interest (beg bal x 10%) 20,000 10,000 30,000
Bonus - - -
Remainder (270,000)(135,000)(405,000)
Allocation - 1st year (10,000) (5,000) (15,000)

2ND YR
Salary 240,000 120,000 360,000
Interest 18,000 8,500 26,500
Bonus = 10% ( NI - B) 5,000 5,000
B = 10%(55,000-B)
B= 5,500 - 10%B
B + 10%B = 5,500
1.10B =5,500
1.10B÷1.10= 5,500 ÷1.10
B = 5,000
Remainder (168,250)(168,250)(336,500)
Allocation 94,750 (39,750) 55,000

P284,750

P264,750

P180,000

P184,750

Question 7 1 / 1 point
Melody is trying to decide whether to accept a salary of P40,000 or a salary of P25,000 plus a bonus
of 10% of net income after salaries and bonus as a means of allocating profit among the partners.
Salaries traceable to the other partners are estimated to be P100,000. What amount of income
would be necessary so that Melody would consider the choices to be equal?

P330,000

P610,000

290,000

P530,000

Question 8 1 / 1 point
In its first year of operations, Angel and Company, a partnership, made a net income of P40,000
before providing for salaries of P10,000 and P6,000 per annum for Alba and Bana, respectively, as
stipulated in the partnership agreement. Capital contributions are as follows:

Angel P60,000
Berryl 40,000
Cherry 20,000

Assuming that no profit-and-loss ratios are provided in the partnership agreement and there has
been no change in the capital contributions during the year, how much profit share would Angel be
entitled to receive?

P30,000

P20,000

P22,000

P10,000

Question 9 1 / 1 point
ABC’s partnership provided for the following distribution of profits and losses;
“First”. A to receive 10% of the net income up to P2,000,000 and 20% on the amount of excess
thereof;
“Second”, B and C each, are to receive 5% of the remaining income in excess of P3,000,000 after A’s
share as per above and;
“The balance to be divided equally among the partners.”
For the year just ended, the partnership realized a net income of P5,000,000 before distribution to
partners. The share of A is:

P2,600,000

P2,160,000

P2,200,000

P2,000,000

Question 10 1 / 1 point
The partnership agreement of Jellian and Alfredo provides that interest at 10% per year is to be
credited to each partner on the basis of weighted-average capital balances. A summary of Alfredo’s
capital account for the year ended December 31, 2020 is as follows:

Balance, January 1 P140,000.00


Additional investment, July 1 40,000.00
Withdrawal, August 1 (15,000.00)
Balance, December 31 165,000.00

The amount of interest that should credited to Alfredo’s capital account for 2020 is

P17,250

P15,250

P15,375

P16,500

Question 11 0 / 1 point
Kristel, Sophia and Wilchen formed a partnership on January 1, 2020. Each contributed P144,000.
Salaries were to be allowed as follows:

KK P36,000
SS 36,000
WW 54,000

Drawings were equal to salaries and be taken out evenly throughout the year.
With sufficient partnership net income, Kristel and Sophia could split a bonus equal to 25 percent of
partnership net income after salaries and bonus (in no event could the bonus go below zero).
Remaining profits were to be divided as follows: 30% for Kristel, 30% for Sophia, and 40% for
Wilchen.
For the year, partnership total comprehensive income was P144,000.
What are the capital balances of the partners on December 31, 2020?

Kristel, P150,600; Sophia, P150,600; Wilchen, P148,800

Kristel, P151,200; Sophia, P151,200; Wilchen, P149,400

Kristel, P186,120; Sophia, P186,120; Wilchen, P203,760

Kristel, P150,120; Sophia, P150,120; Wilchen, P149,760

Question 12 0 / 1 point
Regen, a partner in the Debora partnership, is entitled to 40% of profits and losses. During 2020,
Regen contributed land to the partnership that cost her P100,000, but had a fair value of P120,000.
Also during 2020, Regen had drawings of P160,000. The balance of Regen’s capital accounts was
P240,000 at the beginning of the year and P300,000 at the end of the year.
What is the partnership’s comprehensive income (loss) for 2020.
P250,000

P(100,000)

P(150,000)

P300,000

Question 13 1 / 1 point
Ara and Bryan entered into a partnership as of March 1, 2020 by investing P125,000 and P75,000,
respectively, they agreed that Ara, as the managing partner, was to receive a salary; P30,000 per
year and a bonus computed at 10% of the net profit after adjustment for the salary; the balance of
the profit was to be distributed in the ratio of their original capital balances. On December 31, 2013,
account balances were as follows:

Cash P70,000 Accounts Payable P60,000


Accounts receivable 67,000 Ara, capital 125,000
Furnitures and fixtures 45,000 Bryan, capital 75,000
Sales returns 5,000 Ara, drawing (20,000)
Purchases 196,000 Bryan, drawing (30,000)
Operating expenses 60,000 Sales 233,000

Inventories on December 31, 2020 were as follows: supplies, P2,500, merchandise, P73,000, prepaid
insurance was P950 while accrued expenses were P1,550. Depreciation rate was 20% per year.
The partners’ capital balances of Ara and Bryan on December 31, 2020, after closing the net profit
and drawing accounts, were:

P135,940; P47,960

P142,350; P47,670

P139,540; P49,860

P139,680; P48,680

Question 14 1 / 1 point
Ferrer, Gleeann, and Hannah form a partnership and agree to maintain average investments of
P2,500,000, P1,250,000 and P1,250,000, respectively. Interest on the excess or deficiency in a capital
contribution is to be computed at 6% per annum. After the interest allowances, Ferrer, Gleeann, and
Hannah are to share any balance in the ratio of 5:3:2. Average amounts invested during the first six
months were as follows: Ferrer, P3,000,000; Gleeann, P1,375,000; and Hannah, P1,000,000. A loss
from operations of P62,500 was incurred for the first six months. How is this loss distributed among
the partners Ferrer, Gleeann, and Hannah?

P18,375; P21,875; P22,250


P21,875; P18,375; P22,250

P12,500; P10,000; P49,500

P31,250; P18,750; P12,500

Question 15 1 / 1 point
On January 1, 2020, Arianne, Bjarn, Cleo and Debora formed ABAKADA Trading Co., a partnership,
with capital contributions as follows: Arianne, P50,000; Bjarn, P25,000; Cleo, P25,000; and Debora,
P20,000. The partnership contract provided that each partner shall receive a 5% interest on
contributed capital, and that Arianne and Bjarn shall receive salaries of P5,000 and P3,000,
respectively. The contract also provided that Cleo shall receive a minimum of P2,500 per annum, and
Debora a minimum of P6,000 per annum, which is inclusive of amounts representing interest and
share of remaining profits. The balance of the profits shall be distributed to Arianne, Bjarn, Cleo, and
Debora in a 3:3:2:2 ratio.
What amount must be earned by the partnership, before any charge for interest and salaries, so that
A may receive an aggregate of P12,500 including interest, salary and share of profits?

P30,667

P30,000

P16,667

P32,333

PARTNERSHIP DISSOLUTION
0 / 1 point
This transaction will most likely be recorded in the partnership books as a transfer within equity.

E withdraws from the partnership when he was bought out by B and F.

C retires and the partnership pays C P80,000 as full settlement of his capital balance.

G dies and his wife receives settlement of G’s interest from the partnership.

A invests P50,000 cash for a 20% interest in the partnership.

Question 2 1 / 1 point
Which of the following transactions or events does not affect the total assets of a partnership?

An old partner retires and his capital balance is settled by the partnership at a lower
amount.

An incoming partner purchases interest from an existing partner.

A new partner is admitted in a partnership when he invested noncash asset to the


partnership.

A partnership is dissolved and its assets and liabilities revalued to fair value.

Question 3 1 / 1 point
When Partner D retires, the partnership paid D an amount which was lower than the balance of his
capital account. Which of the following statements is incorrect?

The other partners’ capital balances increase.

The number of capital accounts in the partnership chart of accounts decreases.

The partnership assets are not affected.

The partnership assets decrease as a result of the retirement of D.

Question 4 1 / 1 point
It is the change in the relation of the partners caused by any partner ceasing to be associated in the
carrying on of the business.

Dissolution

Liquidation

Break-up

Incorporation

Question 5 1 / 1 point
In all cases of dissolution, the partnership assets and liabilities at date of dissolution may need to be
revalued to their fair values. Any revaluation increase or decrease is

Allocated only to the partner ceasing to be associated with the partnership.

Allocated to all of the existing partners as at the date of dissolution.

No revaluation shall be made.

Allocated only to the partners existing after the dissolution.

Question 6 1 / 1 point
The admission of a new partner effected through the purchase of interest in the partnership is

Recorded in the partnership books as a transfer within equity.


Recorded in the partnership books as a debit to cash or other asset and credit to the
incoming partner’s capital account.

Not recorded in its entirety.

Recorded in the partnership books as a transfer from equity to liability.

Question 7 1 / 1 point
After the admission of a new partner, the total partnership capital account increased by the fair value of
the new partner’ net contributions to the partnership. The admission was accounted for

As a purchase of interest

Under the goodwill method

As an investment in the partnership

Under the bonus method

Question 8 1 / 1 point
In the AAA-BBB partnership, AAA and BBB had a capital ratio of 3:1 and a profit and loss ratio of 2:1,
respectively. The bonus method was used to record CCC’s admittance as a new partner. What ratio
would be used to allocate, to AAA and BBB, the excess of CCC’s contribution over the amount credited
to Colter’s capital account?

AAA and BBB’s new relative capital profit and loss ratio

AAA and BBB’s old capital ratio

AAA and BBB’s old profit and loss ratio

AAA and BBB’s new relative capital ratio

Question 9 1 / 1 point
When Mill retired from the partnership of Mill, Yale, and Lear, the final settlement of Mill’s interest
exceeded Mill’s capital balance. Under the bonus method, the excess

Had no effect on the capital balances of Yale and Lear.

Reduced the capital balance of Yale and Lear.

Was recorded as goodwill.

Was recorded as an expense.

Question 10 1 / 1 point
Allen retired from the partnership of Allen, Beck, and, Chale. Allen’s cash settlement from the
partnership was based on new goodwill determined at the date of retirement plus the carrying amount
of the other net assets. As a consequence of the settlement, the capital accounts of Beck and Chale
were decreased. In accounting for Allen’s withdrawal, the partnership could have used the (1) Bonus
method (2) Goodwill method

(1) Yes; (2) Yes

(1) No; (2) No

(1) Yes; (2) No

(1) No; (2) Yes

Question 11 1 / 1 point
Blau and Rubi are partners who share profits and losses in the ratio of 6:4, respectively. On May 1, 20X3,
their respective capital accounts were as follows:

Blau 60,000
Rubi 50,000

On the date, Lind was admitted as a partner with a one-third interest in capital and profits for an
investment of P40,000. The new partnership began with total capital of P150,000. Immediately after
Lind’s admission, Blau’s capital should be
SOLUTION:
LIND:1/3*150,000=50,000-40,000=10,000
10,000*.6=6,000
60,000-6,000=54,000

50,000

56,667

54,000

60,000

Question 12 1 / 1 point
Kern and Pate are partners with capital balances of P60,000 and P20,000, respectively. Profits and losses
are divided in the ratio of 60:40. Kern and Pete decided to form a new partnership with Grant, who
invested land valued at P15,000 for a 20% capital interest in the new partnership. Grant’s cost of the
land was P12,000. The partnership elected to use the bonus method to record the admission of Grant
into the partnership. Grant’s capital account should be credited for
SOLUTION:
15,000+60,000+20,000=95,000
95,000*.20=19,000
19,000

15,000

16,000

12,000

Question 13 0 / 1 point
Dunn and Grey are partners with capital account balances of P60,000 and P90,000, respectively. They
agree to admit Zorn as a partner with a one-third interest in capital and profits, for an investment of
P100,000, after revaluing the assets of Dunn and Grey. Goodwill to the original partners should be (use
PFRSs) in answering this problem)

33,333

50,000

66,667

Question 14 0 / 1 point
Use the following information for the next two questions:
On June 30, 20X3, the condensed balance sheet for the partnership of Eddy, Fox, and Grimm, together
with their respective profit and loss sharing percentages were as follows:

Assets, net liabilities 320,000

Eddy, capital (50%) 160,000


Fox, capital (30%) 96,000
Grimm, capital (20%) 64,000
320,000

Eddy decided to retire from the partnership and by mutual agreement is to be paid P180,000 out of
partnership funds for his interest. No goodwill is to be recorded. After Eddy’s retirement, what are the
capital balances of the other partners? (1) Fox (2) Grimm

SOLUTION:
180,000-160,000=20,000
FOX:96,000- ( 20,000*.3/.5)=84,000
GRIM: 64,000-(20,000*.2/.5)=56,000

(1) 108,000 (2) 72,000

(1) 84,000 (2) 56,000

(1) 120,000 (2) 80,000

(1) 102,000 (2) 68,000

Question 15 1 / 1 point
On June 30, 20X3, the condensed balance sheet for the partnership of Eddy, Fox, and Grimm, together
with their respective profit and loss sharing percentages were as follows:

Assets, net liabilities 320,000

Eddy, capital (50%) 160,000


Fox, capital (30%) 96,000
Grimm, capital (20%) 64,000
320,000
Hamm is admitted as a new partner with a 25% interest in the capital of the new partnership for a cash
payment of P140,000. The bonus method shall be used to record the admission of Hamm. Immediately
after admission of Hamm, Eddy’s capital account balance should be
SOLUTION:
96,000+64,000+140,000+160,000=460,000
460,000*.25=115,000
140,000-115,000=25,000
25,000*.5=12,500
160,000+12,500=172,500

280,000

140,000

172,500

160,000

Question 16 1 / 1 point
On June 30, 20X3, the balance sheet for the partnership of Coll, Maduro, and Prieto, together with their
respective profit and loss ratios, were as follows:

180,000
Assets, at cost

Coll, loan 9,000


Coll, capital (20%) 42,000
Maduro, capital (20%) 39,000
Prieto, capital (60%) 90,000
Total 180,000

Coll has decided to retire from the partnership. By mutual agreement, the assets are to be adjusted to
their fair value of P216,000 at June 30, 20X3. It was agreed that the partnership would pay Coll P61,200
cash from Coll’s partnership interest, including Coll’s loan which is to be repaid in full. No goodwill is to
be recorded. After Coll’s retirement what is the balance of Maduro’s capital account?
SOLUTION:
COLL: 42,000+ .2(216,000-180,000)=49,200
MADURO: 39,000 + .2(216,000-180,000)=46,200
BONUS: 61,200-(49,200+9,000)=3,000
MADURO CAPITAL ACCOUNT: 46,200- (3,000*.2/.8)=45,450

39,000

45,450

36,450

46,200

Question 17 1 / 1 point
The net assets of ABC Partnership is P1,000,000 as of the beginning of the period. Partner A retired from
the partnership on June 30, 20X1. The partnership earned a profit of P300,000 for the six months ended
June 30, 20X1. Partners A, B and C share profits and losses equally. If Partner A was paid P200,000 for his
interest in the partnership, how much is the adjusted net assets of the partnership immediately after
Partner A’s retirement?
SOLUTION:
300,000/3=100,000
200,000-100,000=100,000+1,000,000

1,100,000
900,000

Cannot be determined due to insufficiency of given information

800,000

Question 18 0 / 1 point
The capital balances of Happy and Sad are as follows:

Happy, Capital (20%) 60,000


Sad, Capital (80%) 20,000

The net assets of the partnership approximate fair values. If Angry wants to invest in the partnership for
a 20% interest, (1) how much should Angry invest if the amount of investment must reflect the fair value
of the interest required? (2) how much is the adjusted capital balance of Sad after the admission of
Angry?
SOLUTION:
80,000/.8=100,000
100,000*.2=20,000

(1) 20,000; (2) 24,000

(1) 25,000; (2) 16,000

(1) 20,000; (2) 20,000

(1) 25,000; (2) 20,000

Question 19 1 / 1 point
The capital balances of the partners in Seasons Partnership are as follows:

Rainy, Capital (20%) 40,000


Summer, Capital (40%) 50,000
Cold, Capital (40%) 10,000

Partner Cold became Crazy and was kicked out of the partnership. Cold was paid nothing because of the
crazy things he has done in the partnership. How much is the capital balance of Summer after Cold was
separated from the partnership?
SOLUTION:
10,000*.4/.6=6,667
6,667+50,000=56,667

50,000
56,667

54,000

43,333

Question 20 1 / 1 point
Presented below is the condensed balance sheet of the partnership of KK, LL and MM who are profits
and losses in the ratio of 6:3:1, respectively:

Cash P85,000 Liabilities P80,000


Other assets 415,000 KK, Capital 252,000
LL. Capital 126,000
MM, Capital 42,000
Total P500,000 Total P500,000

The partners agree to sell NN 20% of their respective capital and profit and loss interests for a total
payment of P90,000. The payment by NN is to be made directly to the individual partners. The capital
balances of KK, LL and MM, respectively after admission of NN are:
SOLUTION:
252,000*.80=201,600
126,000*.8=100,800
42,000*.8=33,600

P255,600; P127,800; P42,600

P201,600; P100,800; P33,600

P198,000; P99,000; P33,000

P216,000; P108,000; P36,000

Question 21 1 / 1 point
The capital accounts of the partnership NN, VV and JJ on June 1, 20X1 are presented below with their
respective profit and loss ratios:

NN P139,200 1/2
VV 208,800 1/3
JJ 96,000 1/6

On June 1, 20X1, LL is admitted to the partnership when LL purchased, for P132,000, a proportionate
interest from NN and JJ in the net assets and profits of the partnership. As a result of a transaction LL
acquired a one-fifth interest in the net assets and profits of the firm. What is the combined gain realized
by NN and JJ upon the sale of a portion of their interest in the partnership to LL?
SOLUTION:
139,000+208,800+96,000=444,000
SELLING PRICE: 132,000
LESS INTEREST: 444,000*1/5= 88,800
GAIN 43,200

P62,400

P0

P82,000

P43,200

Question 22 1 / 1 point
The following condensed balance sheet is presented for the partnership of LL, PP and QQ, who share
profits and losses in the ration of 4:3:3 respectively:

Cash P90,000
Other assets 830,000
LL, loan 20,000
P940,000
Accounts payable P210,000
QQ, loan 30,000
LL, capital 310,000
PP, capital 200,000
QQ, capital 190,000
P940,000

Assume that the assets and liabilities are fairly valued on the balance sheet and that the partnership
decides to admit FF as a new partner, with a 20% interest. No goodwill or bonus is to be recorded.
How much should FF contribute in cash or other assets?
SOLUTION:
310,000+200,000+190,000=700,000
(700,000/.8)*.2=175,000

P175,000

P140,000

P177,500
P142,000

Question 23 1 / 1 point
On June 30, 20X1, the statement of financial position for the partnership of CC, MM, and PP, together
with their respective profit and loss ratios, were as follows:

Assets, at cost P180,000


CC, loan 9,000
CC, capital (20%) 42,000
MM, capital (20%) 39,000
PP, capital (60%) 90,000
Total P180,000

CC decided to retire from the partnership. By mutual agreement, the assets are to be adjusted to their
fair value of P216,000 at June 30, 20X1. It was agreed that the partnership would pay CC P61,200 cash
for CC’s partnership interest, including CC’s loan which is to be repaid in full. No goodwill is to be
recorded. After CC’s retirement, what is the balance of MM’s account?

P39,000

P46,200

P39,000

P36,450

Question 24 1 / 1 point
Roy and Gil are partners sharing profits and losses in the ratio of 1:2, respectively. On July 1, 20X1, they
decided to form the R&G Corporation by transferring the assets and liabilities from the partnership to
the Corporation in exchange of its shares. The following is the post-closing trial balance of the
partnership:

Debit Credit
Cash P45,000
Accounts receivable (net) 60,000
Inventory 90,000
Fixed Assets (net) 174,000
Liabilities P60,000
Roy, Capital 94,800
Gil, Capital 214,200
P369,000 P369,000

It was agreed that adjustments be made to the following assets to be transferred to the corporation:

Accounts Receivable P40,000


Inventory 68,000
Fixed Assets 180,600

The R&G Corporation was authorized to issue P100 par preference shares and P10 par ordinary shares.
Roy and Gil agreed to receive for their equity in the partnership 720 ordinary share each, plus even
multiples of 10 shares for their remaining interest.
The total number of shares of preference and ordinary share issued by the Corporation in exchange of
the assets and liabilities of the partnership are (1) preference shares (2) ordinary shares

2,540 shares, 1,500 shares

2,592 shares, 1,440 shares

2,642 shares, 1,440 shares

2,642 shares, 1,550 shares

Question 25 1 / 1 point
Partners Art and Tony, who share equally in profits and losses, have the following balance sheet as of
December 31, 20X1:

Cash P120,000 A/Payable P172,000


A/Receivable 100,000 Accum. dep’n 8,000
Inventory 140,000 Art, capital 140,000
Equipment 80,000 Tony, capital 120,000
Total P440,000 Total P440,000

They agreed to incorporate their partnership, with the new corporation absorbing the net assets after
the following adjustments: provision of allowance for bad debts of P10,000; restatement of the
inventory at its current fair value of P160,000; and, recognition of further depreciation on the
equipment of P3,000. The corporation’s capital stock is to have a par value of P100, and the partners are
to be issued corresponding total shares equivalent to their adjusted capital balances.
The total par value of the shares of capital stock that were issued t partners Art and Tony was:
SOLUTION:
Notes:
• Adjusted net assets = total par value of the shares of capital stock
• Provision of allowance and accumulated depreciation are deductions to net assets
• Net assets= assets-liabilities

Solution:
Adjusted net assets:
440,000+ (160,000-140,000) - 10,000 - 3,000 - 172,000 - 8,000 = 267,000
P260,000

P267,000

P280,000

P273,000

1 / 1 point
The capital accounts of the partnership of Nickolas, Ortiz, and Perez on June 1, 2020 and are presented
below with their respective profit and loss ratios:

Nickolas P139,200 1/2


Joan P208,800 1/3
Raeneil 96,000 1/6

On June 1, 2020, Clessna is admitted to the partnership when he purchased, for P132,000, a
proportionate interest from Nickolas and Joan in the net assets and profits of the partnership. As a
result of a transaction, Clessna acquired a one-fifth interest in the net assets and profits of the firm.
What is the combined gain realized by Nickolas and Joan upon the sale of a portion of their interest in
the partnership to Clessna?
SOLUTION:
139,000+208,800+96,000=444,000
SELLING PRICE: 132,000
LESS INTEREST: 444,000*1/5= 88,800
GAIN 43,200

P43,200

P82,000

P62,400

P0

Question 2 1 / 1 point
Sunshine Partnership had a net income of P8,000,000 for the month ended September 30, 2020.
Sunshine purchased an interest in the Sunshine Partnership of Lheyrie and Danya by paying Lheyrie
P32,000 for half of her capital and half of her 50% percent profit sharing interest on October 1, 2020. At
this time, Lheyrie capital balance was P24,000 and Danya capital balance was P56,000. Lheyrie should
receive a debit to her capital account of:

SOLUTION:
NET INCOME: 8,000,000
.5*24,000=12,000

P26,667

P16,000

P12,000

P20,000

Question 3 1 / 1 point
Partners Adelyn, Euleve and Korina sharing profit and loss based on 4:3:2 ratio have the following
condensed statement of financial position:

Total assets P1,880,000

Liabilities P480,000
Adelyn, capital 620,000
Euleve, capital 400,000
Korina, capital 380,000

Total liabilities and capital P1,880,000

Dorothy will be admitted as a new partner for 20% interest after he pays the three partners with a
minimum of 10%. Thus, the old partner will have to transfer to Dorothy 20% of their interest.

SOLUTION:
620,000+400,000+380,000=1,400,000
1,400,000*.2= 280,000

P376,000
P280,000

P200,000

P350,000

Question 4 0 / 1 point
Partners Andrew, Brian, and Cheddie share profits and losses 50:30:20, respectively. The statements of
financial position at April 30, 2020 follows:

Cash P40,000 Accounts payable P100,000


Other assets 360,000 Andrew, Capital 74,000
Brian, Capital 130,000
Cheddie, Capital 96,000

Total P400,000 Total P400,000

The assets and liabilities are recorded and presented at their respective fair values, Jester to be
admitted as a new partner with a 20% capital interest and a 20% share of profits and losses in exchange
for cash contribution. No bonus is to be recorded. How much cash should Jester contribute?
SOLUTION:
74,000+130,000+96,000=300,000
X/(X+300,000) = .2
X=.2X+60,000
.8X=60,000
X=75,000

P80,000

P72,000

P75,000

P60,000

Question 5 1 / 1 point
The following is the condensed statement of financial position of the partnership Janice, Lalaine and
Berryl who share profits and losses in the ratio of 4:3:3.

Cash P180,000 Accounts payable P420,000


Other assets 1,660,000 Berryl, Loan 60,000
Jo, Receivable 40,000 Janice, Capital 620,000
Lalaine, Capital 400,000
Berryl, Capital 380,000
Total P1,880,000 Total P1,880,000

Assume that the assets and liabilities are fairly valued on the balance sheet and the partnership decides
to admit Maria as a new partner, with a 20% interest. No bonus is to be recorded. How much Maria
should contribute in cash or other assets?

SOLUTION:
620,000+400,000+380,000=1,400,00
X/(1,400,000+X)=.2
X=280,000+.2X
.8X=280,000
X=350,000

P355,000

P350,000

P284,000

P280,000

Question 6 1 / 1 point
Chres and Glyden are partners who share profits and losses in the ratio of 7:3, respectively. On October
5, 2020, their respective capital accounts were as follows:

Chres P35,000
Glyden 30,000

On that date they agreed to admit Sherry as a partner with a one-third interest in the capital and profits
and losses, and upon his investment of P25,000. The new partnership will begin with a total capital of
P90,000. Immediately after Sherry’s admission, what are the capital balances of Chres, Glyden, and
Sherry, respectively?
SOLUTION:
90,000*(1/3)=30,000 Chres:35,000-3,500=31,500
30,000-25,000=5,000 Glyden: 30,000-1,500=28,500
5,000*.7=3,500 Sherry=30,000
5,000*.3=1,500

P31,667; P28,333; P30,000

P31,500; P28,500; P30,000

P30,000; P30,000; P30,000

P50,000; P30,000; P30,000

Question 7 1 / 1 point
On June 30, 2020, the balance sheet for the partnership of Clint, Mareve and Patricia, together with
their respective profit and loss ratio, were as follows:

Assets, at cost P180,000

Clint, loan P9,000


Clint, capital (20%) 42,000
Mareve, capital (20%) 39,000
Patricia, capital (60%) 90,000

P180,000

Clint had decided to retire from the partnership. By mutual agreement, the assets are to be adjusted to
their fair value of P216,000 at June 13, 2020. It was agreed that the partnership would pay Clint P61,200
cash for Clint’s partnership interest, including Clint’s loan which is to be repaid in full. No goodwill is to
be recorded. After Clint’s retirement, what is the balance of Mareve’s capital account?

SOLUTION:
MAREVE: 39,000 + .2 *(216,000-180,000)=46,200
46,200-750=45,450
CLINT: 42,000 + .2 *(216,000-180,000)=49,200
3,000*.2/.8=750
61,200-(49,200+9,000)=3,000

P45,450

P36,450

P46,200
P39,000

Question 8 1 / 1 point
Christen, Danya and Lucy are partners with capital balances on December 31, 2020 of P300,000,
P300,000 and P200,000 respectively. Profit are shared equally. Lucy wishes to withdraw and it is agreed
that she is to take certain furniture and fixtures with second hand value of P50,000 and note for the
balance of her interest. The furniture and fixtures are carried in the books at P65,000. Brand new, the
furniture and fixtures may cost P80,000. Lucy’s acquisition of the second-hand furniture will result to:

SOLUTION:
(65,000-50,000)/3=5,000 REDUCTION

Reduction in capital of P5,000 each for Christen, Danya and Lucy.

Reduction in capital of P7,500 each for Christen and Danya.

Reduction in capital of P10,000 for Lucy.

Reduction in capital of P15,000 each for Christen and Danya.

Question 9 1 / 1 point
On December 31, 2020 the condensed statement of financial position of ABC Partnership is presented
below:

Total assets P180,000

Ann loan P10,000


Ann, capital 45,000
Bjarne, capital 40,000
Charlotte, capital 85,000

Total liabilities and capital P180,000

Ann, Bjarne and Charlotte share profits and losses in the ratio of 3:2:1 respectively, It was agreed among
the partners that Ann retires from the partnership and the partnership’s assets to be adjusted to their
fair value of P210,000. The partners further agreed to pay Ann P64,000 cash for her total interest in the
partnership.
What is the capital balance of Charlotte after the retirement of Ann?
SOLUTION:
CHARLOTTE: 85,000+ 1/6 ( 210,000-180,000)=90,000 CHARLOTTE: 90,000+2,000=92,000
6,000*(1/6)/.5=2,000
ANN: 45,000+ 3/6 (210,000-180,000)=60,000
64,000- (60,000+10,000)=-6,000

P92,000

P27,000

P35,000

P33,000

Question 10 0 / 1 point
Lyca, Mary and Noerine were partners with capital balances on January 2, 2020 of P300,000, P200,000
and P100,000, respectively. On July 1, 2020 Lyca retires from the partnership. On the date of retirement
the partnership net loss is P60,000 and the partners agreed that certain asset is to be revalued at
P80,000 from its original cost of P50,000. The partners agreed further to pay Lyca P225,000 in
settlement of her interest. The remaining partners continue to operate under a new partnership, MN
partnership.
What is the total capital of MN partnership?
SOLUTION:
300,000+200,000+100,000=600,000
60,000 *.5=30,000 60,000*2/6=20,000 60,000*1/6=10,000
LYCA: 300,000 + .5(80,000-50,000)= 315,000
MARY: 200,000+ 2/6 (80,000-50,000)= 210,000-20,000+40,000=230,000
NOERINE: 100,000 + 1/6 (80,000-50,000)= 105,000-10,000+20,000=115,000
MN PARTNERSHIP: 230,000+115,000=345,000
60,000* (2/6)/.5=40,000
60,000* (1/6)/.5=20,000
225,000-(315,000-30,000)=-60,000

P340,000

P285,000

P345,000

P280,000
1 / 1 point
Capital balances of ABC Partnership are as follows:

P 100,000
Ant, Capital (50%)

200,000
Boar, Capital (30%)

100,000
Cat, Capital (20%)

P 400,000

Boar assigns half of his interest in the partnership to Dog for P90, 000
cash. Dog pays directly to Boar. What is the total capital of ABC
partnership after the assignment of interest of Boar to Dog?

490,000

200,000

400,000

310,000

Question 2 1 / 1 point
Rodel, Mahal, Gerry and Mura formed the Enchanted Partnership
on January 1, 2020 with cash contributions from Rodel,
P100,000; Mahal, P50,000; Gerry, P50,000 and Mura P40,000.
The partnership agreement provides that profits and losses are
to be distributed as follows:

a. Interest at 5% of capital contribution of the partners.

b. Salary of P10,000 to Rodel and P6,000 to Mahal.


c. Gerry to receive a minimum of P5,000 per annum from the
partnership and Mura a Minimum of P12,000 per annum, both
including amounts allowed as interest on capital and their
respective shares in profits,

d. The balance of the profit is to be shared in the following


proportions: Rodel, 30%; Mahal, 30%; Gerry, 20% and Mura 20%.

How much should be the partnership profit in order to give and


satisfy the minimum share of Gerry?

P48,000

P62,000

P78,000

P58,000

Question 3 1 / 1 point
Nobita and Damulag are partners with capital balances of
P30,000 and P 40,000 and sharing profits and losses 40% and
60% respectively. If Suneo is admitted as partner paying P
20,000 in exchange for 50% of Nobita equity, the entry to record
this should be:

DR Nobita, Capital 15,000

CR Suneo, Capital 15,000

DR Cash 15,000

DR Other Assets 5,000


CR Suneo, Capital 20,000

DR Nobita, Capital 20,000

CR Cash 20,000

DR Cash 20,000

CR Suneo, Capital 10,000

CR Nobita, Capital 10,000

Question 4 0 / 1 point
Alejo, Garciano and Almodiel are partners with capital balances
of P 180,000, 100,000 and 120,000 respectively. Urbano is
admitted into the partnership with a one-fourth interest upon
payment of P 160,000. If the old partners share profits and
losses in the ratio of 2/5, 2/5 and 1/5, then the capital account
of Almodiel after the admission of Urbano will show a balance of

124,000

100,000

114,000

105,000

Question 5 0 / 1 point
The income statement of Sarah-Fatima Partnership for the year
ended Dec. 31, 2020 appear below:

Sales P600,000
Less: Cost of goods sold 380,000
Gross profit 220,000
Less: Operating expenses 60,000
Net income P160,000
Additional information:

a. Sarah and Fatima began the year with a capital balance of


P81,600 and P224,000, respectively.

b. On April 1, Sarah invested an additional P30,000 into the


partnership and on August 1, Fatima invested an additional
P40,000 into the partnership.

c. Throughout 2020, each partner withdrew P800 per week in


anticipation of partnership net income. The partners agreed that
these withdrawals are not to be included in the computation of
average capital balances for purposes of income distributions.

Sarah and Fatima have agreed to distribute partnership net


income according to the following:

Sarah Fatima
a. Interest on average capital balances 6% 6%
b. Bonus on net income before the bonus 10%
but after interest on average capital
balances
c. Salaries P50,000 P60,000
d. Residual (if positive) 70% 30%
e. Residual (if negative) 50% 50%

The ending capital balance of Fatima:

P301,456

P301,716

P301,836

P304,656

Question 6 1 / 1 point
On June 30, 2020, the balance sheet for the partnership of
ALLAN, CHARISSE and FRANDEL and their profit and loss ratios
were as follows:

Assets, at cost P750,000

ALLAN, loan P 37,500


ALLAN, capital (20%) 175,000
CHARISSE, capital (20%) 162,500
FRANDEL, capital (60%) 375,000
Total Equities P750,000

ALLAN decided to retire from the partnership and by mutual


agreement, the assets were adjusted to their current fair value of
P900,000. The partnership paid P255,000 cash for ALLAN’s
equity in the partnership, exclusive of the loan which was repaid
in full.

The capital balances of CHARISSE and FRANDEL, respectively,


after ALLAN’s retirement from the partnership was:

150,000; 337,500

180,000; 427,500

192,500; 465,000

205,000; 502,500

Question 7 1 / 1 point
When Jane retired from the partnership of Jane, Jeng and Jing the
final settlement of her interest exceeded her capital balance.
Under the bonus method, the excess:
Was recorded as goodwill

Reduced the capital balances of continuing partners

No effect on capital balances of continuing partners

Was recorded as expense

Question 8 0 / 1 point
Aquino’s interest in the partnership is P 110,000. Jerabuys
Aquino’s interest for P 120,000. How much is the capital balance
of Jera after the purchase?

120,000

110,000

130,000

140,000

Question 9 1 / 1 point
JJ admits RR as a partner in business. Accounts in the ledger for
JJ on November 30, 2020, just before the admission of RR, show
the following balances: Cash - P6,800; Accounts receivable -
P14,200; Merchandise inventory – P20,000; Accounts payable -
P8,000; JJ, capital - P33,000.

It is agreed that for purposes of establishing JJ’s interest the


following adjustments shall be made: (a) An allowance for
doubtful accounts of 3% of accounts receivable is to be
established; (b) The merchandise inventory is to be valued at
P23,000; and (c) Prepaid salary expenses of P600 and accrued
rent expense of P800 are to be recognized.

RR is to invest sufficient cash to obtain in a 1/3 interest in the


partnership.

(1) JJ’s adjusted capital before the admission of RR; and (2) the
amount of cash investment by RR.

(1) P35,347; (2) P11,971

(1) P36,374; (2) P18,487

(1) P35,374; (2) P17,687

(1) P28,174; (2) P14,087

Question 10 0 / 1 point
When a partner retires and receives less than his capital balance,
using bonus method how should the difference be treated?

The difference should be credited to the remaining partners in their profit and
loss ratio

The difference should be debited to all the partners in their profit and loss ratio

The difference should be debited to the remaining partners in their remaining


profit and loss ratio

The difference should be credited to all partners in their profit and loss ratio

Question 11 0 / 1 point
Manolo, Jane, Joshua and Loisa own a publishing company that
they operate as a partnership. Their agreement includes the
following:
a. Manolo will receive a salary of P20,000 and a bonus of 3% of
income after all the bonuses
b. Jane will receive a salary of P10,000 and a bonus of 2% of
income after all the bonuses
c. All partners are to receive the following: Manolo – P5,000;
Jane – P4,500; Joshua – P2,000; and Loisa – P4,700, representing
10% interest on their average capital balances.
d. Any remaining profits are to be divided equally among
partners
e. Partnership reports a profit of P40,000.
How much is Jane’s share in the profit if the profit is distributed
in the following order of priority: interest on invested capital,
then bonuses, then salary, and then according to profit and loss
percentage?

P12,560

P12,830.75

P12,443

P13,235.75

Question 12 1 / 1 point
If the partners have not drawn up an agreement, then they must
share profits and losses

Equally

According to capital contribution


By any means that will save taxes

By any appropriate ratio

Question 13 0 / 1 point
The following Balance Sheet for the partnership of C, I and G
were taken from the books on October 1, 2020.

Assets Liabilities and Capital


Cash P100,000 Liabilities P200,000
Other Assets 400,000 C, Capital 120,000
I, Capital 95,000
G, Capital 85,000
Total Assets 500,000 Total Liabilities and Capital 500,000

The partners agreed to distribute profits as follows:

1. Annual salaries to C and I of P5,000 each

2. Annual interest of 5% on beginning capital

3. Bonus of 15% to C based on income after salaries, interest


and bonus

4. Remaining profit: 25% to C, 35% to I and 40% to G

The partnership began its operations on Oct. 1, 2020 and net


income as of Dec. 31, 2020 is P69,500. Which of the following is
true?

I’s total share in the net income is P21,688

Net Income after salaries, interest and bonus is P38,696


G’s share on the profit after salaries, interest and bonus is P13,543

The bonus to C is P5,804

Question 14 1 / 1 point
When admitting a new partner into an existing partnership, any
allocation of asset revaluation to the old partners is based on

An equal distribution among partners

The relative capital balances of the partners

The profit and loss ratio

The fair values of the assets each partner has contributed to the partnership

Question 15 1 / 1 point
Jonathan Yap is trying to decide whether to accept a salary of
P20,000 or a salary of P12,500 plus a bonus of 10% of net
income after salary and bonus as a means of allocating profit
among the partners. Salaries traceable to the other partners are
estimated to be P50,000. What amount of income would be
necessary so that Jonathan Yap would consider the choices to be
equal?

P145,000

P132,500

P82,500

P152,500

Question 16 0 / 1 point
Tonya, Tonye, and Tonyo are forming a new partnership. Tonya
is to invest cash of P100,000 and stapling equipment originally
costing P120,000 but has a second-hand value in the market at
P50,000. Tonye is to invest cash of P160,000, while Tonyo,
whose family is engaged in selling stapling equipment to be
used by the partnership with a regular price of P120,000 but
which cost their family’s business P100,000. In addition, Tonyo
invest cash amounting to P50,000. Partners agree to share
profits equally. The capital balances upon formation are:

Tonya, P150,000; Tonye, P160,000; and Tonyo, P170,000

Tonya, P176,666; Tonye, P176,666; and Tonyo, P176,668

Tonya, P220,000; Tonye, P160,000; and Tonyo, P150,000

Tonya, P160,000; Tonye, P160,000; and Tonyo, P160,000

Question 17 1 / 1 point
When Mill retired from the partnership of Mill, Yale, and Lear,
the final settlement of Mill’s interest exceeded Mill’s capital
balance. Under the bonus method, the excess

Was recorded as goodwill.

Had no effect on the capital balances of Yale and Lear.

Was recorded as an expense.

Reduced the capital balances of Yale and Lear.


Question 18 1 / 1 point
Alex and Carlo are partners agreeing to allow monthly salaries
(P6,000 and P5,000, respectively), 6% interest on the capital
investment at the beginning of the year (P300,000 and
P230,000, respectively) and on the remaining balance, to be
equally shared. The first year registered a net income of
P100,000. Partners should be entitled to:

Alex, P50,000 and Carlo, P50,000

Alex, P58,100 and Carlo, P41,900

Alex, P56,600 and Carlo, P43,400

Alex, P54,500 and Carlo, P45,500

Question 19 1 / 1 point
When property other than cash is invested in a partnership, at
what amount should the noncash property be credited to the
contributing partner’s capital account?

Assessed valuation for property tax purposes.

Contributing partner’s tax basis.

Fair value at the date of contribution.

Contributing partner’s original cost.

Question 20 1 / 1 point
Recom invests P 160,000 in a partnership for a ¼ interest. Prior
to Recom’s admission, the partnership had two partners with
capital balances of P 190,000 each. If no asset revaluation is
recognized prior to Recom’s admission, what amount is credited
to his capital account?

135,000

170,000

187,000

160,000

Question 21 1 / 1 point
Before the withdrawal of Alice from their partnership, the
partners agreed to adjust assets to their fair values.
Accordingly, the appraisal increase was credited to

Income Summary.

Partners’ Capital Accounts.

Appraisal Capital.

Deferred Credit.

Question 22 1 / 1 point
The partnership of Navigar, De Leon and Bacol have capital
account balance as of: Navigar, P 70,000; De Leon, P 100,000;
Bacol, P 80,000. Their profit and loss ratios are 30%, 50%, and
20% respectively. With the consent and knowledge of Navigar
and De Leon, Bacol sold his interest to Dalangin. Bacol was paid
P92,000 in cash. The new capital balances of (1) Navigar, (2) De
Leon and (3) Dalangin would be
(1) 73,800; (2) 106,000; (3) 86,000

(1) 70,000; (2) 100,000; (3) 92,000

(1) 70,000; (2) 100,000; (3) 80,000

(1) 70,000; (2) 80,000; (3) 92,000

Question 23 1 / 1 point
This refers to the changes in relations of the partners caused by
any of the partner.

Liquidation

Dissolution

Formation

Operation

Question 24 1 / 1 point
Four individuals who were previously sole proprietors form a
partnership. Each partner contributes inventory and equipment
for use by the partnership. What basis should the partnership
use to record the contributed assets?

Paid-in capital and retained earnings accounts.

Preferred and common stock accounts.

Paid-in capital, retained earnings, and dividend accounts.

Retained earnings account.

Question 25 0 / 1 point
Alma and Becca have just formed a partnership. Alma
contributed cash of P176,400 and office equipment that cost
P75,600. The equipment had been used in his sole
proprietorship and had been 70% depreciated, the current value
of the equipment is P50,400. Alma also contributed a note
payable of P16,800 to be assumed by the partnership. Alma is
to have a 30% interest in the partnership. Becca contributed
P256,000 land at fair market value. Becca should make
additional investment of:

P234,000

P256,000

P210,000

P490,000

Question 26 1 / 1 point
Which of the following conditions constitutes a legal dissolution
of a partnership?

Admission of a partner

All of the above

Retirement of a partner

Death of a partner

Question 27 1 / 1 point
Rhodelio, Arch and Ryan are partners sharing profits and losses
in the ratio of 4;3;3 respectively. The balanced sheet of their
partnership as of December 31, 2012 as follows:

Cash P100,000 Liabilities P80,000


Other Assets 260,000 Rhodelio, Capital 120,000
Arch, Capital 80,000
Ryan, Capital 80,000
Total Assets P360,000 Total Liabilities and Capital P360,000

All the partners agree to admit Bhe as 1/5 partner in the


partnership without any asset revaluation nor bonus. Bhe shall
contribute assets amounting to:

56,000

70,000

20,000

120,000

Question 28 0 / 1 point
The partnership of Star, Moon and Sun has reached an impasse
as Sun is no longer willing to contribute amount of time and
effort to the partnership that he had previously given. The
partners share profits and losses in the ratio of 3:3:4
respectively. The partners have the following capital balances
just prior to Sun’s withdrawal from the partnership.

Star P45, 000


Moon 35,000
Sun 20,000
If Moon purchases Sun’s interest from Sun for P 32,000 and no
asset revaluation is recorded, the balance of Moon capital
account immediately after the withdrawal of Sun is

P 55,000

P 50,000

P 61,000

P 41,400

Question 29 0 / 1 point
Rodel, Mahal, Gerry and Mura formed the Enchanted Partnership
on January 1, 2020 with cash contributions from Rodel,
P100,000; Mahal, P50,000; Gerry, P50,000 and Mura P40,000.
The partnership agreement provides that profits and losses are
to be distributed as follows:

a. Interest at 5% of capital contribution of the partners.

b. Salary of P10,000 to Rodel and P6,000 to Mahal.

c. Gerry to receive a minimum of P5,000 per annum from the


partnership and Mura a Minimum of P12,000 per annum, both
including amounts allowed as interest on capital and their
respective shares in profits,

d. The balance of the profit is to be shared in the following


proportions: Rodel, 30%; Mahal, 30%; Gerry, 20% and Mura 20%.
How much is the partnership profit if Rodel received an
aggregate of P24,000 including interest, salary and share in
profits?

P48,000

P78,000

P58,000

P62,000

Question 30 0 / 1 point
Cutie partnership had a net income of P 2,000 for the month
ended September 30, 2012. Jr purchased an interest in the DJ
partnership of Yano and Louis by paying Yano P 8,000 for half of
her capital and half of his 50% profit sharing interest on October
1, 2012. At this time Yano capital balance was P 6,000 and Louis
capital balance was P 14,000. Jr should receive a credit to her
capital account balance of:

4,000

5,000

6,667

3,000

Question 31 1 / 1 point
The Articles of Partnership of A and E had the following
provisions stipulated:
a. Annual salary of P60,000 each.

b. Bonus to A of 20% of the net income after partners’


salaries, the bonus being treated as an expense.

c. Balance to be divided equally.

The partnership reported a net income of P360,000 after


partners’ salaries but before bonus. How much is the share of E
in the profit?

P210,000

P150,000

P90,000

P60,000

Question 32 0 / 1 point
The partnership agreement of Paul, Simon and Peter provides for
the division of net income as follows:

• Simon, who manages the partnership is to receive an


annual salary of P120,000.

• Each partner is to be allowed interest at 10% on ending


capital.

• Balance is to be divided 40:25:35.


During 2008, Paul invested an additional P90,000 in the
partnership. Simon made an additional investment of P75,000
and withdrew P110,000 and Peter withdrew P60,000. No other
investments or withdrawals were made during 2008. On January
1, 2008, the capital balances were Paul, P300,000; Simon,
P410,000; and Peter, P220,000. Total capital at year-end was
P600,000. Compute the capital balances of (1) Paul, (2) Simon
and (3) Peter at year-end:

(1) P214,000; (2) P410,250; (3) P24,250

(1) P390,000; (2) P375,000; (3) (P165,000)

(1) P214,000; (2) P398,125; (3) (P12,125)

(1) (P176,000); (2) P948,125; (3) (P172,125)

Question 33 1 / 1 point
On December 31, 2020 the balance sheet for the partnership of
Price, Water and house, together with their respective profit and
loss ratios were as follows:

Assets at cost P 180,000

Price, loan 9,000


Price, Capital(20%) 42,000
Water, Capital (20%) 39,000
House, Capital(60%) 90,000
Total P 180,000

Price has decided to retire from the partnership. By mutual


agreement, the assets are to be adjusted to their fair value of P
216,000 at this date. It was agreed that the partnership would
pay Price P 61,200 cash for his interest, including Price’s loan
which is to be repaid in full. No revaluation is to be recorded.
After Price’s, retirement, what is the balance of Water’s capital
account?

P 36, 450

P 26, 450

P 45, 450

P 54,450

Question 34 1 / 1 point
Among the various options available for determining the
partners share of net income are the following except:

Stated ratio

Capital contributions

Loans to the partnership

Capital contributions and service to the partnership

Question 35 0 / 1 point
Joy and Eve form a partnership and have capital balances of P
100,000 and P 200,000 respectively. If they agree to admit
Hershey into the partnership, how much will she have to invest
to have a ¼ interest?

100,000
75,000

125,000

50,000

Question 36 0 / 1 point
The income statement of Sarah-Fatima Partnership for the year
ended Dec. 31, 2020 appear below:

Sales P600,000
Less: Cost of goods sold 380,000
Gross profit 220,000
Less: Operating expenses 60,000
Net income P160,000

Additional information:

a. Sarah and Fatima began the year with a capital balance of


P81,600 and P224,000, respectively.

b. On April 1, Sarah invested an additional P30,000 into the


partnership and on August 1, Fatima invested an additional
P40,000 into the partnership.

c. Throughout 2020, each partner withdrew P800 per week in


anticipation of partnership net income. The partners agreed that
these withdrawals are not to be included in the computation of
average capital balances for purposes of income distributions.

Sarah and Fatima have agreed to distribute partnership net


income according to the following:

Sarah Fatima
a. Interest on average capital balances 6% 6%
b. Bonus on net income before the bonus 10%
but after interest on average capital
balances
c. Salaries P50,000 P60,000
d. Residual (if positive) 70% 30%
e. Residual (if negative) 50% 50%

The share of Sarah and Fatima on the net income:

P80,564 and P79,436, respectively.

P77,870 and P82,130, respectively.

P80,944 and P79,056, respectively.

P80,684 and P79,316, respectively.

Question 37 0 / 1 point
The equity accounts of the partnership of MAYBELLE and NICK at
March 31, 2013 are as follows:

MAYBELLE, Capital P256,000


NICK, Capital 128,000
MAYBELLE, Loan (credit 24,000
NICK, Drawing (debit) 12,000

The partners share profits and losses in the ratio of 3:2,


respectively. The partnership is in desperate need of cash, and
the partners agree to admit YENER as a partner with a 1/3
interest in the capital and profits & losses upon her investments
of P96,000.

Immediately after YENER’s admission, what should be the capital


balances of MAYBELLE, NICK and YENER, respectively?
P217,600; P102,400; P160,000

P213,333; P102,400; P160,000

P239,200; P 88,800; P164,000

P192,000; P192,000; P192,000

Question 38 0 / 1 point
A partner’s drawing account is, in substance

A contra-capital account

A capital account

A salary expense account

A loan account (a loan from the partnership)

Question 39 0 / 1 point
Partners F, G and H share profits and losses 5:3:2, respectively,
and their balance sheet on October 30, 2020 follows:

Cash P 160,000 Accounts payable P 400,000


Other assets 1,440,000 F, capital 296,000
G, capital 520,000
H, capital 384,000
P1,600,000 P1,600,000

The assets and liabilities are recorded at their current fair value.
I is to be admitted as a new partner with a 20% interest in capital
and earnings. F was credited a bonus of P10,000.

How much cash should I contribute?


P325,000

P320,000

P305,000

P300,000

Question 40 1 / 1 point
The trial balance of POWER HOUSE Partnership is as follows:

Debit Credit
Cash P25,000
Accounts receivable (net) 30,000
Inventory 50,000
Equipment (met) 95,000
Accounts payable P50,000
JAYVEE, Capital 100,000
SHENEN, Capital 50,000
Total P200,000 P200,000

JAYVEE and SHENEN decide to incorporate their partnership. The


partnership's books will be closed, and new books will be used
for POWER HOUSE Corporation. The following additional
information is available:
1. The estimated fair values of the assets follow:

Accounts receivable P26,000


Inventory 50,000
Equipment 84,000

2. All assets and liabilities are transferred to the corporation.


3. The ordinary shares is P10 par. JAYVEE and SHENEN receive a
total of 10,000 shares.
4. The partners share profits and losses in the ratio 7:3.
Based on the preceding information, the journal entry on POWER
HOUSE Corporation's books to record the assets and the
issuance of the ordinary shares will include a credit to Share
Premium for:

P0.

P50,000.

P81,000.

P31,000.

Question 41 0 / 1 point
The following capital accounts pertain to BABE MAGNETS
Partnership:

Capital P/L Ratio


BRIAN P200,000 40%
GAB 240,000 60%

HENO is admitted by purchase of one-half interest of both


BRIAN and GAB, for P240,000.

The P240,000 is divided between BRIAN and GAB as follows:

BRIAN, P109,090; GAB, P130,910

BRIAN, P 96,000; GAB, P144,000

BRIAN, P120,000; GAB, P120,000


BRIAN, P108,000; GAB, P132,000

Question 42 0 / 1 point
Rodel, Mahal, Gerry and Mura formed the Enchanted Partnership
on January 1, 2020 with cash contributions from Rodel,
P100,000; Mahal, P50,000; Gerry, P50,000 and Mura P40,000.
The partnership agreement provides that profits and losses are
to be distributed as follows:

a. Interest at 5% of capital contribution of the partners.

b. Salary of P10,000 to Rodel and P6,000 to Mahal.

c. Gerry to receive a minimum of P5,000 per annum from the


partnership and Mura a Minimum of P12,000 per annum, both
including amounts allowed as interest on capital and their
respective shares in profits,

d. The balance of the profit is to be shared in the following


proportions: Rodel, 30%; Mahal, 30%; Gerry, 20% and Mura 20%.

How much should be the partnership profit in order to give and


satisfy the minimum share of Mura?

P48,000

P58,000

P62,000

P78,000

Question 43 0 / 1 point
C2 partnership had a net income of P24,000 for the month
ended September 30, 2012. Carreon purchased an interest in
the C2 partnership of Calvo and Calma by paying Calvo P96,000
for half of his capital and half of his 50% profit sharing interest.
At this time, the capital balance ofCalvo was P 72,000 and the
capital balance of Calma was P 168,000. Carreon should receive
a credit to his capital account of:

24,000

42,000

36,000

28,000

Question 44 1 / 1 point
On June 30, 2020, the balance sheet of Naruto, Sasuke and
Sakura together with their respective profit and loss sharing
percentages was as follows:

Assets, net of liabilities P 320,000


Naruto, Capital (50%) P 160,000
Sasuke, Capital (30%) 96,000
Sakura, Capital (20%) 64,000

Naruto decided to retire from the partnership and by mutual


agreement is to be paid P 180,000 out of partnership funds.
Revaluation of assets took place. After Naruto’s retirement, the
capital balances of (1) Sasuke and (2) Sakura are:

P102,000; P68,000
P84,000; P56,000

P108,000; P72,000

P120,000; P80,000

Question 45 1 / 1 point
When NANA retired from the partnership of NANA, NINA, and
NONA, the final settlement of NANA’s interest exceeded her
capital balance. Under the bonus method, the excess is

Recorded as an expense.

Of no effect to the capital accounts of Nina and Nona

Deducted from the capital account balances of Nina and None.

Recorded as goodwill.

PARTNERSHIP LUMP-SUM LIQUIDATION


1 / 1 point
Oliver, Patrick & Quincy LLP, is beginning liquidation. It has no cash, total liabilities of P60,000
including a P10,000 loan payable to Patrick, and equal partners’ capital account balances of P40,000.
The income-sharing ratio is 5:1:4, respectively. If a portion of the noncash assets with a carry ing
amount of P140,000 realizes P120,000, the cash payment that Patrick receives
Some other amount

P44,000

P53,000

P20,000

Question 2 1 / 1 point
The balance sheet given below is presented for the partnership of Janet, Anton, and Millet:

Cash P60,000 Liabilities P80,000


Noncash assets 150,000 Janet, capital 80,000
Anton, capital 30,000
Millet, capital 20,000
Total P210,000 Total P210,000

The partners are share profits and losses in the ratio of 5:3:2, respectively. The partners agreed to
dissolve the partnership after selling the other assets for P50,000. On dissolution of the partnership,
Janet should receive:

P30,000

P10,000

P0

P80,000

Question 3 1 / 1 point
who share profits and losses in the ratio 5:3:2, respectively:

Cash P60,000
Other assets 640,000
Liabilities 140,000
Anders, Capital 280,000
Barnes, Capital 28,000
Crowley, Capital 252,000

The partners decide to liquidate the partnership. The other assets are sold for P500,000. How should
the available cash be distributed?

Anders, P210,000; Barnes, P-0-; Crowley, P210,000

Anders, P200,000; Barnes, P-0-; Crowley, P220,000


Anders, P280,000; Barnes, P28,000; Crowley, P252,000

Anders, P210,000; Barnes, P-0-; Crowley, P224,000

Question 4 1 / 1 point
Phil, Harry, and Bill are partners in a company that is being liquidated. The three partners have
capital account balances of P35,000, P28,000, and P27,000. In addition, the partnership previously
loaned Bill P5,000 and Phil had loaned the partnership P10,000. Assuming the right of offset is
applied, what is the net capital position of (1) Phil, (2) Harry, and (3) Bill when the liquidation begins?

(1) P35,000; (2) P28,000; (3) P27,000

(1) P25,000; (2) P28,000; (3) P32,000

(1) P40,000; (2) P28,000; (3) P17,000

(1) P45,000; (2) P28,000; (3) P22,000

Question 5 1 / 1 point
Rick, Mary, and Fran are partners in a company that is being liquidated. The three partners have
capital account balances of P46,000, P39,000, and P29,000. In addition, the partnership previously
loaned Mary P15,000 and Fran had loaned the partnership P10,000. Assuming the right of offset is
applied, what is the net capital position of (1) Rick, (2) Mary, and (3) Fran when the liquidation
begins?

(1) P46,000; (2) P29,000; (3) P44,000

(1) P46,000; (2) P39,000; (3) P29,000

(1) P46,000; (2) P24,000; (3) P39,000

(1) P46,000; (2) P54,000; (3) P19,000

Question 6 1 / 1 point
A local partnership has assets of cash of P5,000 and a building worth P80,000. All liabilities have been
paid and the partners are all insolvent. The partners capital accounts are as follows Harry P40,000,
Landers P30,000, and Waters P15,000. The partners share profits and losses 4:4:2. If the building is
sols for P50,000, how much cash will Harry receive in the final settlement?

P28,000

P9,000

P18,000

P5,000

P55,000
Question 7 1 / 1 point
The year-end balances sheet and residual profit and loss sharing percentages for the Lang, Maas, and
Neal partnership on December 31, 20X5, are as follows:

Cash P30,000 Accounts payable P200,000


Loan to Lang 40,000 Loan from Maas 50,000
Other assets 480,000 Lang, capital (25%) 70,000
Maas, capital (25%) 80,000
Neal, capital (50%) 150,000
Total assets P550,000 Total liabilities and equity P550,000

The partners agree to liquidate the business and distribute cash when it becomes available. A cash
distribution plan for Lang, Maas and Neal partnership will show that cash available, after outside
creditors are paid, will initially go to

Neal in the amount of P90,000.

Maas in the amount of P45,000.

Lang in the amount of P20,000.

Maas in the amount of P55,000.

Question 8 1 / 1 point
The Abrams, Bartle and Creighton partnership began the process of liquidation with the following
balance sheet:

Cash P16,000 Liabilities P150,000


Noncash assets 434,000 Abrams, capital 80,000
Bartle, capital 0,000
Creighton, capital 130,000
Total P450,000 Total Liabilities and Equities P450,000

Abrams, Bartle and Creighton share profits and losses in a ratio of 3:2:5. Liquidation expenses are
expected to be P12,000. If the non-cash assets were sold for P234,000, what amount of the loss
would have been allocated to Bartle?
The Abrams, Bartle and Creighton partnership began the process of liquidation with the following
balance sheet:

Cash P16,000 Liabilities P150,000


Noncash assets 434,000 Abrams, capital 80,000
Bartle, capital 0,000
Creighton, capital 130,000
Total P450,000 Total Liabilities and Equities P450,000
Abrams, Bartle and Creighton share profits and losses in a ratio of 3:2:5. Liquidation expenses are
expected to be P12,000. If the non-cash assets were sold for P234,000, what amount of the loss
would have been allocated to Bartle?

P48,000

P42,400

P40,000

P43,100

P46,800

Question 9 1 / 1 point
The Keaton, Lewis and Meador partnership had the following balance sheet just before entering
liquidation:

Cash P10,000 Liabilities P130,000


Noncash assets 300,000 Keaton, capital 60,000
Lewis, capital 40,000
Meador, capital 80,000
Total P310,000 Total P310,000

Keaton, Lewis and Meador share profits and losses in a ratio of 2:4:4. Non-cash assets were sold for
P180,000. Liquidation expenses were P10,000. Assume that Lewis was personally insolvent and could
not contribute any assets to the partnership, while Keaton and Meadona were both solvent. What
amount of cash would Keaton have received from the distribution of partnership assets?

P38,000

P30,000

P24,000

P34,000

Question 10 1 / 1 point
The Henry, Isaac and Jacobs Partnership was about to enter liquidation with the following account
balances:

Cash P90,000 Liabilities P60,000


Noncash assets 300,000 Keaton, capital 80,000
Lewis, capital 110,000
Meador, capital 140,000
Total P390,000 Total P390,000
Estimated expenses of liquidation were P5,000. Henry, Isaac and Jacobs shared profits and losses in a
ratio of 2:4:4. Before liquidating any assets, the partners determined the amount of cash available
for safe payments. How should the cash be distributed?

In a ratio of 1:2 between Henry and Jacobs

P15000 to Henry and P10,000 to Jacobs

P18,333 to Henry and P16,667 to Jacobs

In a ratio of 1:2:2 among the partner

Question 11 1 / 1 point
liquidated and, prior to the liquidation process, the partnership balance sheet was as follows:

Cash P60,000 Gilligan, Capital P216,000


Noncash assets 540,000 Skipper, Capital 240,000
Professor, Capital 144,000
Total P600,000 Total Liabilities and Equities P600,000

After the partnership was liquidated and the cash was distributed, Skipper received P96,000 in cash
in full settlement of his interest. The liquidation loss must have been:

P504,000

P144,000

P360,000

P480,000

Question 12 1 / 1 point
The Abrams, Bartle and Creighton partnership began the process of liquidation with th following
balance sheet:

Cash P16,000 Liabilities P150,000


Noncash assets 434,000 Keaton, capital 80,000
Lewis, capital 90,000
Meador, capital 130,000
Total P450,000 Total P450,000

Abrams, Bartle and Creighton share profits and losses in a ratio of 3:2:5. Liquidation expense are
expected to be P12,000, After the liquidation expenses of P12,000 had been paid and the non -cash
assets sold, Creighton had a deficit of P8,000. For what amount were the non-cash assets sold?

P264,000
P158,000

P170,000

P146,000

Question 13 1 / 1 point
Gonda, Herron and Morse is considering possible liquidation because Morse is insolvent. The
partners have the following capital balances: P60,000, P70,000 and P40,000, respectively and share
profits and losses 30%, 45% and 25%, respectively. The partnership has P200,000 in assets that can
be sold for P150,000. What is the minimum that Morse’s creditors would receive if they have filed a
claim for P50,000?

P50,000

P-0-

P45,000

P27,500

Question 14 1 / 1 point
Shrek, Donkey and Fiona are partners in SDF and share profits and losses in the ratio of 5:3:2,
respectively. The partnership has cash of P10,000 and noncash assets of P90,000 when they decide
to liquidate. Liabilities at the time of liquidation are P40,000 including a note payable to Fiona of
P5,000. The partner capital accounts are Shrek P40,000, Donkey P15,000 and Fiona P5,000. The non-
cash assets of the partnership were sold for P26,000. The liabilities other than the note payable to
Fiona are paid. Fiona is personally insolvent. Shrek and Donkey are not insolvent. Under the
circumstances:

Shrek will receive a distribution in liquidation of P8,000.

Fiona will be required to contribute P2,800 to the partnership.

Donkey will be required to contribute P4,200 to the partnership.

Shrek will receive a distribution in liquidation of P6,250.

Question 15 1 / 1 point
Taylor, Ulman &Vicor Partnership, whose partners share net income or loss equally, is in liquidation.
Partner Taylor, whose capita account had a debit balance of P3,000, paid a P4,000 trade account
payable of the partnership. The appropriate journal entry (explanation omitted) for the partnership
is:

DR Trade Accounts Payable 4,000


CR Taylor, Capital 3,000
CR Ulman, Capital 500
CR Victor, Capital 500

DR Trade Accounts Payable 4,000


CR Loan Payable to Taylor 4,000

DR Trade Accounts Payable 4,000


CR Taylor, Capital 4,000

DR Cash 4,000
CR Taylor, Capital 4,000

Question 16 1 / 1 point
White, Sands and Luke has the following capital balances and profit and loss ratios: P50,000 (30%),
P100,000 (20%), and P200,000 (50%). If the partnership is to be liquidated and P150,000 becomes
available for the partners immediately, who gets the money?

P20,000 White; P57,143 Sands; P82,857 Luke

P0 White; P47,143 Sands; P102,857 Luke

P0 White; P57,143 Sands; P92,857 Luke

P10,000 White; P47,143 Sands; P92,857 Luke

Question 17 1 / 1 point
During the liquidation of Gym, Hob and Ing Partnership, Partner Hob withdrew equipment with a
cost to the partnership of P18,000, accumulated depreciation of P8,000, and a current fair value of
P13,000. The partners shared net income and losses equally. The net debit of Hob’s capital account
(including any gain or loss on disposal of the equipment), assuming the noncash asset may be
distributed safely to Hob, is:

P13,000

P12,000

P18,000

P10,000

Question 18 1 / 1 point
During the liquidation of the partnership of Karr, Rice and Long, Karr accepts, in partial settlement of
his interest, machine with a cost to the partnership of P150,000, accumulated depreciation of
P70,000, and a current fair market value of P110,000. The partners share net income and loss
equally. The net debit to Karr’s account (including any gain or loss on disposal of the machine) is

P110,000
P100,000

P90,000

P15,000

Question 19 1 / 1 point
X, Y and Z have capital balances of P90,000, P60,000 and P30,000, respectively. Profits are allocated
35% to X, 35% to Y, and 30% to Z. The partners have decided to dissolve and liquidate the
partnership. After paying all creditors, the amount available for distribution is P60,000. X, Y and Z are
all personally solvent. Under the circumstances, Z will

Personally have to contribute an additional P36,000.

Receive P30,000

Personally have to contribute an additional P6,000.

Receive P18,000

Question 20 1 / 1 point
In a simple partnership liquidation, the last remaining cash distribution should be made according to
the ratio of

The individual partner’s capital accounts, increased by partnership loans to the partners
and decreased by partner loans to the partnership.

The individual partner’s capital accounts, increased by partner loans to the partnership.

The individual partner’s profit and loss agreement.

The individual partner’s capital accounts, decreased by partnership loans to the


partners and increased by partner loans to the partnership.

Question 21 1 / 1 point
If a partner with a debit capital balance during liquidation is personally solvent, the

Partner’s debit balance will be allocated to the other partners.

Partner must invest additional assets in the partnership.

Partnership will loan the partner enough cash to absorb the debit balance.

Other partners will give the partner enough cash to absorb the debit balance.

Question 22 1 / 1 point
The first step in the liquidation process is to

Convert noncash assets into cash.

Allocate any gains or losses to the partners.

Compute any net income (loss) up to the date of dissolution.

Pay partnership creditors.

Question 23 1 / 1 point
In a partnership liquidation, how are partner salary allocations treated?

Salary allocations take precedence over amounts due to partners with respect to their
capital interests, but not profits.

Salary allocations are disregarded.

Salary allocations take precedence over amounts due to partners with respect to their
capital profits, bt not capital interests.

Salary allocations take precedence over creditor payments.

Question 24 1 / 1 point
If cash payments to partners of a limited partnership in liquidation are delayed until all noncash
assets have been realized, any cash remaining after all partnership creditors have been paid is
distributed:

In the ratio for sharing net income and losses

According to the liquidator’s best judgment

In some other manner

In the amounts equal to the partners’ loan and capital account balances

Question 25 0 / 1 point
In the liquidation of a limited partnership, a loan payable to a partner by the partnership is:

Considered to be a liability of the partnership

Paid immediately after all outside creditors have been paid in full

Paid immediately before all outside creditors have been paid in full

Liabilities still to be paid

Question 26 1 / 1 point
Which item is not shown on the schedule of partnership liquidation?

Property owned by the partnership

Personal assets of the partners

Current cash balances

Liabilities still to be paid

Question 27 1 / 1 point
The following is the priority sequence in which liquidation proceeds will be distributed for a
partnership:

Partnership liabilities, partnership capital balances, partnership loans.

Partnership liabilities, partnership loans, partnership drawings, partnership capital


balances.

Partnership liabilities, partnership loans, partnership capital balances.

Partnership drawings, partnership liabilities, partnership loans, partnership capital


balances.

Question 28 1 / 1 point
Which of the following statements is correct regarding a partners’ debit capital balances?

All of these statements are correct.

If contributions are not possible, the other partners with credit capital balances will be
allocated a portion of the debit balance based on their proportionate profit-and-loss-
sharing percentages.

The partner should make contributions to reduce the debit balance to whatever extent
possible.

Partners who absorb another’s debit capital balance have a legal claim against the
deficient partner.

Question 29 1 / 1 point
If a partnership has only non-cash assets, all liabilities have been properly disbursed, and no
additional liquidation expenses are expected, the maximum potential loss to the partnership in the
liquidation process is:

The fair market value of the non-cash assets

The book value of the non-cash assets


None of the above

The estimated proceeds from the sale of the assets less the book value of the non-cash
assets

Question 30 1 / 1 point
Which of the following procedures is acceptable when accounting for a deficit balance in a partner’s
capital account during partnership liquidation?

If a partner with a negative capital balance is personally insolvent, the negative capital
balance may be absorbed by those partners having a positive capital balance according
to the residual profit and loss sharing ratios that apply to all the partners.

If a partner with a negative capital balance is personally insolvent, the negative capital
balance may be absorbed by those partners having a positive capital balance according
to the residual profit and loss sharing ratios that apply to those partners having positive
balances.

All the above procedures are acceptable.

A partner with a negative capital balance must contribute personal assets to the
partnership that are sufficient to bring the capital account to zero.

Question 31 1 / 1 point
A partnership dissolution differs from liquidation in that

Payments are made to creditors before partners receive value.

Periodic payments to partners are made when cash becomes available.

A partner withdraws from the business and the enterprise continues to function.

Full payment is made to all outside creditors before remaining cash is distributed
partners in a final lump sum payment.

Question 32 1 / 1 point
A partnership in liquidation has converted all assets into cash and paid all liabilities, the order of
payment

Will have amounts due to partners with respect to their capital accounts take
precedence over amounts owed by partners other than for capital and profits

Will be by any manner that is both reasonable and rational for the partnership.

Will have amounts owed by partners other than for capital and profits take precedence
over amounts due to partners with respect to their capital accounts.

Will be according to the partners’ residual profit and loss sharing ratios.
Question 33 1 / 1 point
If conditions produce a debit balance in a partner’s capital account when liquidation losses are
allocated

The partner receives further allocations of liquidation losses, but not gains.

The partner has an obligation of personal net assets to the other partners.

The partner is no longer obligated to partnership creditors.

The partner receives no further allocation of liquidation losses and gains.

Question 34 1 / 1 point
Which of the following statements is correct?
1. Personal creditors have first claim on partnership assets.
2. Partnership creditors have first claim on partnership assets.
3. Partnership creditors have first claim on personal assets.

Both 2 and 3

Question 35 1 / 1 point
Offsetting a partner’s loan balance against his debit capital balance is referred to as the:

Liquidation of assets

Right of offset

Marshaling of assets

Allocation of assets
PARTNERSHIP INSTALLMENT LIQUIDATION

1 / 1 point
The balance sheet for the partnership of JJ, CC and TT, whose shares of profits and losses are 40, 50
and 10 percent, is as follows:

Cash P50,000 Accounts payable P150,000


Inventory 360,000 JJ, Capital 160,000
CC, Capital 45,000
TT, Capital 55,000
Total Assets P410,000 Total Liabilities and Equities P410,000

The partnership will be liquidated in instalments. As cash becomes available, it will be distributed to
the partners. If inventory costing P200,000 is sold for P140,000, how much cash should be
distributed to (1) JJ, (2) CC and (3) TT at this time?

JJ P20,000; CC P-0-: TT P20,000

JJ P32,000; CC P-0-; TT P8,000

JJ P56,000; CC P70,000; TT P14,000

JJ P16,000; CC P20,000; TT P4,000

Question 2 1 / 1 point
The partnership of Peter, Paul and Mary share profits and losses in the ratio of 4:4:2, respectively.
The partners voted to dissolve the partnership when its assets, liabilities and capital were as follows:

Assets: Liabilities and Capital:


Cash P250,000 Liabilities P200,000
Other assets 1,000,000 Peter, capital 300,000
Paul, capital 350,000
Mary, capital 400,000
Total assets P1,250,000 Total Liabilities and Equity P1,250,000
The partnership will be liquidated over a prolonged period of time. As cash is available, it will be
distributed to the partners. The first sale of noncash assets having a book value of P600,000 realized
P475,000. How much cash should be distributed to each partner after this sale?

Peter, P150,000; Paul, P175,000; Mary, P200,000

Peter, P210,000; Paul, P290,000; Mary, P145,000

Peter, P90,000; Paul, P140,000; Mary, P295,000


Peter, P290,000; Paul, P210,000; Mary, P105,000

Question 3 1 / 1 point
who share profits and losses in the ratio 4:4:2, respectively:

Cash P20,000 Liabilities P50,000


Other assets 180,000 Alpha, capital 37,000
Baker, capital 65,000
Charley, capital 48,000
Total Assets P200,000 Total Liabilities and Equity P200,000

Assume that the partners decided to liquidate the partnership. The first sale of noncash assets
having a book value of P90,000 realized P50,000, and all cash available after settlement with
creditors was distributed. How should the available cash have been distributed?

Alpha, P-0-; Baker, P13,333; Charley, P6,667

Alpha, P8,000; Baker, P8,000; Charley, P4,000

Alpha, P6,667; Baker, P6,667; Charley, P6,666

Alpha, P-0-; Baker, P18,500; Charley, P1,500

Question 4 1 / 1 point
A partnership has the following capital balances: A (20% profits and losses) = P100,000; B (30% of
profits and losses) = P120,000; C (50% of profits and losses) = P180,000. If the partnership is to be
liquidated and P30,000 becomes immediately available, who gets that money?

P22,000 to A; P8,000 to B; P-0- to C

P6,000 to A; P9,000 to B; P15,000 to C

P22,000 to A; P3,000 to B; P5,000 to C

P24,000 to A; P6,000 to B; P-0- to C

Question 5 0 / 1 point
On December 1, 20x5, the partners of Tim, Williams, and Levin, who share profits and losses in the
ration of 4:4:2, decided to liquidate their partnership. On this date the partnership condensed
balance sheet was as follows:

Cash P100,000 Liabilities P90,000


Other assets 300,000 Tim, capital (40%) 100,000
William, capital (40%) 120,000
Levin, capital (20%) 90,000
On December 11, 20x5, the first cash sale of other assets with a carrying amount of P200,000
realized P140,000. Safe installment payments to the partners were made on the same date. How
much should be distributed to (1) Tim, (2) William and (3) Levin?

(1) P40,000; (2) P40,000; (3) P20,000

(1) P24,000; (2) P24,000; (3) P12,000

(1) P36,000; (2) P56,000; (3) P58,000

(1) P40,000; (2) P48,000; (3) P18,000

Question 6 1 / 1 point
The following condensed balance sheet is presented for the partnership of Archer, Bows and Cross,
who share profits and losses in the ratio 6:3:1, respectively:

Cash P10,000 Liabilities P130,000


Other assets 290,000 Note Payable to Archer 15,000
Note Payable to Bows 5,000
Archer, Capital 43,000
Bows, Capital 43,000
Cross, Capital 64,000
Total assets P300,000 Total liabilities and equity P300,000

Bows paid P30,000 to creditors out of her own personal funds – this has not been reflected in the
above balance sheet. Archer is personally solvent but temporarily not liquid. The partners decided to
liquidate the partnership. The first sale of noncash assets having a book value of P140,000 realized
P120,000. How should the available cash be distributed?

Archer, P-0-; Bows, P-0-; Cross, P-0-

Archer, P-0-; Bows, P-0-; Cross, P30,000

Archer, P10,000; Bows, P10,000; Cross, P10,000

Archer, P21,000; Bows, P8,000; Cross, P1,000

Archer, P18,333; Bows, P8,333; Cross, P3,334

Question 7 1 / 1 point
On January 1, 20x4, the partners of CC, DD and EE, who share profits and losses in the ratio of 5:3:2,
decided to liquidate their partnership. On this date the partnership condensed balance sheet was as
follows: cash, P50,000; other assets, P250,000; liabilities, P60,000; CC, capital P80,000; DD, capital
P90,000; and EE, P70,000.
On January 15, 20x4, the first cash sale of other assets with a carrying amount of P150,000 realized
P120,000. Safe instalment payments to the partners were made on the same date. How much cash
should be distributed to (1) CC, (2) DD, and (3) EE?
(1) P40,000; (2) P45,000; (3) P35,000

(1) P60,000; (2) P36,000; (3) P24,000

(1) P55,000; (2) P33,000; (3) P22,000

(1) P15,000; (2) P51,000; (3) P44,000

Question 8 0 / 1 point
The capital balances, prior to the liquidation of the XYZ partnership, were as follows:

X, Capital P130,000
Y, Capital P130,000
Z, Capital P100,000

X, Y and Z share profits and losses in the ratio of 5:3:2. As a result of a loan, the partnership owes Y
P80,000. Using the information above, which partner has the highest loss absorption power (LAP)
prior to liquidation?

Both X and Y

Question 9 0 / 1 point
After all noncash asset have been converted into cash in the liquidation of the AA and KK
Partnership, the ledger contains the following account balances:

Debit Credit
Cash P47,000
Account payable P32,000
Loan payable to AA P15,000
AA, Capital P7,000
KK, Capital P7,000

Available cash should be distributed with P32,000 going to accounts payable and then:

P7,000 to AA and P8,000 to KK

P15,000 loan payable to AA

P8,000 to AA and P7,000 to KK


P7,500 each to AA and KK

Question 10 1 / 1 point
Dancey, Reese, Newman and John were partners who shared profits and losses on a 4:2:2:2 basis,
respectively. They were beginning to liquidate their business. At the start of the process, capital
balances were as follows:

Dancey, capital P72,000 Newman, capital P52,000


Reese, capital P32,000 John, capital P24,000

Which one of the following statements is true?

The first available P4,000 would go to John

The first available P8,000 would go to John

The first available P8,000 would go to Reese

The first available P16,000 would go to Dancey

The first available P16,000 would go to Newman

Question 11 0 / 1 point
Harding, Jones and Sandy is in the process of liquidating and the partners have the following capital
balances: P20,000, P22,000 and P(10,000) respectively. The partners share all profits and losses 50%,
35% and 15%, respectively. Sandy has indicated that the P(10,000) deficit will be covered with a
forthcoming contribution. The remaining partners have requested to receive P18,382 in cash that is
available. How should this cash be distributed?

Harding P10,813; Jones P7,569

Harding P8,000; Jones P18,382

Harding P6,107; Jones P12,275

Harding P7,500; Jones P18,882

on 12 0 / 1 point
A, B and C have capital balances of P80,000, P80,000, and P40,000, respectively. Profits are allocated
40% to A, 40% to B and 20% to C. The partners have decided to dissolve and liquidate the
partnership. After paying all creditors the amount available for distribution is P20,000. A and B are
personally solvent. C is personally insolvent. Under the circumstances, A and B will each

Receive P9,000.

Receive P8,000.
Receive P6,000.

Receive P10,000.

Question 13 1 / 1 point
The ABC partnership has the following capital accounts on its books at December 31, 20x4:

Credit: A, Capital P200,000


B, Capital 120,000
C, Capital 40,000

All liabilities have been liquidated and the cash balance is zero. None of the partners have personal
asset in excess of his personal liabilities. The partners share profits and losses in the ratio of 3:2:5. If
the noncash assets are sold for P150,000, the partners should receive as a final payment:

A, P152,000; B, P88,000; C, P40,000

A, P152,000; B, P88,000; C, P-0-

A, P60,000; B, P40,000; C, P100,000

A, P128,000; B, P72,000; C, P-0-

Question 14 1 / 1 point
Partners Dalton, Edwards, and Finley have capital balances of P40,000, P90,000 and P30,000,
respectively, immediately prior to liquidation. Total remaining assets have a book value of P160,000,
the liabilities having been paid. Among these remaining assets is a machine with a fair value of
P35,000. The partners split profits and losses equally. Edwards covets the machine and is willing to
accept it for P35,000 in lieu of cash. The other partners have no designs on specific assets, only cash
in liquidation. How much cash, in addition to the machine, would be first distributed to Edwards,
before any of the other partners received anything?

P300,000

P15,000

P166,667

P50,000

Question 15 1 / 1 point
A partnership is currently holding P400,000 in assets and P234,000 in liabilities. The partnership is to
be liquidated, and P20,000 is the best estimation of the expenses that will be incurred during this
process. The four partners share profits and losses as show. Capital balances of the start of
liquidation follow:

KK, capital (40%) P59,000


MM, capital (30%) 39,000
BB, capital (10%) 34,000
JJ, capital (20%) 34,000

The partners realize that BB will be the first partner to start receiving cash. How much cash will BB
receive before any of the partners collect any cash?

P14,750

P17,000

P19,500

P12,250

Question 16 1 / 1 point
CC, PP, MM and HH are partners who share profits and losses on a 4:3:2:1 basis, respectively. They
are beginning to liquidate the business. At the start of this process, capital balances are as follows:

CC, capital P60,000


PP, capital 27,000
MM, capital 43,000
HH, capital 20,000

Which of the following statements is true?

The first available P2,000 will go to HH.

CC will be the last partner to receive any available cash.

The first available P3,000 will go to MM.

CC will collect a portion of any available cash before HH receives money.

Question 17 1 / 1 point
A partnership has gone through liquidation and now reports the following account balances:

Cash P16,000
Loan from JJ 3,000
WW, capital (2,000) (deficit)
JJ, capital (5,000) (deficit)
FF, capital 13,000
RR, capital 7,000

Profits and losses are allocated on the following basis: WW, 30 percent; JJ, 20 percent; FF, 30
percent; RR, 20 percent. Which of the following events should occur now?
FF should receive P11,800 and RR P4,200.

JJ should receive P3,000 cash because of the loan balance.

JJ should receive P3,000, FF P8,800, and RR P4,200.

FF should receive P10,600 and RR P5,400.

Question 18 1 / 1 point
Allen, Branden & Caylin are in the process of liquidating their partnership. They have the following
capital balances and profit and loss percentages:

Capital Balance Profit/Loss %


Allen 5,000 debit 20%
Branden 18,000 credit 50%
Caylin 6,000 credit 30%

The partnership balance sheet shows cash of P5,000, non-cash assets of P14,000, and no liabilities.
Assuming no liquidation expenses, what safe payment could be made?

P1,000 to Allen, P2,500 to Branden, and P1,500 to Caylin

P18,000 to Branden only

P5,000 split between Branden & Caylin by a ratio of 5/8 and 3/8, respectively.

P5,000 to Branden only

Question 19 1 / 1 point
The following account balances were available for the Perry, Quincy and Renquist partnership just
before it entered liquidation:

Cash P90,000 Liabilities P170,000


Noncash assets 300,000 Perry, capital 70,000
Quincy, capital 50,000
Renquist, capital 100,000
Total P390,000 Total P390,000

Perry, Quincy and Renquist had shared profits and losses in a ratio of 2:4:4.Liquidation expenses
were expected to be P8,000. Assume that Quincy was insolvent and could not contribute assets to
cover any deficit in her capital account. For what amount must the non-cash assets have been sold,
so that Renquist would have received some cash from the liquidation?
Any amount in excess of P58,000

Any amount in excess of P50,000

Any amount in excess of P108,000

Any amount in excess of P201,600

Question 20 1 / 1 point
A local partnership was in the process of liquidating and reported the following capital balances:

Justice, capital (40% share of all profits and losses) P23,000


Zobart, capital (35%) 22,000
Douglass, capital (25%) (14,000)

Douglass indicated that the P14,000 deficit would be covered by a forthcoming contribution,
However, the two remaining partners asked to receive he P31,000 that was then available. How
much of this money should Justice receive?

P15,467

P15,553

P15,000

P17,333

Question 21 0 / 1 point
The balance sheet of the Partnership Duro, Kemp and Ruth on December 31, 20x4 before liquidation
shows the following:

Cash P120,000 Accounts payable P150,000


Other Assets 560,000 Notes payable 100,000
Loan to Ruth 20,000 Loan from Kemp 10,000
Duro, capital (50%) 170,000
Kemp, capital (30%) 170,000
Ruth, capital (20%) 100,000
Total P700,000 Total P700,000

The partnership decided to liquidate as soon as possible after December 31, 20x4, and all cash on
and except for P10,000 contingency balance is to be distributed at the end of each month until the
liquidation is completed. If in the first month of realization and distribution, the partnership pays
liquidation expenses of P5,000 and Kemp receive P60,000. Compute he cash proceeds from the initial
sale of other assets?

P160,000
P200,000

P180,000

P205,000

Question 22 1 / 1 point
The PQR partnership is being dissolved. All liabilities have been paid and the remaining assets being
realized gradually. The equity if the partners are as follows:

Partners’ Loans to (from) Profit and


Accounts Partnership Loss Ratio
P P24,000 P6,000 3
Q 36,000 - 3
R 60,000 (10,000) 4

The second cash payment to any Partner(s) under a program of priorities shall be made thus

To R P2,000

To Q P6,000 and R P8,000

To Q P6,000

To R P8,000

Question 23 1 / 1 point
A cash distribution plan (payment priority program) for the Matthew, Norell, and Reams partnership
appears below:

Priority Matthew Norell Reams


Creditors
First P300,000 100%
Next P80,000 70% 30%
Next P70,000 3/7 4/7
Remainder 22% 34% 44%

If P550,000 of cash is to be distributed, how much will be received by the (1) priority creditors, (2)
Matthew, (3) Norell and (4) Reams?

(1) P-0-; (2) P-0-: (3) P-0-; (4) P-0-

(1) P300,000; (2) P108,000: (3) P58,000; (4) P84,000


(1) P300,000; (2) P108,000: (3) P58,000; (4) P84,000

(1) P-0-; (2) P121,000: (3) P187,000; (4) P242,000

Question 24 1 / 1 point
Under the rule of offset, what is the proper disposition of a partnership loan that was made from a
partner who has a debit balance?

The loan is written off as a partnership loss if the partner does not have the ash to cover
the debit balance.

The loan is charged off to the capital accounts of all the partners in their profit and loss
sharing ratios.

The loan is charged off to the capital account of the debtor partner.

The loan is first paid to the debtor partner before cash payments are made to partners.

Question 25 1 / 1 point
In partnership liquidation, what are safe payments?

The amounts of distributions that can be made to the partners, after all creditors have be
full.

The amounts of distributions that can be made to the partners, after all non-cash assets h
adjusted to fair market value.

All the above are examples of the safe payments concept.

The amounts of distributions that can be made to the partners with assurance that such a
will not have to be returned to the partnership.

6 1 / 1 point
If all partners are included in the first installment of an instalment liquidation, then in future
installments

A safe payments schedule must be prepared before each cash distribution to avoid
excessive payments to partners.

A cash distribution plan must be prepared so that partners will know when they will be
included in cash distributions.

Cash will be distributed according to the residual profit and loss sharing ratio.

Cash should not be distributed until all non-cash assets are converted into cash.

Question 27 1 / 1 point
Partner’s maximum loss absorbable is calculated by

Multiplying the partner’s capital balance by his or her profit-and-loss-sharing


percentage.

Dividing the partner’s capital balance by his or her percentage interest in capital.

Dividing the partner’s capital balance by his or her profit-and-loss-sharing percentage.

Multiplying distributable assets by the partner’s profit-sharing percentage.

Question 28 1 / 1 point
A schedule prepared each time cash is to be distributed is called a(n)

Marshalling of assets schedule.

Advance cash distribution schedule.

Safe payment schedule.

Loss absorption potential schedule.

Question 29 0 / 1 point
Which partner is considered the most vulnerable as a result of a computation of vulnerability
rankings?

The partner with the lowest vulnerability ranking, who also has the highest loss
absorption potential.

The partner with the lowest vulnerability ranking, who also has the lowest loss
absorption potential.

The partner with the highest vulnerability ranking, who also has the highest loss
absorption potential.

The partner with the highest vulnerability ranking, who also has the lowest loss
absorption potential.

Question 30 0 / 1 point
The rank order is for claims against a bankrupt partner of
I. Those owing to partners by way of contribution
II. Those owing to separate creditors
III. Those owing to partnership creditors

III first; II second and I third.


I first; III second and II third.

II first; I second and III third.

II first; III second and I third.

Question 31 1 / 1 point
The partnership of Clapton, Seidel and Thomas was insolvent and will be unable to pay P30,000 in
liabilities currently due. What recourse was available to the partnership’s creditors?

They must try to obtain a payment from the partner with the largest capital account
balance

They must present their claims to the three partners in the order of the partners’
capital account balances

They must present equal claims to the three partners as individuals

They may seek remuneration from any partner they choose

They cannot seek remuneration from the partners as individuals

Question 32 1 / 1 point
The partnership of Nurr, Cleamons and Kelly was insolvent, as was Cleamons personally. The
partnership had begun liquidating its assets and Cleamons’ capital account had a debit balance. How
would the claim of Nurr and Kelly against Cleamons be ranked in comparison with the claims of
Cleamons’ other creditors?

It ranks higher in priority than Cleamons’ personal creditors but lower in priority than
the creditors of the partnership

It ranks higher in priority than Cleamons’ personal creditors and the creditors of the
partnership

Ranks equal in priority with the claims of Cleamons’ personal creditors

It ranks lower in priority than Cleamons’ personal creditors and the creditors of the
partnership

It ranks lowers in priority than Cleamons’ personal creditors but higher in priority than
the creditors of the partnership

Question 33 1 / 1 point
What accounting transactions are not recorded by an accountant during liquidation?

The payment of liabilities and expenses

The allocation of the resulting gains and losses


Remaining unpaid debts settled and the distribution of any remaining assets to the partne
on their profit and loss ratio

The conversion of partnership assets into cash

4 1 / 1 point
Which of the following statements is false concerning the Schedule of Liquidation?

The Schedule of Liquidation provides a listing of property still being held by the
partnership and liabilities remaining unpaid.

The Schedule of Liquidation keeps creditors and partners apprised of the results of the
process of dissolution

Frequent reporting by the accountant is rarely necessary.

The Schedule of Liquidation provides a listing of transaction to date, current cash and
capital balances.

Liquidations may take a considerable length of time to complete.

Question 35 1 / 1 point
The Schedule of Liquidation keeps creditors and partners apprised of the results of the process of
dissolution

The other partners must contribute personal assets to cover the deficit balance.

The partner with a deficit balance contributed personal assets only if those personal
assets exceed personal liabilities.

Partners never have a deficit balance.

The partnership must sell assets in order to cover the deficit balance

The partner with a deficit balance must contribute personal assets to cover the deficit
balance.

Question 36 1 / 1 point
Which of the following statements is true concerning the distribution of safe payments?

Safe payments are equal to the recorded capital balances of partners with positive
capital balances.

The distribution of safe payments may only be made after all liabilities have been paid.

The distribution of safe payments assumes that any capital deficit balances will prove to
be a total loss to the partnership.
In computing safe payments, partners with positive capital balances are assumed to
absorb an equal share of any deficit balance(s)

Question 37 1 / 1 point
A schedule prepared each time cash is to be distributed is called a(n)

Safe payment schedule.

Loss absorption potential schedule.

Advance cash distribution schedule.

Marshaling of assets schedule.

Question 38 1 / 1 point
An advance cash distribution plan is prepared

Each time cash is distributed to partners in an installment liquidation.

Each time a partnership asset is sold in an installment liquidation.

None of these.

To determine the order and amount of cash each partner will receive as it becomes
available for distribution.

Question 39 1 / 1 point
In a partnership liquidation, the final cash distribution to the partners should be made in accordance
with the:

Balances of the partners’ capital accounts.

Ratio of capital contributions less withdrawals by the partners.

Ratio of the capital contributions by the partners.

Partners’ profit and loss-sharing ratio.

Question 40 1 / 1 point
The first step in preparing an advance cash distribution plan is to

Allocate any gains (loss) to the partners in their profit-sharing ratio.

Determine the net capital interest if each partner.

Determine the order in which partners are to participate in cash distributions.

Compute the amount of cash each partner is to receive as it becomes available for
distribution.

Question 41 1 / 1 point
Offsetting a personal’s loan balance against his debit capital balance is referred to as the:

Right of offset.

Liquidation of assets.

Marshaling of assets.

Allocation of assets.

Question 42 1 / 1 point
If a partner with a debit capital balance during liquidation is personally solvent, the

Partnership will loan the partner enough cash to absorb the debit balance.

Partner must invest additional assets in the partnership.

Other partners will give the partner enough cash to absorb the debit balance.

Partner’s debit balance will be allocated to the other partners.

Question 43 1 / 1 point
In a partnership liquidation, the final cash distribution to the partners should be made in accordance
with the:

Partners’ profit and loss sharing ratio.

Ratio of capital contributions less withdrawals by the partners.

Balances of the partners’ capital accounts.

Ratio of the capital contributions by the partners.

Question 44 1 / 1 point
In an advance plan for installment distributions of cash to partners of a liquidating partnership, each
partner’s loss absorption potential is computed by

Dividing the total of each partners’ capital account less receivables from the partner
plus payables to the partner by the partner’s profit and loss percentage.

Some other method.

Dividing each partner’s capital account balance by the percentage of that partner’s
capital account balance to total partners’ capital.
Multiplying each partner’s capital account balance by the percentage of that partner’s
capital account balance to total partners’ capital.

Question 45 1 / 1 point
In accounting for partnership liquidation, cash payments to partners after all creditors’ claims have
been satisfied, but before the final cash distribution, should be according to:

The partners’ relative profit and loss-sharing ratios.

Safe payments computations.

The final balances in partner capital accounts.

The partners’ relative share of the gain or loss on liquidation.

Question 46 1 / 1 point
Which of the following is not correct with regard to creditor claims against partnerships and
individual partners?

Partner creditors can have claims against individual partner assets and partnership assets
extent of the partner’s capital account balance

Partnership creditors can have claims against partnership assets and individual partner as
to the extent that the partner has a deficit capital account balance

All of the above

Partnership creditors can have claims against partnership assets and individual partner as

7 1 / 1 point
Which of the following is not a possible claim against a partner’s personal assets?

Partnership creditors if claim is not fully paid from partnership assets

Personal creditors of other partners

Other partners, if the partner in question has a deficit capital account

Personal creditors of partner in question

Question 48 1 / 1 point
Which of the following statements is correct with regard to a partnership liquidation?

Loans from partners to the partnership have the same priority claim against
partnership assets as to a creditor claims from other entities.

All creditors must be paid in full before distributions can be made to partners
All creditors are equal with regard to priority of claim against partnership assets

Partner capital contributions and undistributed partnership income are viewed as


distinct in the Uniform Partnership Act

Question 49 0 / 1 point
Which of the following is not a part of the partnership liquidation process?

Recognition of market value adjustments of assets and liabilities

Liquidation of noncash assets

Closing of the accounting records

Allocation of any remaining profit or loss to partners’ capital accounts

Question 50 1 / 1 point
Which of the following describes a partnership lump-sum liquidation?

Keeping the partnership assets and liabilities separate from the partners’ personal
assets and liabilities

The combining of a partner’s capital account with loans to/from partnership

The sale of all noncash assets and payment of liabilities before a single distribution to
partners

A series of interim distributions to partners while the sale of noncash assets and the
payment of liabilities is occurring

Question 51 1 / 1 point
Which of the following describes a partnership installment liquidation?

The combining of a partner’s capital account with loans to/from partnership

The sale of all noncash assets and payment of liabilities before a single distribution to
partners

A series of interim distributions to partners while the sale of noncash assets and the
payment of liabilities is occurring

Keeping the partnership assets and liabilities separate from the partners’ personal
assets and liabilities

Question 52 0 / 1 point
Which of the following is not correct with regard to a partnership Statement of Realization and
Liquidation?
The statement details all business transactions during the partnership liquidation

Gains and losses are allocated to capital accounts

Balance sheet and income statement accounts appear on the statement

Residual profit and loss ratios are typically used to make allocations to partners’ capital
accounts

REVENUE RECOGNITION: REVENUE FROM


CONTRACTS WITH CUSTOMERS
1 / 1 point
Items 1 to 10 are based on the following information:
SmaGlo, Inc., a telecommunications operator, entered into a contract with Kim Dorothy on March 1,
20x7. In line with the contract, Kim Dorothy subscribes for SmaGlo’s monthly plan for 12 months and
in return Kim Dorothy received a free Apple I-Phone handset from SmaGlo. Kim Dorothy will pay a
monthly fee of P1,200. Kim Dorothy gets the handset immediately after contract signature.
SmaGlo sells the same handsets for P2,400 and the same monthly plans for P800 per month without
handset.
Identify the contract with a customer – what kind of contract between SmaGlo, Inc. and Kim
Dorothy?

Oral contract

Written contract

Customary business practice

No contract

Question 2 1 / 1 point
Identify the performance obligations (PO) – how many performance obligations?

1-Performance Obligation: Network services (monthly/installment plan)

1-Performance Obligation: Apple I-Phone Handset


2-Performance Obligations: Network services (monthly/installment plan) and Apple I-
Phone Handset

No performance obligation since there is no existing contract

Question 3 1 / 1 point
Determine the transaction price – the total transaction price?

P2,400

P9,600

P12,000

P14,400

Question 4 1 / 1 point
Allocate the transaction price to the performance obligations: the allocated transaction price to
each performance obligations?

None, since there is no contract.

P9,600 for network service and P2,400 for Apple I-Phone Handset

P0 for network service and P12,000 for Apple I-Phone Handset

P11,520 network service and P2,880 for Apple I-Phone Handset

Question 5 1 / 1 point
Recognize revenue when (or as) an entity satisfies a performance obligation: timing of revenue
recognition?

None, since there is no contract

Network service and for Apple I-Phone Handset – both overtime

Network service – over time and for Apple I-Phone Handset – point in time

Network service – point in time and for Apple I-Phone Handset – over time

Question 6 1 / 1 point
On March 1, 20x7, the amount of accounts receivable to be recorded:

None

P2,800

P9,600
P12,400

Question 7 1 / 1 point
On March 1, 20x7, the revenue from sales of goods amounted to:

None

P2,800

P9,600

P12,400

Question 8 1 / 1 point
On March 31, 20x7, the amount of accounts receivable to be recorded

None

P960

P1,200

P9,600

Question 9 1 / 1 point
On March 31, 20x7, the revenue from network services amounted to:

None

P960

P1,200

P12,400

Question 10 1 / 1 point
On December 31, 20x7, the total revenue amounted to:

None

P960

P1,200

P12,400

Question 11 1 / 1 point
On 25 June 20x9 Cambridge Co. received an order from a new customer, Circus Co. for products with
a sales value of P900,000. Circus Co. enclosed a deposit with the order of P90,000. On 30 June
Cambridge Co. had not completed credit checks on Circus Co and had not dispatched any goods.
Cambridge Co is considering the following possible entries for this transaction in its financial
statements for the year ended 30 June 20x9.
(i) Create a trade receivable for P810,000.
(ii) Include P90,000 in revenue for the year.
(iii) Recognize P90,000 as a contract liability.
(iv) Include P900,000 in revenue for the year.
(v) Do not include anything in revenue for the year.
According to PFRS 15 Revenue from Contracts with Customers, how should Cambridge Co. record
this transaction in its financial statements for the year ended 30 June 20x9?

(i) and (iv) only

(ii) and (iv) only

(ii) and (v) only

(iii) and (iv) only

Question 12 1 / 1 point
On June 1, 20x5, Johnson & Sons sold equipment to James Landscaping Services. In exchange fro a
zero-interest bearing note with a face value of P55,000, with payment de in 12 months. The fair
value of the equipment on the date of sale was P50,000. The amount of revenue to be recognized on
this transaction in 20x5 is:

P5,000

P55,000

P50,000

P50,000 sales and P2,917 interest revenue

Question 13 1 / 1 point
On 31 March 20x9 Dune entered into a contract to sell some machinery for P700,000. The contract
required Dune to buy back the goods in two-year’s time for P500,000. How should the contract be
accounted for in Dune’s financial statements?

Recognize P700,000 revenue and account for costs of repurchase in two years’ time

Account for the financial transaction as a lease in accordance with PFRS 16 Leases

Recognize a financial liability for a purchase price and recognize P700,000 revenue

Recognize a financial liability for P500,000 and recognize revenue for P200,000 for the
difference between the selling price and the repurchase price
Question 14 1 / 1 point
OC signed a contract to provide office services to PQ for one year from 1 October 20x6 for P500 per
month. The contract required PQ to make a single payment to OC for all 12 months at the beginning
of the contract. OC received P6,000 on October 1, 20x6. What mount of revenue should OC recognize
in its statement of profit or loss for the year ended March 31, 20x7?

Nil

P300

P3,000 profit

P6,000 profit

Question 15 1 / 1 point
LP received an order to supplier a customer with 10,000 units of product A every month for two
years. The customer had negotiated a low price of P200 per 1,000 units and agreed to pay P12,000 in
advance every six months. The customer made the first payment on July 1, 20x6 and LP supplied the
goods each month from that date. LP’s year end is September 30. In addition to the effect of cash
received, what is the effect of this order on LP’s financial statement for the year ended September
30, 20x6, in accordance with PFRS 15 Revenue from Contracts with Customers? (1) Revenue; (2)
Statement of financial position

(1) P6,000; (2) P36,000 trade receivable

(1) P6,000; (2) P6,000 current liability

(1) P12,000; (2) P36,000 trade receivable

(1) P12,000; (2) No effect

Question 16 1 / 1 point
OC signed a contract to provide office cleaning supplied for a period of one year from October 1,
20x8 for a fee of P500 per month. The contract required the client to make one payment to OC
covering all 12 months’ service in advance. The contract cost was estimated at P300 per month for
wages, materials and administration costs. OC received P6,000 on October 1, 20x8. What profit or
loss should OC recognize in its statement of profit or loss for the year ended March 31, 20x9?

P600 loss

P1,200 profit

P2,400 profit

P4,200 profit

Question 17 1 / 1 point
LP received an order to supply 10,000 units of product A every month for two years. The customer
had negotiated a low price of P200 per 1,000 units and agreed to pay P12,000 in advance every 6
months. The customer made the first payment on July 1, 20x2 and LP supplied the goods each month
from July 1, 20x2. LP’s year end is September 30. In addition to recording the cash received, what
entries should LP record, in its financial statements for the year ended September 30, 20x2, in with
PFRS 15 Revenue from Contract with Customers?

Include P6,000 in revenue for the year and create a trade receivable for P36,000

Include P6,000 in revenue for the year and create a current liability for P6,000

Include P12,000 in revenue for the year and create a trade receivable for P36,000

Include P12,000 in revenue for the year but do not create a trade receivable or current
liability

Question 18 1 / 1 point
On 31 March DT received an order from a customer, XX, for products with a sales value of P900,000.
XX enclosed a deposit with the order of P90,000. On March 31, DT had not obtained credit
references of XX and has not determined if it will meet this order. According to PFRS 15 Revenue
from Contract with Customers, how should DT record this transaction in its financial statements for
the year ended March 31?
(1) Include P90,000 as revenue for the year
(2) Include P90,000 as revenue for the year
(3) Do not include anything as revenue for the year
(4) Create a trade receivable for P810,000
(5) Create a trade payable for P90,000

1 and 4

2 and 5

3 and 4

3 and 5

Question 19 0 / 1 point
Coldwear Co entered into a 12-month contract to sell coats to a fashion retailer on October 1, 20x5.
The contract specified a price of P200 per coat, which would be reduced to P175 per coat if a
minimum of 400 were purchased in the year. During first six months of the contract, the retailer
purchased 150 coats and Coldwear Co determined that this level of demand would continue into the
second half of the year. In March 31, 20x6, due to a famous singer wearing one of the coats, demand
increased considerably, and the retailer purchased a further 500 coats in total between April 1, 20x6
and September 3, 20x6. What revenue from the contract should Coldwear recognize in its financial
statements for the years ended March 31, 20x6 and 20x7? (1) Y/e March 31, 20x6; (2) Y/e March 31,
20x7
(1) P26,250; (2) P87,500

(1) P30,000; (2) P83,750

(1) P30,000; (2) P87,500

(1) P30,000; (2) P100,000

Question 20 1 / 1 point
On March 20x7, DT received an order from a new customer, XX, for goods with sales value of
P900,000. XX enclosed a deposit with the order of P90,000. On March 31, 20x7, DT had dispatched
any goods. DT is considering the following possible entries for the transaction in its financial
statements for the year ended 31 March 20x7:
(1) Include P900,000 in revenue
(2) Include P90,000 in revenue
(3) Do not include any amount in revenue
(4) Recognize a trade receivable for P810,000
(5) Recognize a trade payable for P90,000
How should DT account for this transaction in its financial statements for the year ended March 31,
20x7 in accordance with PFRS?

1 and 4

2 and 5

3 and 4

3 and 5

Question 21 1 / 1 point
Build Company contracts with Palijo Inc. to build 100,000 mobile phones. Palijo agrees to pay Build
Company for P5,000,000 and contributed several parts with a fair value of P1,000,000 to assist in the
manufacture of the phones to Palijo’s specifications. If Build concludes that it has control over the
contributed parts, the transaction price would be:

Nil

P1,000,000

P5,000,000

P6,000,000

Question 22 1 / 1 point
On January 1, 20x9, Blesilda Prudencio Company enters into a contract to transfer Product X and
Product Y to Virginia and Nanette Co. for P200,000. The contract specifies that payment of Product X
will not occur until Product Y is also delivered. In other words, payment will not occur until both
Product X and Product Y are transferred to Virginia and Nanette. Blesilda Prudencio determines that
standalone prices are P60,000 for Product X and P140,000 for Product Y. Blesilda Prudencio delivers
Product X to Virginia and Nanette on February 1, 20x8. On March 1, 20x9, Blesilda Prudencio delivers
Product Y to Virginia and Nanette.
On January 1, 20x9, the amount of accounts receivable to be recorded:

None

P60,000

P140,000

P200,000

Question 23 1 / 1 point
On February 1, 20x9, the amount of accounts receivable to be recorded:

None

P60,000

P140,000

P200,000

Question 24 1 / 1 point
On February 1, 20x9, the amount of revenue to be recorded:

None

P60,000

P140,000

P200,000

Question 25 1 / 1 point
On March 1, 20x9, the amount of accounts receivable to be recorded:

None

P60,000

P140,000
P200,000

6 1 / 1 point
On March 1, 20x9, the amount of revenue to be recorded:

None

P60,000

P140,000

P200,000

Question 27 1 / 1 point
Items 27 and 31 are based on the following information:
On March 1, 20x9, Marissa Molina Company enters into a contract to transfer a product to Lilibeth
Hosena Inc. on July 31, 20x9. It is agreed that Lilibeth Hosena will pay the full price of P20,000 in
advance on April 1, 20x9. The contract is non-cancellable. Lilibeth Hosena, however, does not pay
until April 15, 20x9, and Marissa Molino delivers the product on July 1, 20x9. The cost of the product
is P15,000.
On March 1, 20x9, the amount of accounts receivable to be recorded:

None

P1,000

P15,000

P20,000

Question 28 1 / 1 point
On March 1, 20x9, the amount of revenue to be recorded:

None

P1,000

P15,000

P20,000

Question 29 1 / 1 point
On April 1, 20x9, the amount of revenue to be recorded:

None

P1,000
P15,000

P20,000

Question 30 1 / 1 point
On April 15, 20x9, the amount of revenue to be recorded:

None

P1,000

P15,000

P20,000

Question 31 1 / 1 point
On July 31, 20x9, the amount of sales revenue to be recorded:

None

P1,000

P15,000

P20,000

Question 32 1 / 1 point
Wood Designs Co enters into a contract on July 1, 20x5 to make and deliver furniture to a hotel chain
in two years’ time, when a new hotel is due for completion. Wood Designs Co offers its customer the
opportunity to pay P6,000,000 on delivery or P5,144,000 at the inception of the contract on July 1,
20x5. The customer pays the lower amount immediately. The interest rate implicit in the contract is
8 per cent and Wood Design Co’s incremental borrowing rate is 7 per cent. What contract liability is
recognized in Wood Design Co’s financial statements in the year ended June 30, 20x6?

P5,144,000

P5,504,000

P5,555,520

P6,000,000

Question 33 1 / 1 point
Fonesell Co enters into a contract on September 1, 20x5 to conduct telephone marketing activities
on behalf of a customer. The contract has a price of P8,000 and required Fonesell Co to contact
10,000 households over a period of six months in order to enquire about buying habits and promote
its customer. The customer is invoiced equal amounts three months and six months after the
commencement of the contract. By Fonesell’s Co’s year-end of December 31, 20x5, it has contacted
3,500 of the 10,000 customers. What amounts does Fonesell Co recognize in its financial statements
in the year ended December 31, 20x5?

Revenue of P4,000 and a receivable of P4,000

Revenue of P4,000 and a contract liability of P4,000

Revenue of P2,800, a receivable of P4,000 and a contract asset of P1,200

Revenue of P2,800, a receivable of P4,000 and a contract liability of P1,200

Question 34 1 / 1 point
Items 34 to 39 are based on the following information:
On March 1, 20x7, Giordano Company enters into a contract to transfer a product to Hotter on July
31, 20x7. The contract is structured such that Warmer is required to pay the full contract price of
P57,000 on August 31, 20x7. The cost of the goods transferred is P34,200. Giordano delivers the
product to Hotter on July 31, 20x7.
The contract exists on:

March 1, 20x7

July 31, 20x7

August 31, 20x7

Incomplete data

Question 35 1 / 1 point
On March 1, 20x7, the amount of accounts receivable to be recorded:

None

P34,200

P57,000

Incomplete data

Question 36 1 / 1 point
On July 31, 20x7, the amount of accounts receivable to be recorded:

None

P34,200

P57,000

Incomplete data
Question 37 1 / 1 point
On July 31, 20x7, the amount of revenue to be recorded:

None

P34,200

P57,000

Incomplete data

Question 38 1 / 1 point
The entry on July 31, 20x7 to record the contract:

No contract exists

Credit Unearned Sales revenue

Credit Sales revenue

Debit Unearned Sales revenue

Question 39 1 / 1 point
On August 31, 20x7, the amount of revenue to be recorded

None

P34,200

P57,000

Incomplete data

Question 40 1 / 1 point
Items 40 and 41 are based on the following information:
Rema Pulido Outsourcing enters into a contract to operate ReSa Review School’s information
technology data center for 3 years. Rema Pulido Outsourcing incurs selling commission costs of
P40,000 to obtain the contract. Before performing the services. Rema Pulido Outsourcing designs
and builds a technology platform that interfaces with ReSa Review’s systems. That platform is not
transferred to ReSa. ReSa promises to pay a fixed fee of P80,000 per month.
Rema Pulido Outsourcing incurs the following costs:
• Design services for the platform P60,000;
• Hardware for the platform P200,000,
• Software P120,000, and
• Migration and testing of data center P130,000

The contract costs that should be capitalized:


P60,000

P100,000

P220,000

P350,000

Question 41 1 / 1 point
The contract costs should be considered revenue expenditure:

P100,000

P130,000

P220,000

P350,000

Question 42 1 / 1 point
Meyer & Smith is a full-service technology company. They provide equipment, and installation
services as well as training. Customers can purchase any product or service separately or as a
bundled package. Container Corporation purchased computer, equipment, installation and training
for a total cost of P120,000 on March 15, 20x5. Estimated standalone fair values of the equipme nt,
installation, and training are P75,000, P50,000, and P25,000 respectively. The transaction price
allocated to equipment, installation and training:

P75,000, P50,000, P25,000 respectively

P40,000, P40,000, P40,000 respectively

P120,000 for the entire bundle

P60,000, P40,000, P20,000 respectively

Question 43 1 / 1 point
Meyer & Smith is a full-service technology company. They provide equipment, and installation
services as well as training. Customers can purchase any product or service separately or as a
bundled package. Container Corporation purchased computer, equipment, installation and training
for a total cost of P120,000 on March 15, 20x5. Estimated standalone fair values of the equipment,
installation, and training are P75,000, P50,000, and P25,000 respectively. The journal entry to record
the transaction on March 15, 20x4 will include a:

Credit to Sales Revenue for P120,000

Debit to Unearned Service Revenue of P25,000

P120,000 for the entire bundle


Credit to Service Revenue of P50,000

Question 44 1 / 1 point
Bella Pool Company sells prefabricated pools that cost P100,000 to customers for P180,000. The
sales price includes an installation fee, which is valued at P25,000. The fair value of the pool is
P160,000. The installation is considered a separate performance obligation and is expected to take 3
months to complete. The transaction price is allocated to the pool and the installation is:

P155,676 and P24,324 respectively

P160,000 and P25,000 respectively

P180,000 and P25,000 respectively

P138,378 and P21,622 respectively

Question 45 1 / 1 point
On January 31, O’Malley Company contracted to have two products built by Taylor Manufacturing for
a total of P185,000. The contract specifies that payment will only occur after both products have
been transferred to O’Malley Company. O’Malley determines that the standalone prices are
P100,000 for Product 1 and P85,000 for Product 2. On August 1, when Product1 has been
transferred, the journal entry to record this event include a:

Debit to Accounts Receivable for P100,000

Debit to Accounts Receivable for P85,000

Debit to Contract Assets for P85,000

Debit to Contract Assets for P100,000

Question 46 1 / 1 point
Sonya, Inc. (SONI) sells a big screen TV package consisting of a 60-inch plasma TV, a universal remote,
and onsite installation by SONI staff. The installation includes programming the remote to have the
TV interface with other parts of the customer’s home entertainment system. SONI concludes that the
TV, remote, and installation service are separate performance obligations. SONI sells the 60 -inch TV
separately for P17,000, sells the remote separately for P1,000 and offers the installation service
separately for P2,000. The entire package sells for P19,000. How much revenue would be allocated
to the TV, the remote, and the installation service, respectively?

P16,150; P950; P1,900

P17,100; P0, P1,900

P18,050; P950; P0

P19,000; P0; P0

Question 47 1 / 1 point
Items 47 to 49 are based on the following information:
Sonya, Inc. (SONI) sells a big screen TV package consisting of a 60-inch plasma TV, a universal remote,
and onsite installation by SONI staff. The installation includes programming the remote to have the
TV interface with other parts of the customer’s home entertainment system. SONI concludes that the
TV, remote, and installation service are separate performance obligations. SONI sells the 60 -inch TV
separately for P17,500, sells the remote separately for P1,000 and offers the entire package P19,000.
SONI does not sell the installation service separately. SONI is aware that other similar vendors charge
P1,500 for the installation service. SONI also estimated that it incurs approximately P1,000 of
compensation and other costs for SONI staff to provide the installation service. SONI typically
charges 40% above cost on similar sales.
Estimate the stand-alone selling price of the installation service using the adjusted market
assessment approach?

P0

P500

P1,400

P1,500

Question 48 1 / 1 point
Estimate the stand-alone selling price of the installation service using the estimated cost plus a
margin approach?

P0

P500

P1,400

P1,500

Question 49 1 / 1 point
Estimate the stand-alone selling price of the installation service using the residual approach?

P0

P500

P1,400

P1,500

Question 50 1 / 1 point
Mercedes Consultants provided Benz Construction with assistance in implementing various cost -
savings initiatives. Mercedes’ contract specifies that it will receive a flat fee of P50,000 and an
additional P20,000 if Benz reaches a pre specified target amount of cost savings. Mercedes estimates
that there is a 20% chance that Benz will achieve the cost-savings target.
Assuming Mercedes uses the expected value as its estimate of variable consideration, calculate the
transaction price.

P14,000

P40,000

P50,000

P54,000

1 1 / 1 point
Assuming Mercedes uses the most likely value as its estimate of variable consideration, calculate the
transaction price.

P14,000

P40,000

P50,000

P54,000

Question 52 1 / 1 point
Assuming Mercedes uses the expected value as its estimate of variable consideration, but is very
uncertain of that estimate due to a lack of experience with similar consulting arrangements.
Calculate the transaction price.

P14,000

P40,000

P50,000

P54,000

Question 53 1 / 1 point
Telephone Sellers Inc. sells prepaid telephone cards to customers. Telephone Sellers then pays the
telecommunication company, TeleExpress, for the actual use of its telephone lines related to the
prepaid telephone cards. Assume that Telephone Sellers sells P8,000 of prepaid cards in January
20x5. It then pays TeleExpress based on usage, which turns out to be 50% in February, 30% in March,
and 20% in April. The total payment by Telephone Sellers for TeleExpress lines over the 3 months is
P6,000. Indicate how much income Telephone Sellers should recognize in January, February, March,
and April, respectively:

P0; P1,000; P600; P400


P250; P250; P250; P250

P0; P2,000; P2,000; P2,000

P250; P1,000; P600; P150

Question 54 1 / 1 point
On February 12, 20x6, Mohawk Home and Garden enters into a contract with a local business to
provide weekly grass-cutting services between May and September of that year, and received P2,000
in advance. As part of a local business promotion, Mohawk offers a 50% discount on any barbecue
grill with a list price in excess of P200. In the past, Mohawk charged the same amount (P2,000) for
the same weekly grass-cutting service, but without the grill discount coupon. Based on historical
experience with other clients, Mohawk estimates that about 40% of the coupons will be redeemed,
purchasing grills with an average total list price of P400.
How many performance obligations are in the contract?

Question 55 1 / 1 point
How much of the contract price would be allocated to each performance obligation (PO)?

Coupon, P80; Grass-cutting service, P2,000

Coupon, P77; Grass-cutting service, P1,923

Coupon, P0; Grass-cutting service, P2,000

No allocation since there is only 1 PO

Question 56 1 / 1 point
Items 56 to 59 are based on the following information:
On May 1, 20x6, Chrome Computer Inc., enters into a contract to sell 5,000 units of keyboard to one
of its clients, Website Inc., at a fixed price of P95,000, to be settled by a cash payment on May 1.
Delivery is scheduled for June 1, 20x6. As part of the contract, the seller offers a 25% discount
coupon to Website for any purchases in the next six months. The seller will continue to offe r a 5%
discount on all sales during the same time period, which will be available to all customers. Based on
experience, Chrome Computer estimates a 50% probability that Website will redeem the 25%
discount voucher, and that the coupon will be applied to P20,000 of purchases. The stand-alone
selling price for the Comfort Office Keyboard is P19.60 per unit.
How many performance obligations are in this contract?
0

Question 57 1 / 1 point
The journal entry that Chrome would record on May 1 20x6?

DR Cash 95,000
CR Deferred revenue – keyboards 93,100
CR Deferred revenue – discount option 1,900

DR Cash 95,000
CR Revenue – keyboards 93,100
CR Revenue – discount option 1,900

DR Cash 95,000
CR Deferred revenue – keyboards 95,000

DR Cash 95,000
CR Revenue – keyboards 93,100
CR Deferred revenue – discount option 1,900

Question 58 1 / 1 point
Assume the facts and circumstances as above, except that Chrome gives a 5% discount option to
Website instead of 25%. In this case, what journal entry would Chrome record on May 1, 20x6?

DR Cash 95,000
CR Deferred revenue – keyboards 93,100
CR Deferred revenue – discount option 1,900

DR Cash 95,000
CR Revenue – keyboards 93,100
CR Revenue – discount option 1,900

DR Cash 95,000
CR Deferred revenue – keyboards 95,000
DR Cash 95,000
CR Revenue – keyboards 93,100
CR Deferred revenue – discount option 1,900

Question 59 1 / 1 point
Items 59 to 61 are based on the following information – Customer options
A Manila daily newspaper called “Manila Today” charges an annual subscription fee of P1,350.
Customers prepay their subscriptions and receive 260 issues over the year. To attract more
subscribers, the company offered new subscribers the ability to pay P1,300 for an annua l
subscription that also would include a coupon to receive a 40% discount on a one-hour ride through
Rizal Park in a carabao-drawn carriage. The list price of a carriage ride is P1,250 per hour. The
company estimates that approximately 30% of the coupons will be redeemed.
How much revenue should Manila Today recognize upon receipt of the P1,300 subscription price?

P0

P10,800

P12,500

P13,000

Question 60 1 / 1 point
How many performance obligations exist in this contract?

Question 61 0 / 1 point
The journal entry to recognize sale of 10 new subscriptions, clearly identifying the revenue of
deferred revenue associated with each performance obligation.

DR Cash 13,000
CR Deferred revenue – subscription 11,700
CR Deferred revenue – coupon 1,300

DR Cash 13,000
CR Deferred revenue – subscription 13,000
DR Cash 13,000
CR Deferred revenue – coupon 13,000

DR Cash 13,000
CR Revenue – subscription 11,700
CR Revenue – coupon 1,300

Question 62 1 / 1 point
Items 62 to 64 are based on the following information – Customer option and Prepayments
Antonio’s Car Services provides maintenance services for motorized vehicles. In March 20x6, Rick
placed an order for a new set of tires for P350. When a customer purchases goods and services in
excess of P300. Antonio’s gives the customer a 25% discount coupon for future purchases made in
the next three months. Antonio’s estimate that approximately 80% of customers utilize the coupon
and that on average those customers will purchase goods and services that typically sell for P75.
How many performance obligations are in Rick’s contract?

Question 63 1 / 1 point
How much revenue b recognized when the customer purchase goods and services?

Nil

P15

P335

P350

Question 64 1 / 1 point
How much deferred revenue can be recognized when the customer purchase goods and services?

Nil

P15

P335

P350
Question 65 1 / 1 point
Items 65 to 67 are based on the following information – Customer option and Prepayments
On February 12, 20x6, Mohawk Home and Garden enters into contract with a local business to
provide weekly grass-cutting services between May and September of that year and receive P2,000
in advance As part of a local business promotion, Mohawk offers a 50% discount on any barbecue
grill with a list price in excess of $200. In the past, Mohawk charges the same amount (P2,000) for
the same weekly grass-cutting service, but without the grill discount coupon. Based on historical
experience with other clients, Mohawk estimated that about 40% of the coupons will be redeemed,
purchasing grills with an average total list price of P400.
How many performance obligations are in the contract?

Question 66 1 / 1 point
How much revenue be recognized upon the receipt of the P2,000?

Nil

P77

P1,923

P2,000

Question 67 1 / 1 point
How much deferred revenue be recognized upon the receipt of the P2,000?

Nil

P77

P1,923

P2,000

Question 68 1 / 1 point
Items 68 to 70 are based on the following information – Customer options and Residual Approach
Imelda Queral, Josephine Umengan and Associates, Inc. sells computer workstations designed for
architects. In 20x6, it sold 120 workstations for P360,000. For each workstation sold, Imelda Queral,
Josephine Umengan and Associates distributed a 40% discount coupon for any additional future
purchases made in the next 12 months. Based on historical experience, Imelda QUERAL, Josephine
Umengan and Associates expects that approximately 30% of the coupons will be utilized, and the
goods purchased with the coupons would normally sell for P350.
How many performance obligations are in the contract?

Question 69 1 / 1 point
How much revenue be recognized when the customer purchase goods and services?

Nil

P5,040

P354,960

P360,000

Question 70 1 / 1 point
How much deferred revenue be recognized when the customer purchase goods and services?

Nil

P5,040

P354,960

P360,000

Question 71 1 / 1 point
Items 71 and 72 are based on the following information:
Blesilda, Virgie and Nanette Company enter into 50 contracts with customers. Each contract includes
the sale of one product for P120,000. The cost to Blesilda, Virgie ad Nanette of each product is
P48,000. Cash is received upfront and control of the product transfers on delivery. Customers can
return the product within 30 days to receive a full refund. Blesilda, Virgie and Nanette Company can
sell the returned products at a profit. Blesilda, Virgie and Nanette has significant experience in
estimating returns for the product. It estimates that 48 products will not be returned.
The amount of revenue:

Zero

P1,000,000
P5,760,000

P6,000,000

Question 72 1 / 1 point
The amount of refund liability amounted to:

Zero

P96,000

P120,000

P240,000

Question 73 1 / 1 point
Christine Abanes-Jereseno Auto Parts sells parts to Angelita Taguba-Gacosta Car during 20x5.
Abanes-Jereseno offers rebates of 2% on purchases up to P30,000 and 3% on purchases above
P30,000 if the customer’s purchases for the year exceed P100,000. In the past, Taguba-Gacosta
normally purchases P150,000 in parts during a calendar year. On March 25, 20x5, Taguba -Cagosta
Car purchased P37,000 of parts. The journal entry to record the sale

Debit to Accounts Receivable for P37,000

Debit to Accounts Receivable for P36,260

Credit to Sales Revenue for P35,890

Credit to Sales Revenue for P36,260

Question 74 1 / 1 point
Items 74 and 75 are based on the following information:
Josephine Yumul-Sayed Company sells goods to Genalyn Valenzuela-Siriban Inc. on account on
January 1, 20x5. The goods have a sales price of P1,220,000 (cost P1,000,000). The terms of the sale
are net 30. If Genalyn Valenzuela-Siriban pays within 5 days, it receives a cash discount of P20,000.
Past history indicates the cash discount will be taken.
The sales revenue on January 1, 20x5 amounted to:

Zero

P20,000

P1,200,000

P1,220,000

Question 75 0 / 1 point
The sales revenue on January 31, 20x5 assuming Durrant does not make payment until January 31,
20x5 amounted to:

Zero

P20,000

P1,200,000

P1,220,000

on 76 0 / 1 point
On January 3, Mandigma and Associates, an Accounting Services Company enters into a contact with
Dante Blanco International to perform advisory services regarding financial and investment in a
group of companies concerns for 12 months. Mandigma receives a quarterly management fee based
on a percentage of Dante Blanco investment in a group of companies at the end of each quarter. In
addition, Mandigma received a performance-based incentive fee of 20% of the investment’s return in
excess of the return of an observable value at the end of the year. How many performance
obligations are in the contract?

Question 77 1 / 1 point
Geri Corporation sells cosmetics through a network of independent distributors. Geri shipped
cosmetics to its distributors and is considering whether it should record P300,000 of revenue upon
shipment of a new line of cosmetics. Geri expects the distributors to be able to sell the cosmetics,
but is uncertain because it has little experience with selling cosmetics of this type. Geri is committed
to accepting the cosmetics back from the distributors if the cosmetics are not sold. How much
revenue should Geri recognize upon delivery to its distributors?

None

P25,000

P300,000

Incomplete data

Question 78 0 / 1 point
Items 78-83 are based on the following information:
NN Company sells 240 units for P100 each to JJ Inc. for cash. NN allows JJ to return any unused
product within 30 days and receive a full refund. The cost of each product is P60. To determine the
transaction price, NN decided that the approach that is most predictive of the amount of
consideration to which it will be entitled is the most likely amount. Using the most likely amount, NN
estimates that:
1. Six products will be returned.
2. The costs of recovering the products will be immaterial.
3. The returned products are expected to be resold at a profit.
In relation to the sale, the amount of sale amounted to:

None

P600

P13,400

P24,000

Question 79 1 / 1 point
In relation to the sale, the amount of refund liability:

None

P600

P13,400

P24,000

Question 80 1 / 1 point
In relation to the sale, the cost of sales amounted to:

None

P360

P14,040

P14,400

Question 81 1 / 1 point
In relation to the sale, the amount to be debited to estimated inventory returns:

None

P360

P14,040
P14,400

Question 82 1 / 1 point
When a return occurs, assuming 4 units were returned, the refund liability to be debited or to be
deducted:

None

P240

P360

P400

Question 83 1 / 1 point
In relation to No. 82, the estimated inventory returns to be credited amounted to:

None

P240

P360

P400

Question 84 1 / 1 point
Items 84 to 86 are based on the following information:
Allan Silva Perfume, Inc. sold 3,210 boxed of white musk soap during January of 20x7 at the price of
P90 per box. The company offers a full refund for any product returned within 30 days from the date
of purchase. Based on historical experience, Allan Silva expects that 3% of sales will be returned.
How many performance obligations are there in each sale of a box of soap?
How many performance obligations are in the contract?

Question 85 1 / 1 point
How much gross revenue should Allan Silva recognize in January?

None

P280,233
P280,900

Cannot be determined

Question 86 1 / 1 point
How much net revenue should Allan Silva recognize in January?

None

P280,233

P280,900

Cannot be determined

Question 87 1 / 1 point
Botanic Choice sells natural supplements to customers with an unconditional right of return if they
are not satisfied. The right of return exceeds 60 days. On February 10, 20x4, a customer purchases
P3,000 of products (cost P1,500). Assuming that based on prior experience, estimated returns are
20%. The journal entry to record the return of P200 of merchandise includes a:

Credit to Refund Liability for P200

Credit to Returned Inventory for P100

Credit to Returned Inventory for P100

Debit to Estimated Inventory Returns for P100

Question 88 1 / 1 point
The following transactions were incurred in December 20x5. How much should the company
recognize as revenue for that month?

1.12.20x5 Goods provided to a customer on a sale or return basis. The P35,500


customer has confirmed that 50 per cent of these goods were
sold on to a third party during December.
15.12.20x5 Goods sold including servicing fees for 4 months from 16 10,000
December. The fair value of the goods is P10,000 and the fair
value of 4 months’ servicing fees is P2,500.
31.12.20x5 Goods sold and delivered to customer. The invoiced amount 30,000
is payable in three equal instalments of P10,000 on January 1,
February 1 and March 1, 20x6.
An appropriate discount rate, where appropriate is 1 per cent per month. Other than the amounts
payable in instalments all debts have been paid.

P55,704

P56,000
P73,454

P73,750

Question 89 0 / 1 point
Items 89 and 90 are based on the following information:
On January 1, Stephen Lane enters into a contract with Gloria for the sale of an excavator and spare
parts, the manufacturing lead time is 6 months. On July 1, Gloria pays for the machine and spare
parts, but only takes possession of the machine. Gloria inspects and accepts the spare parts, but
requests that the parts be stored in Stephen Lane’s warehouse because Gloria does not have a place
to store the parts and its premises are very close to Stephen Lane’s warehouse. Stephen Lane
expects to store the spare parts in a separate section of its warehouse for three years. The parts are
available for immediate delivery to Gloria. Stephen Lane cannot use the spare parts or transfer them
to another customer.
How many performance obligations are in the contract?

Question 90 0 / 1 point
When should revenue be recognized?

Over time

Point in time

Over time and point in time

No revenue recognized

No performance obligation

Question 91 1 / 1 point
Horowitz Paint Shop sold P3,000 of paint to a local construction company for cash on June 25, 20x6.
Because of a flood in the area, the customer requested that Horowitz not ship the items from its
warehouse until July 3, 20x6, so Horowitz set aside the paint on June 25, packaged and ready to ship
on July 3. For the second quarter ending on June 30, how much revenue should Horowitz recognize
for the sale to the local construction company?

No contract exists
Zero

P1,500

P3,000

Question 92 0 / 1 point
On January 1, Joey enters into a contract with Althea for the sale of an excavator with unique
specifications. Joey and Althea develop the specifications and Joey contracts with a construction
equipment manufacturer to produce the equipment. The manufacturer will deliver the equipment to
Althea when it is completed. Joey agrees to pay the manufacturer P42,000,000 upon delivery of the
excavator to Althea. Anderson and Althea agree to a selling price of P46,200,000 that will be paid by
Althea to Joey. Joey’s profit is P4,200,000. Joey’s contract with Althea requires Althea to seek
remedies for defects from the manufacturer, but Joey is responsible for any corrections due to errors
in specifications. The role of Joey is a:

Customer

Principal

Agent

No agreement at all

Question 93 1 / 1 point
Thelma and Lilibeth Consulting Services operate a website that links experienced statisticians with
businesses that need data analyzed. Statisticians past their rates, qualifications, and references on
the website, and Thelma and Lilibeth Consulting Services receives 25% of the fee paid to the
statisticians in exchange for identifying potential customers. Gemma, Brigitte and Associate’s
contacts Thelma and Lilibeth Consulting Services and arranges to pay a consultant P4,500 in
exchange for analyzing some data. Thelma and Lilibeth’s income statement would include the
following with respect to this transaction:

Revenue of P4,500

Revenue of P4,500 and cost of services of P3,375

Revenue of P1,125

Revenue of P5,625 and cost of services of P4,500

Question 94 1 / 1 point
Repro, a company which sells photocopying equipment, has prepared its draft financial statements
for the year ended September 30, 20x4. It has included the following transactions in revenue at
stated amount below. Which of these has been correctly included in revenue according to PFRS 15
Revenue from Contracts with Customers?

Agency sales P250,000 on which Repro is entitled to a commission


Sales proceeds of P20,000 for motor vehicles which were no longer required by Repro

Sales of P150,000 on September 30, 20x1. The amount invoiced to and received from
the customer was P180,000, which includes P30,000 for ongoing servicing work to be
done to Repro over the next two years.

Sales of P200,000 on October 1, 20x3 to an established customer which (with the


agreement of Repro will be paid in full on September 30, 20x5. Repro has cost of capital
10%.

Question 95 1 / 1 point
Maybelle Paulino Computers manufactures and sells computers that include a warranty to make
good on any defect in its computers for 150 days (often referred to as an assurance warranty). In
addition, it sells separately an extended warranty, which provides protection from defects for three
years beyond the 150 days (often referred to as a service warranty). How many performance
obligations are in the contract?

Question 96 1 / 1 point
Items 96 and 97 are based on the following information:
Skyfall Vacuums sells the Tornado vacuum cleaner. Each Tornado has a one-year warranty that
covers any product defects. When customers purchase a Tornado, they also have the option to
purchase an extended three-year warranty that covers any breakage or maintenance. The extended
warranty sells for the same amount regardless of whether it is purchased at the same time as the
Tornado or at some other time. How many performance obligations exist in the implied contract for
the purchase of a vacuum cleaner?

Question 97 1 / 1 point
Assume the same facts as in No. 96, but that customers pay 20% less for the extended warranty if
they buy it at the same time they buy a Tornado. How many performance obligations exist in the
implied contract for the purchase of a vacuum cleaner?

0
1

Question 98 1 / 1 point
Maybelle Paulino Computers manufactures and sells computers that include a warranty to make
good on any defect in its computers for 150 days (often referred to as an assurance warranty). In
addition, it sells separately an extended warranty, which provides protection from defects for three
years beyond the 150 days (often referred to as a service warranty). How many performance
obligations are in the contract?

Question 99 1 / 1 point
New Age Computers manufactures and sells pagers and radios paging systems which include a 180
day warranty on product defects. It also sells an extended warranty which provides an additional two
years of protection. On May 10, it sold a paging system for P3,850 and an extended warranty for
another P1,200. The journal entry to record this transaction would include

A credit to Service Revenue of P5,050.

A credit to Service Revenue of P1,200.

A credit to Sales of P3,850 and a credit to Service Revenue of P1,200.

A credit to Unearned Service Revenue of P1,200.

Question 100 1 / 1 point


Entertainment Tonight, Inc. manufactures and sells stereo systems that include an assurance -type
warranty for the first 90 days. Entertainment Tonight also offers an optional extended coverage plan
under which it will repair or replace any defective part for 2 years beyond the expiration of the
assurance-type warranty. The total transaction price for the sale of the stereo system and the
extended warranty is P3,000. The standalone price of each is P2,300 and P800, respectively. The
estimated cost of the assurance-warranty is P350. The accounting for warrant will include a

Debit to Warranty Expense, P800.

Debit to Warranty Liability, P350.

Credit to Warranty Liability, P800.


Credit to Unearned Warranty Revenue, P800.

01 1 / 1 point
Items 101 and 102 are based on the following information:
Zayn Inc. enters into an agreement on March 1, 20x5, to sell Wade Metal Company aluminum bars in
2 months. As part of the agreement, Zayn also agrees to repurchase the bars in 60 days of the
original sales price of P400,000 plus 2%. (Because Zayn has an unconditional obligation to repurchase
the bars at an amount greater than the original sales price, the transaction is treated as a financing.)
On March 1, 20x5 the amount of revenue:

None

P200,000

P400,000

P400,000 plus interest

Question 102 1 / 1 point


On May 1, 20x5 the amount of revenue:

None

P8,000

P400,000

P408,000

Question 103 1 / 1 point


ABC Co., sells a subscription to its anti-virus software along with a subscription renewal option that
allows renewal at half the prevailing price for a new subscription. How many performance
obligations exist in this contract? How many performance obligations are in the contract?

Question 104 1 / 1 point


Edwin and Bryan sells gift cars redeemable for EdBry products either Paul Garcia in-store or online.
During 20x6, Edwin and Bryan sold P6,000,000 of gift cards, and P5,400,000 of the gift cards were
redeemed for products. As of December 31, 20x6, P450,000 of the remaining gift cards had passed
the date at which Edwin and Bryan concludes that the cards will never be redeemed. How much gift
cards will never be redeemed. How much gift card revenue should Edwin and Bryan recognize in
20x6?

P5,400,000

P5,550,000

P5,850,000

P6,000,000

Question 105 1 / 1 point


Bull’s Eye sells gift cards redeemable for Bull’s Eye products either in-store or online. During 20x6,
Bu;;’s Eye sold P2,000,000 of gift cards, and P1,800,000 of the gift cards were redeemed for
products. As of December 31, 20x6, P150,000 of the remaining gift cards had passed the date at
which Bull’s Eyes concludes that the cards will never be redeemed. How much gift card revenue
should Bull’s Eye recognize in 20x6?

P2,000,000

P1,950,000

P1,850,000

P1,800,000

Question 106 1 / 1 point


Items 106 and 107 are based on the following information:
Colombo Coffee sells gift cards thatcan be used at it 55 branches. During 20x5, customers purchased
P25,000 of giftcards, of which P3,000 were redeemed during 20x6. It is estimated that a balance of
P1,500 of cards sold in 20x5 remains unused as of the end of 20x6, and Colombo determines that this
amount will be never be redeemed, based on historical experiences. During 20x6, Colombo further
sold P32,000 of gift cards, of which P26,000 were redeemed and P6,700 remain unused but may be
used by customer in 20x7.
When should revenue be recognized?

Over time

Point in time

No revenue recognized

No performance obligation

Question 107 1 / 1 point


How much gift card revenue should Colombo recognize in 20x6?
P1,500

P3,000

P26,000

P30,500

Question 108 1 / 1 point


Items 108 and 109 are based on the following information:
Moretti Department Store sells gift cards that expire three years from the date of purchase. During
20x4, Moretti sold P50,000 of gift cards, of which P1,500 were redeemed during 20x6. At the end of
206, it is estimated that approximately P800 of 20x4 balance remains unused and Moretti concl udes
that it will never be redeemed. Moretti sold another P55,000 of gift cards I 20x5, of which P22,000
were redeemed in 20x6, and P60,000 of gift cards in 20x6, of which P40,000 were redeemed in 20x6.
When should revenue be recognized?

Over time

Point in time

No revenue recognized

No performance obligation

Question 109 1 / 1 point


How much revenue with respect to gift cards should Moretti recognize in 20x6?

P2,300

P22,000

P40,000

P64,300

THEORIES

1 / 1 point
To address inconsistencies and weaknesses, a comprehensive revenue recognition model was
developed entitled the
Revenue Recognition Principle

Principle-based Revenue Accounting

Rule-based Revenue Accounting

Revenue from Contracts with Customers

Question 2 1 / 1 point
The converged standard on revenue recognition

Reduces the number of disclosures required for revenue reporting.

Increases the complexity of financial statement presentation

Recognizes and measures revenue based on changes in assets and liabilities

Simplifies revenue recognition practices across entities and industries.

Question 3 1 / 1 point
The first step in the process for revenue recognition is to

determine the transaction price.

identify the contract with the customer.

allocate the transaction price to the separate performance obligations.

identify the separate performance obligations in the contract.

Question 4 1 / 1 point
The second step in the process for revenue recognition is to

allocate transaction price to the separate performance obligations.

determine the transaction price.

identify the contract with customers.

identify the separate performance obligations in the contract.

Question 5 1 / 1 point
The third step in the process for revenue recognition is to

determine the transaction price.

identify the separate performance obligations in the contract.

allocate transaction price to the separate performance obligations.


recognize revenue when each performance obligation is satisfied.

Question 6 1 / 1 point
The fourth step in the process for revenue recognition is to

recognize revenue when each performance obligation is satisfied.

identify the separate performance obligations in the contract.

allocate transaction price to the separate performance obligations.

determine the transaction price.

Question 7 1 / 1 point
The last step in the process for revenue recognition is to

allocate transaction price to the separate performance obligations.

recognize revenue when each performance obligation is satisfied.

determine the transaction price.

identify the contract with customers.

Question 8 1 / 1 point
A contract

must be in writing to be an enforceable contract.

is an agreement that creates enforceable rights and obligation.

is enforceable if each party can unilaterally terminate the contract.

does not need to have commercial substance.

Question 9 1 / 1 point
Revenue from a contract with a customer

Is recognized when the customer receive the rights to receive consideration.

Is recognized even if the contract is still wholly unperformed.

Can be recognized even when a contract is still pending.

Cannot be recognized until a contract exists.

Question 10 1 / 1 point
Signing of the contract of by the two parties is
Not recorded until one or both parties perform under the contract.

Recorded at the time the contract is approved by both parties.

Not recorded until both parties perform under the contract.

Recorded immediately after the contract is signed.

Question 11 1 / 1 point
A company must account for a contract modification as a new contract if

Goods or services are interdependent of each other.

The promised goods or services are distinct.

The company has the right to receive consideration equal to standalone price.

Goods or services are distinct and company has right to receive standalone price.

Question 12 1 / 1 point
The transaction price

Excludes discounts, volumes, rebates, coupons and free products, or services.

Is the amount of consideration that a company expects to receive from a customer

Excludes time value of money if the contract involves a significant financing component.

Does not consider noncash consideration such as donations, gifts, equipment or labor.

Question 13 1 / 1 point
Companies can use the expected value to estimate variable consideration when

The contract has only two possible outcomes.

A company has a small number of contracts with similar characteristics.

A company can use the most likely amount in a range of possible outcomes.

A company has a large number of contacts with similar characteristics.

Question 14 1 / 1 point
If a contract involves a significant financing component,

The time value of money is used to determine the fair value of the transaction.

The time value of money is not required to determine the transaction price, if the
payment is more than a year.

The transaction amount should be based on the current sales price of goods or services.

Interest is not accrued as a result of the financing component.

Question 15 1 / 1 point
Noncash consideration should be

Recognized on the basis of fair value of what is given up.

Recognized on the basis of original cost paid by customer.

Recognized on the basis of fair value of what is received.

Recognized on the basis of fair value of equivalent goods or services.

Question 16 1 / 1 point
Consideration paid or payable to customers

Includes volume rebates which increases the cost to the customer.

Includes discounts which reduces the cost of purchases to the company.

Reduces the consideration received and the revenue to be recognized.

Includes prompt settlement discount which increases revenues.

Question 17 1 / 1 point
The transaction price for multiple performance obligations should be allocated

Based on the selling price from the company’s competitors.

Based on what the company could sell the goods for on a standalone basis.

Based on forecasted cost of satisfying performance obligation.

Based on total transaction price less residual value.

Question 18 1 / 1 point
When the bundle price is less than the sum of the standalone prices, the discount should be
allocated:

To the product (or products) associated with the discount.

To the entire bundle of products and services.


To the product cost, thereby increasing product margin.

To the selling price of product or services provided.

Question 19 1 / 1 point
A company has satisfied its performance obligation when the

Company has received payment for goods or services.

Company has significant risks and rewards of ownership.

Company has legal title to the asset.

Company has transferred physical possession of the asset.

Question 20 1 / 1 point
The most popular input measure used to determine the progress towards completion is

Units-of-delivery method.

Cost-to-cost basis.

Labor hours worked.

Tons produced.

Question 21 1 / 1 point
The cost-to-cost basis measures progress towards completion by

Comparing costs incurred to date with total costs to complete the contract.

Tracking results of work completed to date; it is an output measure.

Tracking floors of a building completed versus floors still to be completed.

Tracking miles of highway completed versus miles of highway still to be completed.

Question 22 1 / 1 point
When sales are made with a right of return, the company

Should not recognize any revenue.

Should recognize revenue for the full sales price.

Records the returned asset in a separate inventory account.

Record the estimated returns in the Sales Returns account

Question 23 1 / 1 point
When a company has an obligation or right to repurchase an asset for an amount greater than or
equal to its selling price, the transaction price should be treated as a

Outright sale.

Financing transaction.

Repurchase transaction.

Put option.

Question 24 1 / 1 point
When a customer purchases a product but is not yet ready to accept delivery, this is referred to as

A repurchase agreement.

A consignment.

Principal-agent relationship.

A bill-and-hold arrangement.

Question 25 1 / 1 point
The role of the agent in a Principal-Agent relationship is to

Arrange for the principal to provide goods or services to a customer.

Provide the goods or services for a customer.

Market the principal goods and services to prospective customers.

Develop and maintain goodwill of the principal’s customers.

6 1 / 1 point
The use of the net method of recognizing revenue by an agent

Is appropriate as long as both revenue and costs are included.

Is the correct method in a principal-agent relationship.

Could result in an overstatement of the agent’s revenue.

Could result in an understatement of the agent’s revenue.

Question 27 1 / 1 point
A warranty provided when a customer exercises an option to purchase a warranty is recorded as

An expense in the period the goods or services are sold.


A warranty liability for all costs incurred after sale due to correction of defects.

Revenue in the period that the service-type warranty is in effect.

An assurance type warranty which is included in the sales price of the product.

Question 28 1 / 1 point
Nonrefundable upfront fees

Should be recognized immediately upon receipt of payment

Such as activation fees for cable should be recognized as revenue immediately.

Such as a one-time initiation fee in a health club should be recognized immediately.

Should not be recorded as revenue at the time of payment if they are for future
delivery of products and services.

Question 29 1 / 1 point
Unconditional rights to receive consideration because a performance obligation has been satisfied
are

Reported as a receivable on the statement of financial position.

Reported as a contract asset on the statement of financial position.

Reported as a contract liability on the statement of financial position.

Are not reported on the balance sheet.

Question 30 1 / 1 point
Partial satisfaction of a multiple performance obligation is reported on the statement of financial
position

Contract liability.

Receivable.

Contact asset.

Unearned service revenue.

Question 31 1 / 1 point
Contract liability is a company’s obligations to transfer goods or services to a customer for which the
company has received consideration from the customer. An example of contract liability is

Prepaid subscription.
Unearned magazine subscription.

Mortgage Payable.

Service Revenue.

Question 32 1 / 1 point
Disclosure related to revenue

Does not require capitalized costs to obtain and fulfill a contract.

Does not require judgment that affect amount and timing of revenues from contracts.

Requires disclosure of remaining performance obligations.

Requires disclosure of average balance of contract assets.

Question 33 1 / 1 point
When a contract modification does not result in a separate performance obligation, the additional
products are price at the

Standalone price of the product.

Blended price of original contract and contract modification.

Average selling price of original selling price and standalone price.

Selling price specified in contract modification

Question 34 1 / 1 point
A performance obligation exists when

A company receives the right to receive consideration.

A contract is approved and signed.

A company provides a distinct product or service

A company provides interdependent product or service.

Question 35 1 / 1 point
When multiple performance obligations exists in a contract, they should be accounted for as a single
performance obligation when

Each service is interdependent and interrelated.

The performance obligations are distinct but interdependent.


The product is distinct within the contract.

Determination cannot be made.

Question 36 1 / 1 point
Which of the following transactions is NOT within the scope of PFRS 15 Revenue from Contracts with
Customers?

The sale of non-controlling equity shareholding owned by a retailer

The sale of an investment property owned by a manufacturing company

The sale of an extended warranty by a manufacturer of electrical goods

An agreement licensing another party to se an anti-virus package created by a software


designer

Question 37 1 / 1 point
Step 1 of the PFRS 15 five-step model requires the contract with the customer to be identified. PFRS
15 is applied only if a contract has specific attributes. To which one of the following contracts is PFRS
NOT applied?

An oral contract with a customer to deliver goods at a specified price with payment on delivery

A contract to deliver goods to a customer that entitles the customer to return the goods in
exchange for a full refund

A contract to receive payment from a corporate customer to invest in continued research and
development. Exactly how the money is spent is at the discretion of the recipient.

A contract with a customer that operates in a region that is suffering severe economic recession
meaning that at the inception of the contract, only 90 per cent of the contract price is expected to
be received from the customer.

REVENUE RECOGNITION, CONSTRUCTION


CONTRACT

1 / 1 point
I - Performance Obligations
Pentagon Equipment, Inc., specializes in designing and installing customized manufacturing
equipment. On February 1, 2016, it signs a contract to design a fully automatic wrist watch assembly
line for P2 million, which will be settled in cash upon completion of construction. Pentagon
Equipment will install the equipment on the client's property, furnish it with a Customized software
package that is integral to operations, and provide consulting services that integrate the equipment
with Pentagon's other assembly lines. How many performance obligations exist in this contract?

Answer: 1
Question 2 1 / 1 point
II - Performance Obligations
Cabalen Pools, Inc. constructs outdoor swimming pools for wealthy individuals. Recently it obtained
an order to build a three-lane swimming pool of 25 yards in length in the customer's backyard. Under
the contract, Cabalen is also obligated to install a water heater and a filtration system, which are
necessary to make a swimming pool fully functional. Total price for the construction was P55,000.
Each of these smaller components would typically cost P40.000, P10,000, and P20,000 if installed
separately. Given the information above, how many performance obligations are included in this
contract?

Answer: 1
Question 3 1 / 1 point
III - Timing of Recognition-Over Time or Point in Time
Evelyn Corporation constructs highly specialized communication satellites. A customer in Hong Kong
recently placed an order for a cable TV satellite at a price of P20 million. The order was placed in
April 20x6, and the satellite is to be delivered in one year. The customer has guaranteed to pay in full
at the end of 20x6, regardless of progress or cancellation. Evelyn uses "proportion of time" as its
measure of progress toward completion. When should Evelyn recognize revenue: at completion, or
as the construction is performed?

At completion

As the construction is performed

Question 4 0 / 1 point
IV - Timing of Revenue Recognition-Over Time or Point in Time
EE Construction is constructing a building for AJD, a condominium enterprise. Under the construction
agreement, if for any reason EE can't complete construction, AJD would own the partially completed
building and could the construction company to complete the job. When should EE recognize
revenue as the building is constructed, or after construction is completed?

As the building is constructed

After construction is completed


Question 5 1 / 1 point
V- Transaction Price
Luke Consulting enters into a contract with Holand University to restructure Holand's processes for
purchasing goods from suppliers. The contract states that Luke will earn a fixed fee of P25,000 and
earn an additional P10,000 if Holand achieves P100,000 of cost savings. Luke estimates a 50% chance
that Holand will achieve P100,000 of cost savings. Assuming that Luke determines the transaction
price as the expected value of expected consideration, what transaction price will Luke estimate for
this contract?

Answer: 30,000
Question 6 4 / 4 points
VI – Methods of Construction Accounting
Beavis Construction Company was the low bidder on a construction project to build an earthen dam
for P1,800,000. The project was begun in 20x4 and completed in 20x5. Cost and other data are
presented below:

20x4 20x5
Costs to date . . . . . . . . . P450,000 P1,100,000
Estimated costs to complete . . 1,200,000 0
Billings during the year . . . . 400,000 1,400,000
Cash collected during the year . . 300,000 1,500,000

Required: Compute the amount of gross profit recognized during 20x4 and 20x5.
1. Assume that Beavis uses the overtime (percentage of completion) method for revenue
recognition.
a.
a. 20x4
b. 20x5
2. Assume that Beavis uses the point-in-time (cost recovery) method for revenue
recognition.
a.
a. 20x4
b. 20x5

Answer for blank # 1: 40,905 (25 %)


Answer for blank # 2: 209,095 (25 %)
Answer for blank # 3: 0 (25 %)
Answer for blank # 4: 250,000 (25 %)
Question 7 6 / 6 points
VII - Methods of Construction Accounting
Beck Construction Company began work on a new building project on January 1, 20x4. The project is
to be completed by December 31, 20x6, for a fixed price of P108 million. The following are the actual
costs incurred and estimates of remaining costs to complete the project that were made by Beck's
accounting staff:

Actual Costs Estimated remaining costs to


incurred in each year complete the project measured at
Dec. 31 of each year
20x4 P30,000,000 P60,000,000
20x5 P45,000,000 P45,000,000
20x6 P35,000,000 P0
Required:
1. What amount of gross profit (or loss) would Beck record on this project in each year under
the overtime (percentage-of-completion) method?
a. 20x4
b. 20x5
c. 20x6
2. What amount of gross profit (or loss) would Beck record on this project in each year under
the point-in-time (cost recovery) method?
a. 20x4
b. 20x5
c. 20x6

Answer for blank # 1: 6,000,000 (2,000,000)


Answer for blank # 2: -18,000,000 (16.67 %)
Answer for blank # 3: 10,000,000 (16.67 %)
Answer for blank # 4: 0 (16.67 %)
Answer for blank # 5: -12,000,000 (16.67 %)
Answer for blank # 6: 10,000,000 (16.67 %)
Question 8 9 / 10 points
VIII - Input Measures: Overtime/Percentage-of-Completion (Cost-to-Cost) Method versus
Point-in-Time/Cost Recovery Method
DJD Builders has a fixed price contract to build a waiting shed. The initial amount of revenue agreed
is P528,000. At the beginning of the contract on January 1, 20x3 the initial estimate of the
construction costs is P480,000. By the end of 20x3 the estimate of the total costs has risen to
P484,800.
During 20x4 the customer agrees to a variation with increases expected revenue from fire contract
by P12,000 and causes additional costs of P7.200. At the end of 20x4 there de materials stored on
site for use during the following period which cost P6,000.
DJD Builders have decided to determine the stage of completion of the contract by calculating the
proportion that contract costs incurred for work to date bear to the latest estimated total contract
costs. The contract costs incurred at the end of each year (costs incurred to date), billings and
collections for each year were as follows:

Year Direct and Allocable Costs to date Billings Collections


20x3 P126,048 P144,000 P120,000
20x4 370,080 (including materials in store) 240,000 228,000
20x5 492,000 156,000 192,000

Required:
1. Compute the Current asset-Contract Asset/Current liability- Contract Liability:
a. Over Time/Percentage-of-completion method using cost-to-cost method (express
Contract Asset as positive and Current liability as negative)
1.
i. 20x3
ii. 20x4
2.
b. Point-in-Time/Cost recovery method (also known as zero-profit approach)
1.
i. 20x3
ii. 20x4
3. Compute the gross profit.
c. Over Time/Percentage-of-completion method using cost-to-cost method
1.
i. 20x3
ii. 20x4
iii. 20x5
4.
d. Point-in-Time/Cost recovery method (also known as zero-profit approach)
1.
i. 20x3
ii. 20x4
iii. 20x5

Answer for blank # 1: -6,720 (10 %)


Answer for blank # 2: 15,600 (10 %)
Answer for blank # 3: -6,720 (10 %)
Answer for blank # 4: -19,920 (10 %)
Answer for blank # 5: 11,232 (10 %)
Answer for blank # 6: 24,288 (10 %)
Answer for blank # 7: 12,480 (10 %)
Answer for blank # 8: 0 (10 %)
Answer for blank # 9: 0 (10 %)
Answer for blank # 10: 48,000 (10 %)
Question 9 4 / 4 points
X - Incomplete Data
In 20x5, DJ Builders Construction began work on a three-year construction project to build a new
performing arts complex (the "PAC"). The PAC contract price is P150 million. DJ Builders uses the
overtime/percentage-of-completion method of accounting. At the end of 20x5, the following
financial statement information indicates the results to date for the PAC:

Income Statement:
Revenue. . . . . . . . . . . . . . . . P ? million
Cost of Construction . . . . . . . . . 35 million
Gross profit . . . . . . . . . P ? million
Balance Sheet:
Accounts Receivable from construction billings . . . . . . P14 million
Construction in progress . . . . . . . . . . . . . . . . . . 50 million
Less: Billings on construction . . . . . . . . . . . . . . . (P ? million)
Net billings in excess of construction in progress . . . . . . . . . P ? million
Cash Flow Statement
Cash Collections . . . . . . . . . . . . . . . . . . P 46 million

Required: Compute the following, placing your answer in the spaces provided and supporting
computations below:
1. Total revenue recognized during 20x5
2. Billings on construction
3. Net in excess of construction in progress
4. Calculate the percentage of completion during 20x5

Answer for blank # 1: 50,000,000 (25 %)


Answer for blank # 2: 60,000,000 (15,000,000)
Answer for blank # 3: 8,000,000 (60,000,000)
Answer for blank # 4: 33.33% (10,000,000)
Question 10 4 / 4 points
XI - Incomplete Data
In 20x4, Joey Building Inc. began work on a four-year construction project The contract price is P300
million. Joey uses the overtime/percentage-of-completion method accounting. At the end of 20x5,
the following financial statement information indicates the results to date for Cincy One:

Income Statement:
Gross profit (before taxes) recognized in 20x4 P 22 million
Balance Sheet:
Accounts Receivable from construction billings P14 million
Construction in progress P 66 million
Less: Billings on construction (75 million)
Net billings in excess of construction in progress P 9 million
Cash Flow Statement
Cash Collection P 46 million
Required: Compute the following, placing your answer in the spaces provided and supporting
computations below:
1. Cash collected by Joey on Cincy One during 20x4
2. Actual costs incurred by Joey on Cincy One during 20x4
3. On December 31, 20x4, the estimated remaining costs to complete
4. The percentage of Cincy One that was completed during 20x4

Answer for blank # 1: 65,000,000 (25 %)


Answer for blank # 2: 44,000,000 (25 %)
Answer for blank # 3: 156,000,000 (25 %)
Answer for blank # 4: 22% (25 %)
Question 11 2 / 2 points
XII - Incomplete Data
In 20x4, Johnson Builders began construction work under a three-year contract at a price d
P7,525,000. The firm uses the overtime/percentage-of-completion method for finances accounting
purposes. The income to be recognized each year is based on the proportion of cost incurred to the
total estimated costs for completing the contract. The financial statement presentations rel ating to
this contract on December 31, 20x4, are:

Balance Sheet:
Accounts Receivable P 150,000
Construction in progress P 602,000
Less: Progress billings 562,000 40,000
Income Statement
Gross profit on construction contracts P 301,000

Required: Determine the:


1. Cash collected in 20x4.
2. Estimated income on the construction contract.

Answer for blank # 1: 411,500 (50 %)


Answer for blank # 2: 2,508,333 (50 %)
Question 12 19 / 24 points
XIII - Computation of CIP, net and Progress Billings
On January 1, 2005, Cleveland Enterprises obtained a contract to construct a building. It was
estimated at the beginning of the contract that it would take three years to complete the project at
an expected cost of P200,000. The contract price was P250,000. The following information describes
the status of the job at the close of production each year:

20x4 20x5 20x6


Actual costs incurred P110,000 P120,000 P15,000
Estimated costs to complete 100,000 20,000 0
Billings on contract 125,000 125,000 0
Collections on contract 120,000 120,000 10,000
Required: Compute the items listed below for each year assuming (round all percentages decimals)
1. The use of the overtime/percentage-of-completion cost-to-cost method, and
2. The point-in-time/cost recovery (zero-profit) method
Place your answer in the spaces provided and showing supporting computations below:
Percentage-of-completion method (Cost-to-cost approach)

20x4 20x5 20x6


Revenue recognized during the year (1) (6) (11)
Gross profit recognized during the year (2) (7) (12)
Balance in the construction in progress (CIP) account (3) (8) 0
at December 31 after closing entries)
Balance in the progress billing account at December (4) (9) 0
31 (after closing entries)
The balance in CIP, net or Progress Billings, net (5) (10) 0

Cost Recovery Method

20x4 20x5 20x6


Revenue recognized during the year (13) (18) (23)
Gross profit recognized during the year (14) (19) (24)
Balance in the construction in progress (CIP) account (15) (20) 0
at December 31 after closing entries)
Balance in the progress billing account at December (16) (21) 0
31 (after closing entries)
The balance in CIP, net or Progress Billings, net (17) (22) 0

Answer for blank # 1: 130,000 (4.17 %)


Answer for blank # 2: 20,000 (4.17 %)
Answer for blank # 3: 130,000 (4.17 %)
Answer for blank # 4: 125,000 (4.17 %)
Answer for blank # 5: 5,000 (4.17 %)
Answer for blank # 6: 100,000 (4.17 %)
Answer for blank # 7: -15,000 (4.17 %)
Answer for blank # 8: 235,000 (4.17 %)
Answer for blank # 9: 250,000 (4.17 %)
Answer for blank # 10: -15,000 (4.17 %)
Answer for blank # 11: 15000 (15,000)
Answer for blank # 12: 0 (4.17 %)
Answer for blank # 13: 110,000 (4.17 %)
Answer for blank # 14: 0 (4.17 %)
Answer for blank # 15: 110,000 (4.17 %)
Answer for blank # 16: 125,000 (4.17 %)
Answer for blank # 17: -15,000 (4.17 %)
Answer for blank # 18: 120,000 (4.17 %)
Answer for blank # 19: 0 (4.17 %)
Answer for blank # 20: 230,000 (4.17 %)
Answer for blank # 21: 250,000 (4.17 %)
Answer for blank # 22: -20,000 (4.17 %)
Answer for blank # 23: 20,000 (4.17 %)
Answer for blank # 24: 5,000 (4.17 %)
Question 13 21 / 24 points
XIV - Computation of CIP, net and Progress Billings
On January 1, 20x4, Edwards Inc. obtained a contract to construct a building. It was estimated at the
beginning of the contract that it would take 3 years to complete the project at an expected cost
P200.000. The contract price was P250,000. The following information describes the status of the job
at the close of production each year:

20x4 20x5 20x6


Actual costs incurred P150,000 P100,000 P15,000
Estimated costs to 90,000 20,000 0
complete
Billings on contract 110,000 120,000 20,000
Collections on contract 100,000 120,000 30,000

Required: Compute the items listed below for each year assuming (round all percentages to two
decimals)
1. The use of the overtime/percentage-of-completion cost-to-cost method, and
2. The point-in-time/cost recovery (zero-profit) method
Place your answer in the spaces provided and showing supporting

20x4 20x5 20x6


Revenue recognized during the year (1) (6) (11)
Gross profit recognized during the year (2) (7) (12)
Balance in the construction in progress (CIP) account (3) (8) (13)
at December 31 after closing entries)
Balance in the progress billing account at December (4) (9) (14)
31 (after closing entries)
The balance in CIP, net or Progress Billings, net (5) (10) (15)

Percentage-of-completion method (Cost-to-cost approach)


20x4 20x5 20x6
Revenue recognized during the year (1) (6) (11)
Gross profit recognized during the year (2) (7) (12)
Balance in the construction in progress (CIP) account (3) (8) 0
at December 31 after closing entries)
Balance in the progress billing account at December (4) (9) 0
31 (after closing entries)
The balance in CIP, net or Progress Billings, net (5) (10) 0

Cost Recovery Method

20x4 20x5 20x6


Revenue recognized during the year (13) (18) (23)
Gross profit recognized during the year (14) (19) (24)
Balance in the construction in progress (CIP) account (15) (20) 0
at December 31 after closing entries)
Balance in the progress billing account at December (16) (21) 0
31 (after closing entries)
The balance in CIP, net or Progress Billings, net (17) (22) 0

Answer for blank # 1: 157,500 (150,000)


Answer for blank # 2: 7,500 (4.17 %)
Answer for blank # 3: 157,500 (4.17 %)
Answer for blank # 4: 110,000 (4.17 %)
Answer for blank # 5: 47,500 (4.17 %)
Answer for blank # 6: 75,000 (102,500)
Answer for blank # 7: -27,500 (4.17 %)
Answer for blank # 8: 230,000 (4.17 %)
Answer for blank # 9: 230,000 (4.17 %)
Answer for blank # 10: 0 (4.17 %)
Answer for blank # 11: 17,500 (12,500)
Answer for blank # 12: 5,000 (4.17 %)
Answer for blank # 13: 157,500 (150,000)
Answer for blank # 14: 0 (4.17 %)
Answer for blank # 15: 150,000 (4.17 %)
Answer for blank # 16: 110,000 (4.17 %)
Answer for blank # 17: 40,000 (4.17 %)
Answer for blank # 18: 75,000 (80,000)
Answer for blank # 19: -20,000 (4.17 %)
Answer for blank # 20: 230,000 (4.17 %)
Answer for blank # 21: 230,000 (4.17 %)
Answer for blank # 22: 0 (4.17 %)
Answer for blank # 23: 17,500 (20,000)
Answer for blank # 24: 5,000 (4.17 %)

1 / 1 point
DJD Construction is constructing a building for Hotel Dian. Under the construction agreement if for
any reason DJD can’t complete construction, Hotel Dian would own the partially completed building
and could hire another construction company to complete the job. When should DJD recognize
revenue, as the building is constructed, or after construction is completed?

Over time

Point in time

No revenue recognized.

No performance obligation.

Question 2 1 / 1 point
On January 1, 20x6, Silver Construction Company signed a contract to build a garage for a customer
and received P10,000 in advance for the job. The new garage will be built on the customer’s land. To
complete this project, Silver must first build a concrete floor, construct wooden pillar and walls, and
finally install a roof. Silver normally charges stand-alone selling prices of P3,000, P4,000, and P5,000,
respectively, for each of these three smaller tasks if done separately. How many performance
obligations exists in this contract?

Question 3 1 / 1 point
DJ builders Construction builds luxury houses in remote areas. On June 1, 20x6, the company signed
a contract to build a house in an underdeveloped section of a mountainside, and receive P2 million in
advance for the job. To complete the project, the company must construct a pathway in leading to
the building lot, clear a large hillside, and construct a wooden house. Normally, the company would
charge P400,000, P1,400,000, and P500,00, respectively, for each of these tasks if done separately.
Given the information above, how many performance obligations are included in this contract?
0

Question 4 1 / 1 point
DJ Builders Construction enters into a contract with a customer to build a warehouse for P850,000
on March 30, 20x5 with a performance bonus of P50,000 if the building is completed by July 31,
20x5. The bonus is reduced by P10,000 each week that completion is delayed. DJ Builders commonly
includes these completion bonuses in its contracts and, based on prior experience, estimates the
following completion outcomes:

Completed by Probability
July 31, 20x5 65%
August 7, 20x5 25%
August 14, 20x5 5%
August 21, 20x5 5%

The transaction price amounted to:

P585,000

P895,000

P850,000

P552,500

Question 5 3 / 3 points
Use the following information for the questions 5 through 7:
Nair A.G. enters into a contract with a customer to build an apartment building for P1,800,000. The
customer hopes to rent apartments at the beginning of the school year and provide a performance
bonus of P270,000 to be paid if the building is ready for rental beginning August 1, 20x4. The bonus
is reduced by P90,000 each week that completion is delayed. Nair commonly includes these
completion bonuses in its contracts and, based on prior experiences, estimate the following
completion outcomes:

Completed by Probability
August 1, 20x4 70%
August 8, 20x4 20%
August 15, 20x4 5%
After August 15, 20x5 5%
Determine the transaction price for this contract:
a. P1,800,000
b. P2,029,500
c. P2,070,000
d. P2,160,000

___B___(33.33 %)
Determine the transaction price for the contract, assuming Nair is only able to estimate whether the
building can be completed by August 1, 20x4, or not (Nair estimates that there is a 70% chance that
the building will be completed by August 1, 20x4):
a. P1,800,000
b. P2,029,500
c. P2,070,000
d. P2,160,000

___C___(33.33 %)
Determine the transaction price for the contract, assuming Nair has limited information with which
to develop a reliable estimate of completion by the August 1, 20x4 deadline:
a. Zero
b. P1,800,000
c. P2,070,000
d. P2,160,000

___A___(33.33 %)
0 / 1 point
Mediocre Inc. has entered into a very profitable fixed price contract for constructing a high-rise
building over a period of three years. It incurs the following costs relating to the contract during the
first year:
• Cost of material = P2.5 million
• Site labor cost = P2.0 million
• Agreed administrative costs as per contract to be reimbursed by the customer = P1 million.
• Depreciation of the plant used for the construction = P0.5 million.
• Marketing costs for selling apartments, when they are ready = P1.0 million.
• Depreciation of idle plant and equipment =P.5 million

Total estimated cost of the project = P18 million


The percentage of completion (overtime) of this contract at year-end is:

33 1/3% (= 6.0/18.0)

27% (= 4.5/16.5)

25% (= 4.5/18.0)
39% (= 7.0/18.0)

Question 7 1 / 1 point
Digger commenced contract X47 on July 1, 20x3. Performance obligations under the contract are to
be satisfied over time and the stage of completion is regularly assessed. Details for first year of the
contract were as follows:

Amounts invoiced P2,400,000


Costs incurred at date of last assessment 1,800,000
Costs incurred since last assessment 200,000
Amount received 2,100,000
Total contract price 4,200,000
Estimated costs to complete 1,200,000
Survey of performance completed 2,520,000

Digger invoices the customer immediately on receiving an assessment of the value of the work done.
What amount should Digger include as cost of sales for the X47 contract for the year ended June 30,
20x4, assuming revenue is based on performance completed?

P1,800,000

P1,828,000

P1,920,000

P2,000,000

Question 8 1 / 1 point
Augustus is involved in a number of contracts at September 30, 2x03, which have been assessed as
contracts where the performance obligation is satisfied over a period of time. The company
calculates revenue on an output basis using the value of performance completed. At that date the
following information is available with respect to contract ZX45:

Contract price P225,000


Cost incurred to date 115,000
Estimated further costs to completion 65,000
Survey of performance completed 125,000
Amount invoiced and paid by customer 145,000

This contract is with a new customer. Augustus required the customer to pay an up -front deposit
that is included in the amounts invoiced and paid. What amount should be included in the statement
of financial position of Augustus in respect of contract ZX45 as of September 30, 20x3?

Nil

P5,000 liability
P5,000 asset

P20,000 liability

Question 9 1 / 1 point
A construction company entered into a contract on January 1, 20x5 to build a factory on behalf of a
manufacturer. The performance obligation within the contract is determined to be satisfied over
time. The total contract price was P2.8 million. At December 31, 20x5 the contract was as 35 per cent
complete. Cost incurred during the year were P740,000 and costs to complete are estimated at
P1,400,000, P1,000,000 has been billed to the customer but not yet paid. What amounts are
recognized in respect of this contract in the statement of financial position of the construction
company as at December 31, 20x5?

a receivable of P1,000,000

a contract asset of P1,000,000

a receivable of P1,000,000 and a contract asset of P20,000

a receivable of P1,000,000 and a contract liability of P20,000

Question 10 1 / 1 point
A construction contract has a fixed price contract for P100,000 to construct a building of a design
that has never before been constructed and using materials that have never before been used in the
construction of building (the project). The contractor began construction of the building in 20x4 and
expects that construction will take at least five years. In 20x4 the contractor incurred P5,000 contract
costs on the project. At the end of 20x4 the contractor cannot estimate the outcome of the contract
with sufficient reliability to estimate the project’s percentage of completion (i.e., because of the
uncertainties arising from the new design and new materials the entity cannot estimate total
expected contract costs with sufficient reliability). It is highly likely that the contract price will be
received from the customer. At the end of 20x4, the contractor must recognize revenue of:

P-0-

P5,000

P100,000

Incomplete data

Question 11 1 / 1 point
Monroe Construction Company uses the percentage-of-completion (overtime) method of accounting.
In 20x4, Monroe began work on a contract it had received which provided a contract price of
P15,000,000. Other details follow:

20x4
Costs incurred during the year P7,200,000
Estimated costs to complete as of December 31 4,800,000
Billings during the year 6,600,000
Collections during the year 3,900,000

What should be the gross profit recognized in 20x4?

P600,000

P1,800,000

P3,000,000

P7,000,000

Question 12 1 / 1 point
Adler Construction Co. uses the percentage-of-completion (overtime) method. In 20x4, Adler began
work on a contract for P3,300,000 and it was completed in 20x5. Data on the costs are:

Year Ended December 31


20x4 20x5
Costs incurred P1,170,000 P840,000
Estimated costs to complete 780,000 -

For the years (1) 20x4 and (2) 20x5, Adler should recognize gross profit of

(1) P0; (2) P1,290,000

(1) P774,000; (2) P516,000

(1) P810,000; (2) P480,000

(1) P810,000; (2) P1,290,000

Question 13 1 / 1 point
AJD Company recognizes construction revenue and expenses using the percentage completion (over
time) method. During 20x4, a single long-term project was begun which continued through 20x5,
information on the project were as follows:

20x4 20x5
Accounts Receivable from P200,000 P600,000
Construction Contracts
Construction expenses 210,000 384,000
Construction in progress 244,000 728,000
Partial billings on contract 200,000 840,000

The profit recognized from the long-term construction contract should amount to (1) 20x4; (2) 20x5:

(1) P44,000; (2) P456,000


(1) P44,000; (2) P200,000

(1) P34,000; (2) P256,000

(1) P34,000; (2) P100,000

Question 14 2 / 2 points
Use the following information for questions 16 and 17:
Gomez, Inc. began work in 20x4 on contract #3814, which provided for a contract price of
P7,200,000. Other details follow:

20x4 20x5
Costs incurred during the year P1,200,000 P3,675,000
Estimated costs to complete, 12/31/31 3,600,000 0
Billings during the year 1,350,000 5,400,000
Collections during this year 900,000 5,850,000

Assume that Gomes uses the percentage-of-completion (overtime) method of accounting. The
portion of the total gross profit to be recognized as income in 20x4 is:
a. P450,000
b. P600,000
c. P1,800,000
d. P7,200,000

___B___(50 %)
Assume that Gomes uses the cost recovery method (point-in-time) method of accounting. The
portion of the total gross profit to be recognized as income in 20x4 is:
a. P450,000
b. P600,000
c. P1,800,000
d. P7,200,000

___C___(50 %)
1 / 1 point
Tyro Construction Company has two projects, for which it reported, as of December 31, 20x5, the
following information:

In thousand pesos: Project A Project B


Contract price P4,800 P860
20x4: Costs incurred P3,400
Percent completed 75%
20x5: Costs incurred P1,250 P140
Percent completed 25% 15%
Using the percentage-of-completion (overtime) method of revenue recognition, gross profit on
Project A to be recognized in 20x4 would be:

P200,000

P300,000

P400,000

P900,000

Question 16 2 / 2 points
Use the following information for questions 19 and 20:
Kiner, Inc. began work in 20x4 on a contract for P8,400,000. Other data are as follows:

20x4 20x5
Costs incurred to date P3,600,000 P5,600,000
Estimated costs to complete 2,400,000 0
Billings during the year 2.800,000 8,400,000
Collections to year 2,000,000 7,200,000

If Kiner uses the percentage-of-completion (over time) method, the gross profit to be recognized in
20x4 is
a. P1,440,000
b. P1,600,000
c. P2,160,000
d. P2,400,000

___A___(50 %)
If Kiner uses the cost recovery method (point-in-time), the gross profit to be recognized in 20x5 is:
a. P1,360,000
b. P2,800,000
c. P1,400,000
d. P5,600,000

___B___(50 %)
2 / 2 points
Horner Construction Co uses the overtime/percentage-of-completion method. In 20x4, Horner began
work on a contract for P5,500,000. It was completed in 20x5. The following cost data pertain to this
contract.

Year Ended December 31


20x4 20x5
Costs incurred P1,950,000 P1,400,000
Estimated costs to complete at 1,300,000 -
the end of year

The amount of gross profit to be recognized on the income statement for the year ended December
31, 20x5 is:
a. P800,000
b. P860,000
c. P900,000
d. P2,150,000

___A___(50 %)
If the point-in-time/cost recovery (zero profit approach) of accounting was used, the amount of gross
profit to be recognized in years (1) 20x4 and (2) 20x5 would be:
a. (1) P2,250,000; (2) P0
b. (1) P2,150,000; (2) P(100,000)
c. (1) P0; (2) P2,150,000
d. (1) P0; (2) P2,250,000

___C___(50 %)
1 / 1 point
On October 31, 20x4, Mr. Cruz bought property from D’Vision Heights which had earlier cost the
latter P250,000. The company received a down payment of P100,000 and a P400,000 mortgage note
payable in twenty equal semiannual installments plus 16% interest per annum on unpaid principal.
Assuming the gross profit is recognized in the period of sale, the amount of gross profit to be
recognized by D’Vision Heights in 20x6 would be:

P0

P50,000

P100,000

P250,000

Question 19 1 / 1 point
Hayes Construction Company contracted to construct a building for P1,500,000. Construction began
in 20x4 and was completed in 20x5. Data relating to the contract are summarized below:

Year Ended December 31


20x4 20x5
Costs incurred P600,000 P450,000
Estimated costs to complete at 400,000 -
the end of year
Hayes uses the percentage-of-completion method (overtime) as the basis for income recognition. For
the years ended December 31, 20x4, and 20x5, respectively, Hayes should report gross profit of:

P270,000 and P180,000

P900,000 and P600,000

P300,000 and P150,000

P0 and P450,000

Question 20 1 / 1 point
Ube Construction Company has consistently used the overtime/percentage-of-completion method.
On January 10, 20x4, Ube began work on a P6,000,000 construction contract. At the inception date,
the estimated cost of construction was P4,500,000. The following data relate to the progress of the
contract:

Income recognized at 12/31/20x4 P600,000


Cost incurred 1/10/20x4 through 12/31/20x5 3,600,000
Estimated cost to complete at 12/31/20x5 1,200,000

How much income should Ube recognize for the year ended December 31, 20x5?

P300,000

P535,000

P600,000

P900,000

Question 21 1 / 1 point
Layton Construction Company has consistently used the percentage-of completion method
(overtime) of recognizing income. During 20x4, Layton entered into a fixed-price contract to
construct an office building for P10,000,000. Information relating to the contract is as follows:

December 31
20x4 20x5
Percentage of completion . . . . . . . . . . . . . . . . . . 20% 60%
Estimated total cost at completion . . . . . . . . . . . . P7,500,000 P8,000,000
Income recognized (cumulative) . . . . . . . . . . . . . . 500,000 1,200,000
Contract costs incurred during 20x5 were:

P3,200,000

P3,300,000
P3,500,000

P4,800,000

Question 22 1 / 1 point
Remington Construction Company uses the percentage-of-completion method (overtime). During
20x4, the company entered into a fixed-price contract to construct a building for Sherman Company
for P30,000,000. The following details pertain to the contract:

At December 31, 20x4 At December 31, 20x5


Percentage of completion . . . . . . . . . 25% 60%
Estimated total cost of contract . . . . . . P22,500,000 P25,000,000
Gross profit recognized to date . . . . . . 1,875,000 3,000,000
The amount of construction costs incurred during 20x5 was

P15,000,000

P9,375,000

P5,625,000

P2,500,000

Question 23 1 / 1 point
The Naples Company uses the overtime/percentage-of-completion method and the point in-
time/cost-to-cost method for its long-term construction contracts. On one such contract, Naples
expects total revenues of P260,000 and total costs of P200,000. During the first year, Naples incurred
costs of P50,000 and billed the customer P30,000 under the contract. At what net amount should
Naples' Construction in Progress for this contract be reported at the end of the first year?

P30,000

P35,000

P50,000

P65,000

Question 24 1 / 1 point
Lark Corp. has a contract to construct a P5,000,000 cruise ship at an estimated cost of P4,000,000.
The company will begin construction of the cruise ship in early January 20x4 and expects to complete
the project sometime in late 20x7. Lark Corp. has never constructed a cruise ship before, and th e
customer has never operated a cruise ship. Due to this and other circumstances, Lark Corp. believes
there are inherent hazards in the contract beyond the normal, recurring business risks. Lark Corp.
expects to recover all its costs under the contract. During 20x4 and 20x5, the company has the
following activity:
20x4 20x5
Costs to date P 980,000 P2,040,000
Estimated costs to complete 3,020,000 1,960,000
Progress billings during the year 1,000,000 1,000,000
Cash collected during the year 648,000 1,280,000

For the year ended December 31, 20x5, how much revenue should Lark Corp. recognize on its
income statement?

P980,000

P2,040,000

P1,300,000

P1,060,000

Question 25 1 / 1 point
Lark Corp. has a contract to construct a P5,000,000 cruise ship at an estimated cost of P4,000,000.
The company will begin construction of the cruise ship in early January 20x4 and expects to complete
the project sometime in late 20x7. Lark Corp. has never constructed a cruise ship before, and th e
customer has never operated a cruise ship. Due to this and other circumstances, Lark Corp. believes
there are inherent hazards in the contract beyond the normal, recurring business risks. Lark Corp.
expects to recover all its costs under the contract. During 20x4 and 20x5, the company has the
following activity:
20x4 20x5
Costs to date P 980,000 P2,040,000
Estimated costs to complete 3,020,000 1,960,000
Progress billings during the year 1,000,000 1,000,000
Cash collected during the year 648,000 1,280,000

On its statement of financial position at December 31, 20x5, what amount will be reported related to
the Construction in Process account?

P40,000 costs in excess of billings.

P1,020,000 costs in excess of billings.

P40,000 billings in excess of costs.

P20,000 billings in excess of costs.

Question 26 1 / 1 point
Lark Corp. has a contract to construct a P5,000,000 cruise ship at an estimated cost of P4,000,000.
The company will begin construction of the cruise ship in early January 20x4 and expects to complete
the project sometime in late 20x7. Lark Corp. has never constructed a cruise ship before, and the
customer has never operated a cruise ship. Due to this and other circumstances, Lark Corp. believes
there are inherent hazards in the contract beyond the normal, recurring business risks. Lark Corp.
expects to recover all its costs under the contract. During 20x4 and 20x5, the company has the
following activity:
20x4 20x5
Costs to date P 980,000 P2,040,000
Estimated costs to complete 3,020,000 1,960,000
Progress billings during the year 1,000,000 1,000,000
Cash collected during the year 648,000 1,280,000

On its statement of financial position at December 31, 20x5, what amount will be reported for
accounts receivable?

P352,000.

P720,000

P72,000

P2,000,000

Question 27 1 / 1 point
For a construction firm using the cost recovery method, if costs exceed billings on some contracts by
P1,000,000 and billings exceed costs by P800,000 on others, the contracts should ordinarily be
reported as a

current asset of P200,000.

current liability of P200,000.

current asset of P1,000,000 less a contra-current asset of P800,000.

current asset of P1,000,000 and a current liability of P800,000.

8 1 / 1 point
The Key Largo Company uses the overtime/percentage-of-completion method to recognize profits on
long-term contracts. At the end of the second year of the contract, a project was 70% complete and
an overall loss of P100,000 was expected. A P20,000 profit had been recognized in the first year of
the contract. The loss to be recognized in the second year is:

P 70,000

P 80,000

P120,000

P 100,000

None of these
Question 29 1 / 1 point
Hiser Builders, Inc. is using the point-in-time/cost recovery method for a P5.600.000 contract that
will take two years to complete. Data at December 31, 20x4, the end of the first year are as follows:

Costs incurred to date. . . . . . . . . . . . P 2,560,000


Estimated costs to complete. . . . . . . . . 3,280,000
Billings to date. . . . . . . . . . . . . . . . . . 2,400,000
Collections to date. . . . . . . . . . . . . . . 2,000,000
The gross profit or loss that should be recognized for 20x4 is

P -0-

a P240,000 loss

P120,000 loss

P105.600 loss

Question 30 1 / 1 point
Lake Construction Company uses the overtime/percentage-of-completion method for long- term
construction contracts. The company has a project with a contract price of P7,000 on which P600 of
gross profit has been recognized in prior years. Information for the current year is as follows:

Total cost incurred through current year. . . . . . . P5,000


Estimated costs remaining at end of current year. . 2,800
.

What is the loss that Lake should recognize in the current year?

P600

P800

P1,400

No loss should be recognized.

Question 31 3 / 4 points
Use the following information for questions 36 to 39:
AJD Builders is building a multi-unit residential complex. In the prior year, AJD Builders entered into a
contract with a customer for a specific unit that is under construction. AJD Builders has determined
that the contract is a single performance obligation satisfied over time AJD Builders g athered the
following information for the contract during the year.
AJD Builders - Year Ended December 31, 20x8
Costs to date. . . . . . . . . . . . . . . . . . . . . . . . . . . . . . . . . . . . P15,000,000
Future expected costs. . . . . . . . . . . . . . . . . . . . . . . . . . . . . . 10,000,000
Work certified to date. . . . . . . . . . . . . . . . . . . . . . . . . . . . . . 18,000,000
Expected sales value. . . . . . . . . . . . . . . . . . . . . . . . . . . . . . . 32,000,000
Revenue taken in earlier periods. . . . . . . . . . . . . . . . . . . . . . . . P12,000,000
Cost taken in earlier periods. . . . . . . . . . . . . . . . . . . . . . . . . 9,500,000
*same amount for both methods (output and input method)

Calculate the figures to be included in the statement of profit or loss in respect of revenue and costs
for the year ended December 31, 20x8 on both methods (I) Input Method (Cost basis), and (II)
Output Method (Sales Basis):
Calculate total expected profit for the year 20x8:
a. P22,000,000
b. P17,000.000
c. P 7,000,000
d. P 4,000,000

___C___(25 %)
Measure of progress towards completion (percentage of completion)
Input/Cost Basis Output/Sales Basis Input/Cost Basis Output/Sales Basis
a. 60.00% 56.25% c. 45.45% 32.25%
b. 56.25% 60.00% d. 32.25% 45.45%
___A___ (B)
What amounts should be shown in the statement of profit or for revenue, cost of sales and profit the
year using an output method (sales basis)?
Revenue Cost of Sales Gross Profit
a. P 7,200,000 P 5,500,000 P 1,700,000
b. P 6,000,000 P 4,562,500 P 1,437,500
c. P 12,000,000 P 9,500,000 P 2,500,000
d. P 19,200,000 P 15,000,000 P 4,200,000
___B___(25 %)
What amounts should be shown in the statement of profit or for revenue, cost of sales for the year
using an Input method (cost basis)?
Revenue Cost of Sales Gross Profit
a) P 7,200,000 P 5,500,000 P 1,700,000
b) P 6,000,000 P 4,562,500 P 1,437,500
c) P 12,000,000 P 9,500,000 P 2,500,000
d) P 19,200,000 P 15,000,000 P 4,200,000
___D___ (A)
4 / 4 points
Use the following information for questions 40 to 43:
Anton Builders constructs storage house. The projects generally take a number of months to
complete. The company has three contracts in progress at the year ended 30 April:
J K L
Costs incurred to date P320,000 P 540,000 P 20,000
Costs to complete 40,000 90,000 220,00
Contract price 416,000 384,000 300,000
Work certified to date 312,000 456,000
Progress Payments 250,000 480,000

Anton Builders accrues profit on its construction contracts using the percentage of
completion derived from the sales earned as work certified compared to the total sales value.
Calculate total expected profit for the year:
J K L
a. P 56,000 P 54,000 P 60,000
b. P 104,000 P 228,500 P0
c. P 96,000 P 144,000 P 280,000
d. P166,000 P 204,000 P0
___A___(25 %)
The percentage of completion based on work certified to the total sales value:
J K L J K L
a. 75.00% 66.67% Nothing c. 84.61% 79.82% Nothing
certified certified
b. 97.50% 84.44% 0 d. 76.92% 76.92% 6.67%
___A___(25 %)
What amounts should be shown in the statement of profit or for profit based on work certified to the
total sales value?
J K L
a. P 42,000 P 36,000 P0
b. P 78,000 P 152,000 P0
c. P 72,000 P 96,000 P 20,000
d. P72,000 P 96,000 P0
___A___(25 %)
What amounts should be shown in the statement of financial position as work -in-process inventory
J K L
a. P 42,000 P 36,000 P0
b. P 78,000 P 152,000 P0
c. P 72,000 P 96,000 P 20,000
d. P72,000 P 96,000 P0
___A___(25 %)
3 / 3 points
Use the following information for questions 44 to 46:
Seasons Construction is constructing an office building under contract for Cannon Café. The contract
calls for progress billings and payments of P620.000 each quarter. The total contract price is P7
440.000 and Seasons estimates total costs of P7,100.000. Seasons estimates that the building will
take 3 years to complete, and commences construction on January 2, 20x4.
At December 31, 20x4, Seasons estimates that it is 30% complete with the construction based on
costs incurred. What is the total amount of Revenue from Long-Term Contracts recognized for 20x4
and what is the balance in the Accounts Receivable account assuming Cannon Cafe has not yet made
last quarterly payment?
Revenue Accounts Revenue Accounts
Receivable Receivable
a. P2,480,000 P2,480,000 c. P2,232,000 P620,000
b. P2,130,000 P 620,000 d. P2,130,000 P2,480,000
___C___(33.33 %)
At December 31, 20x5. Seasons Construction estimates that it is 75% complete with the building
however, the estimate of total costs to be incurred has risen to P7,200.000 due to unanticipated
price increases. What is the total amount of Construction Expenses that Seasons will recognize for
the year ended December 31, 20x59
a. 5,400,000
b. 3,150,000
c. 3,195,000
d. 3,270,000
___D___(33.33 %)
At December 31, 20x5, Seasons Constructions estimates that it is 75% complete with the building:
however, the, estimate of total costs to be incurred has risen to P7,200,000 due to unanticipated
price increases. What is reported in the balance sheet at December 31, 20x5 Construction in Process
accounts, and is it a debit or a credit?
Difference between the Debit/Credit
accounts
a. P 1,690,000 Credit
b. P 620,000 Debit
c. P 440,000 Debit
d. P 620,000 Credit
___B___(33.33 %)
1 / 1 point
Seasons construction completes the remaining 25% of the building construction of December 31,
20x6 as scheduled. At that time the total costs of construction are P 7,500,000. What is the total
amount of Revenue from Long-Term Contracts and Construction Expenses that Seasons will
recognize for the year ended December 31, 20x6? (1) Revenue; (2) Expenses

(1) P7,440,000 ; (2) P7,500,000

(1) P1,860,000; (2) P1,875,000

(1) P1,860,000; (2) P2,100,000

(1) P1,875,000; (2) P1,875,000

Question 35 2 / 2 points
The following information relates to questions 48 and 49:
Cooper Construction Company had a contract starting April 20x4, to construct a P 9,000,000 building
that is expected to be completed in September 20x6, at an estimated cost of P8,250,000. At the end
of 20x4 the costs to date were P 3,795,000 and the estimated total costs to complete had not
changed. The progress billings during 2010 were P 1,800,000 and the cast collected during 20x4 was
1,200,000.
For the year ended December 31, 20x4, Cooper would recognize gross profit on the building of
a. P 316,250
b. P 3,795,000
c. P 405,000
d. P -0-
___B___(50 %)
At December 31, 20x4 Cooper would report Construction in Process in the amount of:
a. P 345,000
b. P 4,140,000
c. P 3,795,000
d. P 3,540,000
___B___ (C)
1 / 1 point
During 20x4, gates Corp. started a construction job with a total contract price of P 3,500,000. The job
was completed on December 15, 20x5. Additional data are as follows:

20x4 20x5
Actual costs incurred P 1,350,000 P 1,525,000
Estimated remaining costs 1,350,000 ----
Billed to customer 1,200,000 2,300,000
Received from customer 1,000,000 2,400,000
Under the point-in-time/cost recovery method, what amount should Gates recognize as gross profit
for 20x5?

P225,000

P312,500

P475,000

None of the above

Question 37 2 / 2 points
Use the following information for questions 51 and 52:
In 20x4, Fargo Corporation began construction work under a three-year contract. The contract price
is P2,400,000. Fargo uses the percentage of completion method for financial accounting purposes.
The income to be recognized each year is based on the proportion of costs incurred to total
estimated costs for completing the contract. The financial statement presentations relating to this
contract at December 31, 20x4, follow:

Balance Sheet:
Accounts Receivable-construction contract P 100,000
Billings…………………………………………………………………………………………….
Construction in progress ………………………………………………………………… P 300,000
Less contract billings ……………………………………………………………………… 240,000 60,000

Income Statement:
Income (before tax) on the contract recognized
In 20x4…………………………………………………………….. P 60,000

How much cash was collected in 20x4 on this contract?


a. P 100,000
b. P 20,000
c. P 140,000
d. P 240,000

___C___ (B)
What was the initial estimated total income before tax on this contract?
a. P 400,000
b. P 320,000
c. P 480,000
d. P 300,000
___C___ (D)
2 / 2 points
Use the following information for questions 53 and 54:
Eilert Construction Company had a contact starting April 20x4, to construct a P 15,000,000 building
that is expected to be completed in September 20x5, at an estimated cost of P 13,750,000. At the
end of 20x4, the costs to date were P 6,325,000 and the estimated total costs to complete had not
changed. The progress billings during 20x4 were P 3,000,000 and the cash collected during 20x4 was
P 2,000,000. Eilert uses the percentage-of-completion method.
For the year ended December 31, 20x4, Eilert would recognize gross profit on the building of
a. P -0-
b. P 575,000
c. P 527,083
d. P 675, 000

___B___ (C)
At December 31, 20x4, Eilert would report Construction in Process in the amount of
a. 6,900,000
b. P 5,900,000
c. P 6,325,000
d. P 575,000

___A___(50 %)
1 / 1 point
Belgium Co. is constructing a tunnel for P800 million. Construction began in 20x4 and is estimated to
be completed in 20x8. At December 31, 20x6, Belgium has incurred costs totaling P 356 million with
P 85 million of that incurred in 20x6, P 143 million in 20x7, and the remainder during 20x8. Belgium
believes that it completed 30% of the tunnel during 20x6, although that may change based on future
activity. Belgium Co. uses PAS 11 for its accounting and regards its cost numbers as very uncertain
(cost recovery method/zero-profit approach). What amount of revenue should Belgium Co. recognize
for the year ended December 31,20x6?

No revenue should be recognize until the contract is completed in 20x8.

P356 million

P240 million

P85 million

Question 40 1 / 1 point
Wynn, Inc. has a contract to construct a large hotel for P 12,000,000. The contract was signed on
January 2, 20x4 and it was expected that the hotel would be complete on December 31, 20x7. At the
date the contract was signed, Wynn, Inc. anticipated the costs of P 11,000,000. At the end of 20x5
the total cost estimate rose to P 11,870,000 and at the end of 20x6 the total cost estimate rose to P
12,400,000. Due to certain circumstances, Wynn, Inc. believes there are inherent hazards in the
contract beyond the normal, recurring business risks. Wynn, Inc. expects to recover all its cost under
the contract. Under these conditions what amount of loss, if any, should Wynn, Inc. recognize in each
of the following years? (1) 20x5; (2) 20x6

1) P 870,000 ; (2) P 400,000

(1) P -0- ; (2) P 400,000

(1) P 870,000; (2) P 530,000

(1) P -0- ; (2) P -0-

1 1 / 1 point
Wynn, Inc. has a contract to construct a large hotel for P 12,000,000. The contract was signed on
January 2, 20x4 and it was expected that the hotel would be complete on December 31, 20x7. At the
date the contract was signed, Wynn, Inc. anticipated the costs of P 11,000,000. At the end of 20x5
the costs incurred were P 3,490,000 and its estimate of total contract costs rose to P 11,870,000.
During 20x6, the company incurred costs of P 4,020,00 and by the end of 20x6 the total cost estimate
rose to P 12,400,000. Due to certain circumstances, Wynn, Inc. Believes that there are inherent
hazards in the contract beyond the normal, recurring business risks. Wynn, Inc. expects to recover all
its costs under the contract. Under these conditions what amount of revenue should Wynn, Inc.
recognize in each of the following years?

(1) P 3,490,0,000 ; (2) P 4,020,000

(1) P -0- ; (2) P 400,000

(1) P 3,528,222; (2) P 3,890,323

(1) P 3,380,000 ; (2) P 4,890,000

Question 42 0 / 3 points
Use the following information for questions 58 to 60:

Contract Billings Collections Estimated Costs to Costs


Contract Through Through 12/31/x4 12/31/x4 to Complete
Price 12/31/x4
1 P 3,200,000 P3,150,000 P2,600,000 P2,150,000 ---
2 3,600,000 1,500,000 1,000,000 820,000 1,800,000
3 3,300,000 1,900,000 1,800,000 2,250,000 1,200,00

Which of the following should be shown on the income statement for 20x4 related to Contract 1?
a. Gross profit, P450,000
b. Gross profit, P1,050,000
c. Gross profit, P1,000,000
d. Gross profit, P 600,000

___B___ (C)
Which of the following should be shown on the balance sheet at December 31,20x4 related to
Contract 2?
a. Contract assets, P680,000
b. Contract liabilities, P680,000
c. Contract assets, P820,000
d. Contract liabilities, P 1,500,000

___B___ (C)
Which of the following should be shown on the balance sheet at December 31, 20x4 related to
Contract 3?
a. Contract assets, P 200,000
b. Contract liabilities, P2,100,000
c. Contract assets, P 350,000
d. Contract liabilities, P 2,250,000

___C___ (A)
1 / 1 point
GR & R Enterprise entered into a construction agreement in 20x4 that called for a contact price of
P9,600,000. At the beginning of 20x5, a change order increased the initial contract price by P480,000.
In relation to the project the following data were obtained:

20x4 20x5
Costs incurred to date …………… P 4,920,000 P 8,640,000
Estimated costs to complete ….. 4,920,000 2,160,000
Billings made to date ……………… 5,280,000 8,700,000
Collections made to date ………. 4,920,000 8,7000,000

Compute the amount of construction in progress (net) – contract assets or progress billings (net) –
contract liabilities for the year 20x5: (1) Percentage of completion method/overtime; (2) Cost
Recovery Method Construction Accounting/Point-in-Time

(1) P 780,000 – liabilities; (2) P 780,000 - liabilities

(1) P 780,000 – assets; (2) P 780,000 - assets

(1) P 60,000 – liabilities; (2) P 60,000 - liabilities

(1) P 636,000 – liabilities; (2) P 636,000 - liabilities

Question 44 1 / 1 point
The Giant Construction Company started word in three job sites during the current year. Any costs
incurred are expected to be recoverable. Data relating to the three jobs are given:

Contract Price Costs incurred Estimated costs Billings on Collections on


to complete contract contract
Project 6 P 500,000 P 375,000 P 500,000 P 500,000
Project 7 700,000 100,000 P 400,000 100,000 100,000
Project 8 250,000 100,000 100,000 150,000 100,000

What would be the amount of Construction-in-Progress account to be reported on the year-end


balance sheet if the (1) overtime/percentage-of-completion method, and (2) point-in-time/cost
recovery method of construction accounting were used?

(1) P765,000; (2) P700,000

(1) P765,000; (2) P765,000

(1) P265,000; (2) P265,000

(1) P265,000; (2) P200,000

Question 45 3 / 3 points
Arizona Desert Homes constructed a new subdivision during 20x4 and 20x5 under contract with
Cactus Development Co. Relevant data are summarized below:

Contract amount P3,000,000


Cost 20x4 1,200,000
20x5 600,000
Gross profit 20x4 800,000
20x5 400,000
Contract 20x4 1,500,000
Billings 20x5 1,500,000

ADH uses the overtime/percentage-of-completion method to recognize revenue.


What would be the journal entry made in 20x4 to record revenue?
a. Accounts receivable 1,500,000
Revenue for long-term contracts 1,500,000
b. Accounts receivable 2,300,000
Gross profit 800,000
Revenue for long-term contracts 1,500,000
c. Construction in progress 800,000
Cost of construction 1,200,000
Revenue for long-term contracts 2,000,000
d. Cost of construction 1,200,000
Revenue for long-term contracts 1,200,000
___C___(33.33 %)
In its December 31, 20x4 balance sheet, ADH would report:
a. The asset, cost and profits in excess of billings, of P500,000.
b. The liability, billings in excess of cost, of P300,000.
c. The asset, contract amount in excess of billings, of P1,500,000.
d. The asset, deferred profit, of P400,000.

___A___(33.33 %)
What would be the journal entry to record revenue in 20x5?
a. Accounts receivable 1,500,000
Revenue for long-term contracts 1,500,000
b. Construction in progress 400,000
Cost of construction 600,000
Revenue for long-term contracts 1,000,000
c. Cost of construction 2,000,000
Gross Profit 1,000,000
Revenue for long-term contracts 3,000,000
d. Accounts receivable 1,500,000
Cost of construction 600,000
Gross profit 600,000
Revenue for long-term contracts 300,000
___B___(33.33 %)
1 / 1 point
As of December 31, 20x6, Cady Construction has one construction job for which the construction in
progress (CIP) account has a balance of P20,000 and the billings on construction contract account has
a balance of P14,000. Cady has another construction job for which the construction in progress
account has a balance of P3,000 and the billings on construction account has a balance of P5,000.
Indicate the amount of contract asset and/or contract liability that Cady would show in its December
31, 20x6, balance sheet?

Answer: 6,000 asset 2,000 liability (-2,000)


Question 47 3 / 3 points
Use the following information for questions 67 to 69:
Sahara Desert Homes SDH) reports under PFRS, and constructed a new subdivision during 20x4 and
20x4 under contract with Cactus Development Co. Relevant data are summarized below:

Contract amount P3,000,000


Cost 20x4 1,200,000
20x5 600,000
Gross profit 20x4 800,000
20x5 400,000
Contract 20x4 1,500,000
Billings 20x5 1,500,000
ADH uses the point-in-time/cost recovery method under PFRS to recognize revenue.
What would be the journal entry made in 20x4 to record revenue?
a. Accounts receivable 1,500,000
Revenue for long-term contracts 1,500,000
b. Accounts receivable 2,300,000
Gross profit 800,000
Revenue for long-term contracts 1,500,000
c. Construction in progress 800,000
Cost of construction 1,200,000
Revenue for long-term contracts 2,000,000
d. Cost of construction 1,200,000
Revenue for long-term contracts 1,200,000

___D___(33.33 %)
In its December 31, 20x4 balance sheet, ADH would report:
a. The asset, cost and profits in excess of billings, of P500,000.
b. The liability, billings in excess of cost, of P300,000.
c. The asset, contract amount in excess of billings, of P1,500,000.
d. The asset, deferred profit, of P400,000.

___B___(33.33 %)
What would be the journal entry to record revenue in 20x5?
a. Accounts receivable 1,500,000
Revenue for long-term contracts 1,500,000
b. Construction in progress 400,000
Cost of construction 600,000
Revenue for long-term contracts 1,000,000
c. Cost of construction 2,000,000
Gross Profit 1,000,000
Revenue for long-term contracts 3,000,000
d. Accounts receivable 1,200,000
Cost of construction 600,000
Revenue for long-term contracts 1,800,000

___D___(33.33 %)
1 / 1 point
The company signed an P800,000 contract to build an environmentally friendly access trail to
Morayta, Manila. The project was expected to take approximately 3 years. The following information
was collected for each year of the project – Year 1, Year 2, and Year 3:

Year Cost Expected Support Additional Trail feet Additional


expended additional timbers laid support constructed fee to be
during the cost to during the timbers to during the constructed
year completion year be laid year
1 P100,000 150 850 3,000 3,000 15,200
2 280,000 300 520 7,500 7,500 8,200
3 -0- 500 -0- 8,000 8,000 -0-

Compute the amount of revenue to be recognized in Year 3, assume that the company employs (1)
the effort-expended method of estimating percentage of completion, if the company measures its
progress by the number of support timbers laid in the trail, and (2) an output measure, if the
company measures its progress by the number of trail feet that have been completed:

(1) P428,864; (2) P350,800

(1) P422,640; (2) P350,800

(1) P428,864; (2) P422,640

(1) P350,800; (2) P428,864

Question 49 2 / 2 points
Use the following information for questions 71 and 72:
Chicane Builders, Inc. employs the cost-to-cost method in determining the percentage of completion
for revenue recognition. The company’s records show the following information on a recently
completed project for a contract price of P5,000,000.

20x4 20x5 20x6


Costs incurred to date P900,00 P2,550,000 P?
Gross profit (loss) 350,000 (50,000)

The estimated costs to complete the project at December 31, 20x6:


a. P850,000
b. P1,700,000
c. P2,300,000
d. P2,550,000

___B___(50 %)
The estimated costs to complete the project at December 31, 20x6:
a. P2,550,000
b. P2,300,000
c. P2,200,000
d. P2,050,000

___D___(50 %)
1 / 1 point
On October 1, 20x4, Delta signed a fixed term construction contract. Details are as follows:
• Fixed contract price of P60,000,000; due date of completion, September 30, 20x4; costs
incurred to date on the contract have been P26,000,000. These comprise materials, labor
and overheads of P12,000,000 and other costs of construction of P14,000,000 (purchased
on October 1, 20x5).
• Future materials, labor and overhead costs are estimated at P25,000,000. No further
investment in plant will be needed; before the year-end the contract was certified as 30%
complete by an independent expert. This entitled Delta to a progress payment of
P5,000,000, which was received before the year-end.
Compute the amount contract assets/liabilities using output measure – proportional cost approach?

P21,000,000 liabilities

P27,700,000 assets

P23,700,000 assets

P23,700,000 liabilities

1 1 / 1 point
In 20x4, Kalye Construction Company was contracted to build the private road network of Alaya
Subdivision for P100 million. The project was expected to be finished in 2 years, and the contract
provided for:
• A five percent mobilization fee (to be deducted from the last billing), payable within 14 days
from the contract signing. A retention provision of ten percent on all billings payable with
the final bill after the complete project is cancelled.
• Payment of progress billings within 7 days from acceptance.

Kalye, which uses the percentage-of-completion method of accounting for income, estimated a 25%
gross margin on the project. By the end of the year, Kalye had presented progress billing to Alaya
corresponding to 50% completion. Alaya accepeted all the bills presented, except one for 10% which
was accepted on January 5 of next year. With the exception of the second to the last billing for 8%
which was due on January 3 of next year, all accepted billing were settled. In 20x4, Kalye
Construction Company realized gross profit from the project the amount of:

P7,500,000

P10,000,000

P12,500,000

P25,000,000

Question 52 1 / 1 point
On September 30, 20x4, Jaja Inc. was awarded to contract to build a 1,000-room hotel for P120
million. Among others, the parties agreed the following:
• Ten percent mobilization fee (Deductible from “final billing”) payable within ten days from
the signing of the contract retention of ten percent on all billings (to be paid within the
final billing upon completion and acceptance of the project); and
• Progress billings are to be paid within 2 weeks upon acceptance.

By the end of 20x4, the company has presented one progress billing, corresponding 10% completion,
which was evaluated and accepted by the client on December 29, 20x4 for payment in January of
next year, In 20x4, assuming use of the percentage-of-completion method of accounting, Jaja Inc.
received a total fee of:

P1,200,000

P11,800,000

P12,000,000

P13,200,000

Question 53 1 / 1 point
In 20x4, AJD Construction Company was contracted to build Village Company’s private road network
for P100 million. The project was estimated to be completed in two years and the contract provided
for:
• 5% mobilization fee (to be deducted from the last billing) payable within 15 days after the
signing of the contract; 10% retention provision on all billings, and
• Payment of progress billing within 10 days from acceptance.

AJD, which uses the percentage-of-completion method of accounting, estimated a 25% gross margin
on the project. By the end of 20x4, AJD has presented progress billing corresponding to 50%
completion. All of the progress billings presented in 20x4 were accepted, except the last one for 10%
which was accepted on January 5, 20x5. With the exception of one bill which was due on January 7,
20x5, all of the billings accepted in 20x4 were settled. Payments made by Village Company in 20x4
amounted to:

P33,800,000

P38,500,000

P40,000,000

P45,000,000
1 / 1 point
YES Partnership had the following condensed financial position prior to liquidation:

Assets Liabilities and Capital

Cash P588,000 Liabilities P328,000


Noncash assets 2,880,000 Loan payable to NY 180,000
NY, Capital (25%) 776,000
RB, Capital (40%) 1,320,000
IS, Capital (35%) 864,000
Total P3,468,000 Total P3,468,000

Assuming noncash assets with a book value of P1,360,000 were sold for P1,660,000 and that all
available cash was distributed.

Which of the following statements is false for Partner NY to receive a total of P704,000 cash after
liquidation?

Partner IS will receive a total of P511,200 cash after liquidation

The proceeds from the sale of the remaining noncash assets amount to P212,000

Partner RB will receive the amount of P832,000 on the first distribution of cash

The loss on realization on the sale of the remaining noncash assets amount to P708,000

Question 2 1 / 1 point
Brilliant Inc. is constructing a skyscraper in the heart of town and has signed a fixed price two -year
contract for P21.0 million with the local authorities. It has incurred the following cost relating to the
contract by the end of first year:
• Material cost P5 million
• Labor cost P2 million
• Construction overhead P2 million
• Marketing costs P0.5 million
• Depreciation of the idle plant and equipment P0.5 million
At the end of the first year, it has estimated cost to complete the contract = P9 million
What profit or loss from the contract should Brilliant Inc. recognize at the end of the first year?
P1.5 million (9/18 x 3.0)

P1.28 million (9.5/18.5 x 2.5)

P1.05 million (10/19 x 2.0)

P1.0 million (9/18 x 2.0)

Question 3 1 / 1 point
Natural Designs sells furniture on 12 months’ interest free terms to qualifying customers. On 30 June
2019, Natural Designs sells $20 000 of furniture to T. Bailey, payable by 30 June 2020. The
appropriate interest rate for this transaction is determined to be 6% per annum. The present value of
the $20 000 to be received in one year’s time is $18 868. The journal entry to be recorded by Natural
Designs at 30 June 2019 is:

DR Receivable $20 000; CR Sales revenue $20 000.

DR Receivable $20 000; CR Sales revenue $18 868; CR Deferred interest $1,132.

DR Receivable $20 000; CR Sales revenue $18 868; CR Interest revenue $1,132.

DR Bank $20 000; CR Receivable $20 000.

Question 4 1 / 1 point
When a company has an obligation or right to repurchase an asset for an amount less than its selling
price, the transaction should be treated as a

financing transaction.

lease.

outright sale.

repurchase transaction.

put option.

Question 5 1 / 1 point
On January 1, 20x1, an entity accepted a long-term construction project to build a condominium at a
fixed contract price of 140 million. The outcome of performance obligation in connection with this
contract cannot be measured reasonably as of year-end. The following data are provided by the
accountant and project manager:
Estimated cost to complete construction project as of January 1, 20x1, P90,000,000
Actual costs incurred as of December 31, 20x1, P45,000,000
How much is entity’s gross profit for the year ended December 31, 20x1?

25,000,000

30,000,000

10,000,000

Question 6 1 / 1 point
On June 1st, Joseph & Company received a $500 deposit for 80 cases of wine. On June 10th the
customer identified specific vintages that are included in Joseph's inventory, and asked that Joseph
not ship the wine until June 20 so the customer could ready space to store the wine, so Joseph set
those wines aside for the customer, boxed and ready for shipment to the customer. On June 20th the
wine was shipped and delivered to the customer. Joseph likely would recognize revenue on

Upon consumption of the wine by the customer.

June 10th.

June 1st.

June 20th.
Question 7 3 / 3 points
Rainbow Inc has a P8 million contract started in 2018. The following information is provided for the
construction activities.

Years Actual cost to date Addl cost to complete


2018 P1,024,000 P4,096,000
2019 3,993,600 2,246,400
2020 6,473,600 0

The company uses cost to cost percentages in measuring satisfaction of performance obligations.
Revenue earned from the contract in 2018
a.
a. P1.6 m
b. P1.8 m
c. P 1.76 m
d. P 1.67 m

___A___(33.33 %)
Gross profit realized for 2019 is
a.
a. P576,000
b. P550,400
c. P1,126,400
d. P 480,000

___B___(33.33 %)
Revenue earned in 2020 is
a.
a. P 5.12 m
b. P 2.88 m
c. P3.52 m
d. P8.00 m

___B___(33.33 %)
1 / 1 point
On January 2, 20x5, QUICKBUILD ERECTORS entered into contract to construct two projects. The
following data relate to the construction activities.

Project A Project B
Contract Price 420,000 310,000
Cost incurred during 2019 240,000 280,000
Estimated cost to complete 120,000 40,000
Billed to customers during 2019 150,000 270,000
Received from customers 120,000 250,000
during 2019

What amount of gross profit should QUICKBUILD ERECTORS report in its 20x5 income statement?

₱40,000

₱31,250

None of the choices

₱30,000
Question 9 1 / 1 point
Mediocre Inc. has entered into a very profitable fixed price contract for contracting a high-rise
condominium building over a period of three years. It incurs the following costs relating to the
contract during the first year:
• Cost of material P2.5 million
• Site labor costs P2.0 million
• Agreed administrative costs as per contract to be reimbursed by the customer P1 million
• Depreciation of the plant used for the construction P0.5 million
• Marketing costs for selling apartments when they are ready P1.0 million
Total estimated cost of the project = P18 million
The percentage of completion of this contract at the year-end is

27% (= 4.5/16.5)

33 1/3% (=6.0/18.0)

39% (=7.0/18)

25% (= 4.5/18.0)

Question 10 2 / 3 points
On January 1, 2013, ACJ Partnership entered into liquidation. The partners’ capital balances on this
date were as follows: A (25%) P2,500,000; C (35%) P5,400,000; J (40%) P3,700,000. The partnership
has liabilities amounting to P4,400,000, including a loan from C P600,000. Cash on hand before the
start of liquidation is P800,000.
With the information given, answer the following independent situations:
Noncash assets amounting to P7,400,000 were sold at book value and the rest of the noncash assets
were sold at a loss of P4,200,000. How much cash will be distributed to the partners?

a. P8,000,000
b. P7,400,000
c. P4,400,000
d. P11,800,000
___A___(33.33 %)
After exhausting the noncash assets of the partnership, assuming all partners has personal assets
more than their personal liabilities. How much cash must be invested by the partners to satisfy the
claims of the outside creditors and to pay the amount due to the partner/s?
a. P3,680,000
b. P4,360,000
c. P4,480,000
d. P3,800,000
___B___ (A)
If C received P2,255,000, How much was the loss from the realization of the noncash assets?
a. P5,255,000
b. P10,525,000
c. P10,700,000
d. P9,945,000
___D___(33.33 %)
1 / 1 point
Construction Co. wins a competitive tender to construct a warehouse for a customer. They incur the
following costs to obtain the contract:
• External legal fees for due diligence: $35,000

• Travel costs to deliver proposal: $5,000

• Commission to sales employee if tender is successful: $10,000

• External consulting costs incurred to assist Construction Co preparing the tender


documentation: $10,000
• Administrative costs incurred in preparing the tender document: $15,000.

The completion of this construction project is expected to take 2 years. The contract costs that
should be capitalized is

$25,000

$10,000

$20,000

$15,000

$45,000

Question 12 1 / 1 point
Explodia.com sells fireworks over the Internet. Customers access Explodia's website and select
particular products, and Explodia refers the customer order to a fireworks manufacturer who fulfills
the order, ships to the customer, and pays Explodia a 20% commission. Which of the following is true
about Explodia?

Explodia's income statement would report gross revenue and cost of sales associated
with these transactions.
Explodia warehouses inventory.

Explodia is an agent in this transaction.

Explodia is primarily responsible for providing the product to the customer.

Question 13 1 / 1 point
Under the cost-recovery method

None of these answers are correct.

revenue, cost, and gross profit are recognized during the production cycle.

revenue, cost, and gross profit are recognized at the time the contract is completed.

revenue and cost are recognized during the production cycle, but gross profit
recognition is deferred until the contract is completed.

Question 14 1 / 1 point
CIGNAL ERECTORS began operations on January 2, 20x5. During the year, the company entered into a
contract with TEAM Company to construct a manufacturing facility. At that time CIGNAL estimated
that it would take five years to complete the facility at a cost of P3,937,500. The total contract price
for the construction of the facility is P5,468,750. During the year, the company incurred P962,50 0 in
construction costs related to the construction project. The estimated cost to complete the contract is
P3,412,500. TEAM was billed and paid 30% of the contract price subject to a 10% retention payable
together with the last billing after a third party inspected the manufacturing facility.
Using the percentage of completion method, how is the contract presented in the Balance Sheet?

P437,500 (Contract asset)

P273,437 (Contract asset)

P437,500 (Contract liability)

P273,437 (contract liability)

Question 15 2 / 2 points
Company A enters into a contract to sell 14,000kg of apples to a customer for $14,000 ($1.00/kg).

The apples are packaged in 1kg plastic bags (with each bag being distinct) and are transferred to the
customer over a seven-week period.
Company A recognizes revenue after each delivery (based on the number of 1kg bags delivered).

After Company A has transferred 8,000 1kg bags of apples to the customer, the contract is modified
to require the delivery of an additional 3,000 1kg bags of the same variety of apples to the customer
(i.e. there will now be a total of 17,000 1kg bags of apples delivered to the customer and 9,000 of
those bags have not yet been delivered).

The price for each additional 1kg bag will be $0.95, which reflects the standalone selling price of the
products at the time of the contract modification.

How should Company A account for the sale of the next 2,000 1kg bags of apples to its customer?

a. DR Cash $2,000 (2,000 bags at $1/bag); CR Revenue $2,000


b. DR Cash $1,967 (2,000 bags at $0.9833/bag); CR Revenue $1,967
c. DR Cash $1,982 (2,000 bags at $0.9912/bag); CR Revenue $1,982
d. DR Cash $1,912 (10,000 bags at $0.9912/bag less 8,000 bags at $1/bag); CR Revenue
$1,912
___A___(50 %)
The facts are the same except that the price for each additional 1kg bag will be $0.80, which does
not reflect the standalone selling price of the apples at the time of the contract modification. How
should Company A account for the sale of the next 2,000 1kg bags of apples to its customer?

a. DR Cash $2,000 (2,000 bags at $1/bag); CR Revenue $2,000


b. DR Cash $2,000 (2,000 bags at $1/bag received from the customer); CR Revenue $1,867
(2,000 bags at $0.9333/bag) ; CR Revenue received in advance $133 (balancing
figure)
c. DR Cash $1,867 (2,000 bags at $0.9333/bag received from the customer); CR Revenue
$1,867
d. DR Cash $2,000 (2,000 bags at $1/bag received from the customer); CR Revenue $1,333
(10,000 bags at $0.9333/bag less 8,000 bags at $1/bag)) ; CR Revenue received in
advance $667
___B___(50 %)
2 / 2 points
On April 1st, Bob the Builder entered into a contract of one-month duration to build a barn for
Nolan. Bob is guaranteed to receive a base fee of $5,000 for his services in addition to a bonus
depending on when the project is completed. Nolan created incentives for Bob to finish the barn as
soon as he can without jeopardizing the structural integrity of the barn. Nolan offered to pay an
additional 30% of the base fee if the project finished 2 weeks early and 10% if the project finished a
week early. The probability of finishing 2 weeks early is 30% and the probability of finishing a week
early is 60%.

What is the expected transaction price with variable consideration estimated as the expected value?
A) $4,750
B) $5,000
C) $5,500
D) $5,750
___D___(50 %)
What is the expected transaction price with variable consideration estimated as the most likely
amount?
A) $4,750
B) $5,000
C) $5,500
D) $5,750

___C___(50 %)
1 / 1 point
In November 20X2, an entity contracts with a customer to remodel a 3-storey building and install
new elevators for total consideration of CU5 million. The promised refurbishment service, including
the installation of elevators, is a single performance obligation satisfied over time. Total expected
costs are CU4 million, including CU1.5 million for the elevators.
A summary of the transaction price and expected costs is as follows
Transaction price 5.0m
Expected costs:
Elevators 1.5m
Other costs 2.5m
Total expected costs 4.0m
The customer obtains control of the elevators when they are delivered to the site in December 20X2,
although the elevators will not be installed until June 20X3. The entity is not involved in designing or
manufacturing the elevators.
The entity uses an input method based on costs incurred to measure its progress towards complete
satisfaction of the performance obligation.
As of 31 December 20X2, the total costs incurred (excluding elevators) is CU500,000.
What is the revenue recognized from this contract?

Some other amount

None

2.5M

2.2M
Question 18 1 / 1 point
Which of the following is not true about accounting for long-term construction contracts?

When a customer is billed for payment due, billings on contracts in progress is credited
at the same time accounts receivable is debited.

Billings on contracts in progress is a contra account to accounts receivable.

Long-term construction contracts could show a contract asset or contract liability,


depending on the relation between construction in progress and billings.

Gross profit is debited to construction in progress.

Question 19 1 / 1 point
HM, CM and DM of The M3 Partnership has the following account balances before liquidation:

Cash P420,000 Liabilities P524,000


Noncash assets 3,880,000 Loan from DM 100,000
Loan to CM 192,000 HM, Capital (25%) 1,120,000
Receivable from HM 44,000 CM, Capital (15%) 1,624,000
Expenses 2,556,000 DM, Capital (60%) 2,256,000
Revenues 1,468,000

During June, some noncash assets were sold that resulted to a gain of 72,000. Liquidation expenses
of P124,000 were paid and additional expenses amounting to P96,000 were expected to be incurred
through the following months of liquidating the partnership. Liabilities to outsiders amounting to
P316,000 were paid.
For CM to receive P874,000 on the first distribution of cash, which of the following statements is
correct?

The amount of cash withheld considered in the computation of maximum possible loss
amount to P304,000

The total maximum possible loss for the month of June amount to P2,704,000

The total amount of cash paid to partners in June amount to P1,144,000

The proceeds from the sale of the noncash assets sold in June amount to P1,396,000

Question 20 1 / 1 point
Which of the following is not true about revenue recognition with respect to long-term construction
contracts?

Long-term construction contracts often satisfy the criteria for recognizing revenue over
time.

Long-term construction contracts typically include multiple performance obligations


because of all the different types of goods or services included for each project.

Long-term construction contracts require accounting for construction in progress as


well as billings to customers.

Long-term construction contracts often are viewed as having a single performance


obligation, because goods or services fail the "separately identifiable" criterion.

Question 21 1 / 1 point
Revenue likely is recognized over time for all the following arrangements except for

Bank earning interest on a long term loan.

Construction of a building.

Providing a two-year gym membership.

Manufacturing generally stocked items ordered by a favored customer.

Question 22 1 / 1 point
Verma, Inc. sells office furniture. In 2021, it sold 200 desks for $500 each. For each desk sold, Verma
distributed a 50% discount coupon for purchase of an office chair within one month. Based on
historical experience, Verma expects that approximately 20% of the coupons will be utilized. The
chairs purchased with the coupons are priced at $150 and normally discounted 10%. What would be
the stand-alone sales price used by Verma for the coupon when allocating the $500 transaction price
to performance obligations?

$75

$0

$15

$12

Question 23 1 / 1 point
The accounting principle applied by IFRS15 when determining whether or not revenue should be
recognized in respect of a repurchase agreement is

Relevance

Prudence

Verifiability

Substance over form

Question 24 1 / 1 point
Which of the following is considered a performance obligation?

Up-front registration fees for a gym membership

Quality-assurance warranties on electronic products

A processing fee to obtain a bank loan

Extended warranties on electronic products

Question 25 1 / 1 point
UV, PX, and TG are partners who share profits and losses as follows: UV 45%, PX 15% and TG 40%.
The Statement of Financial Position of VXG Partnership as of December 31, 20x2 is given below:
VXG Partnership
Statement of Financial Position
As of December 31, 20x2

Assets Liabilities and Equity


Cash P268,000 Liabilities P532,000
Noncash Assets 1,940,000 Loan from PX 44,000
UV, Capital 694,000
PX, Capital 354,000
TG, Capital 584,000
Total assets PP2,208,000 Total Liabilities & Equity P2,208,000

On January 1, 20x3, the partners decided to liquidate. For the month of January, some assets were
sold at a gain of P56,000. Payment to partner PX from the initial sale of assets was P181,400. Cash
withheld for possible liquidation expenses and unrecognized liabilities amounted to P146,800.
Which of the following statements is false?

The share of TG in the maximum possible loss is P600,000


The total amount of cash paid and distributed for the month of January is P764,000

Payment to partner UV from the initial sale of assets was P44,200

The book/carrying value of the noncash assets sold in January amount to P642,800

Wilson Links Products sells a product that involves two separate performance obligations: the
SwingRight golf club weight and the SwingCoach teaching software. SwingRight has a stand-alone selling
price of $150. Wilson sells both the SwingRight and the SwingCoach as a package deal for $200. The
SwingCoach software is not sold separately. Wilson is aware that other vendors charge $100 for similar
software, and Wilson's prices are generally 10% lower than what is charged by those vendors. Wilson
estimates that it incurs approximately $65 of cost per copy of the software, and usually charges 50%
above cost on similar products.

Estimate the stand-alone selling price of the software using the adjusted market assessment approach.
A) $50
B) $80
C) $90
D) $97.50
___C___(33.33 %)
Estimate the stand-alone selling price of the software using the expected cost plus margin approach.
A) $50
B) $80
C) $90
D) $97.50
___D___(33.33 %)
Estimate the stand-alone selling price of the software using the residual approach.
A) $50
B) $80
C) $90
D) $97.50
___A___(33.33 %)
1 / 1 point
Boomerang Computer Company sells computers with an unconditional right to return the computer if
the customer is not satisfied. Boomerang has a long history selling these computers under this returns
policy and can provide precise estimates of the amount of returns associated with each sale. Boomerang
most likely should recognize revenue:

When Boomerang delivers a computer to a customer, in an amount that is reduced by


the expected returns.
When Boomerang receives cash from the customer.

When Boomerang delivers a computer to a customer, ignoring potential returns.

When a customer returns a computer.

Question 28 1 / 1 point
Lazy Builders Inc. has incurred the following contract costs in the first year on a two-year fixed price
contract for 4.0 million to construct a bridge
• Material cost P2 million
• Other contract costs (including site labor costs) P1 million
• Cost to complete P2 million
How much profit or loss should Lazy Inc. recognize in the first year of the three-year construction
contract?

Loss of P1.0 million (expensed immediately)

Since 60% is the percentage of completion, recognize 60% of loss (i.e., P0.6 million)

No profit or loss in the year and deferring it to second year

Loss of P0.5 million prorated over two years

Question 29 1 / 1 point
Which of the following are included in the scope of IFRS 15 Revenue from Contracts with Customers?
I. Insurance contracts.
II. Subscriptions.
III. Accounting for investments in associates.
IV. Accounting for share of joint venture revenue.

I and IV only.

II and IV only.

II only.

II and III only

Question 30 4.001 / 6 points


AA, BB, and CC are partners sharing profits and loss in the ratio of 4:3:3, respectively. On January 1,
2012, they decided to liquidate the partnership and the balance sheet were prepared as follows:
Assets Liabilities and Capital
Cash P 2,000 Liabilities P 6,000
Other Assets 46,000 BB, loan 5,000
CC, loan 2,500
AA, capital 14,450
BB, capital 12,550
CC, capital 7,500
Total Assets P48,000 Total liab & capital P48,000

The following transactions as a result of liquidation were as follows:

Book Proceeds Payment of Payment Cash


value of from sale liquidation to withheld
assets sold expense creditors
January P12,000 P10,500 500 P6,000 P2,000
February 7,000 6,000 750 1,000
March 15,000 10,000 1,000 2,500
April 12,000 5,000 5,000 -0-

AA, BB, and CC are partners sharing profits and loss in the ratio of 4:3:3, respectively. On January 1,
2012, they decided to liquidate the partnership and the balance sheet were prepared as follows:

Assets Liabilities and Capital


Cash P 2,000 Liabilities P 6,000
Other Assets 46,000 BB, loan 5,000
CC, loan 2,500
AA, capital 14,450
BB, capital 12,550
CC, capital 7,500
Total Assets P48,000 Total liab & capital P48,000

The following transactions as a result of liquidation were as follows:

Book Proceeds Payment of Payment Cash


value of from sale liquidation to withheld
assets sold expense creditors
January P12,000 P10,500 500 P6,000 P2,000
February 7,000 6,000 750 1,000
March 15,000 10,000 1,000 2,500
April 12,000 5,000 5,000 -0-
The amount to be received by Partner BB for the month of January?
a.
a. P 0
b. 1,800
c. P4,000
d. 5,000
___C___(16.67 %)
The amount to be received by Partner CC for the month of February?
a.
a. P 0
b. 475
c. P 1,875
d. 2,500
___A___ (B)
The amount to be received by Partner AA for the month of March?
a.
a. P 0
b. 3,000
c. P 8,800
d. 14,450
___B___(16.67 %)
The amount to be received by Partner BB for the month of April?
a.
a. P 750
b. 3,000
c. P 4,000
d. 5,000
___A___(16.67 %)
The partner most vulnerable to partnership losses on liquidation is:
a.
a. AA
b. BB
c. CC
d. None
___B___ (C)
The second payment to any partner (s) under a program of priorities shall be made thus:
a.
a. To BB, P6,712.50
b. To AA, P1,116.70
c. To CC, P6,712.50
d. To AA, P1,116.70 and BB, P837.50
___D___(16.67 %)
1 / 1 point
On 1 January 2019, a customer engages Construction Co to provide construction services to build a
house for $500,000 (estimated cost $300,000). On 1 January 2020, Construction Co and the customer
agree to modify the contract to rearrange the layout of the kitchen and bathroom for an additional
$100,000 (estimated cost $50,000).
Construction Co has been recognizing revenue on a stage of completion basis and at 1 January 2020,
50% of the original contract value has been completed.
How should this contract modification be accounted for?

The contract modification is accounted for as a separate contract. Total revenue


recognized in 2019 is $300,000 ($500,000 x 50% + $100,000 x 50%).

The contract modification is accounted for as part of the existing contract which
requires a catch-p adjustment. Total revenue recognized in 2019 is $257,153 ($257,143
x 42.86%).

The contract modification is accounted for as termination of old contract and creation
of a new contract. Total revenue recognized in 2019 is $257,153 ($257,143 x 42.86%).

The contract modification is accounted for as a separate contract. Total revenue


recognized in 2019 is $250,000 ($500,000 x 50%)

Question 32 1 / 1 point
On January 1, 2031, Megaland Inc. entered into a construction contract with an owner to build an oil
refinery. The contract has the following characteristics. The oil refinery is highly customized to the
owner’s specifications and changed to these specification by the owner are expected over the contract
term. The oil refinery does not have an alternative use to the contractor. Non-refundable, interim
progress payments are required as a mechanism to finance the contract. The owner can cancel the
contract at any time (with a termination penalty); any work in process is the property of the owner. As a
result, another entity would not need to reperform the tasks performed to date. Physical possession and
title do not pass until completion of the contract. The contractor determines that the contract has a
single performance obligation to build the refinery. The preponderance of evidence suggests that the
contractor’s performance creates an asset that the customer controls and control is being transferred
over time Megaland Inc. concludes that input method (cost to cost method) instead of output method is
a more reasonable method for measuring the progress toward satisfying the performance obligation.
The contract duration is 3 years with total estimated contract revenue of P300M The total estimated
contract cost as of December 31, 2031 is P200M. The cost incurred during year 2031 is P120M including
P20M related to contractor-caused inefficiencies which do not represent/depict the transfer of goods or
services to the customer. As of December 2032, the total estimated contract cost becomes P250M due
to increase in cost of raw materials. The cost incurred during year 2032 is P105M including P5M related
to contractor-caused inefficiencies which do not represent/depict the transfer of goods or services to
the customer.
Under IFRS 15, what is the net income/(net loss) to be reported by Megaland Inc. for the years ended
December 31, 2031 and 2032, respectively?

40M and (5M)


50M and (10M)

60M and (15M)

30M and (15M)

LICENSES AND ROYALTIES


1 / 1 point
Jennifer Talosig-Tan Enterprises licenses customer-relationship software to Jenny Company. In
addition to providing the software itself, Jennifer Talosig-Tan promises to provide consulting services
by extensively customizing the software to Jenny's information environment, for a total
consideration of P600,000. How many performance obligations the implied contract when a
customer registers for the services?

3
Question 2 1 / 1 point
AA Computers licenses customer-relationship software to ABS Company. In addition to providing the
software itself, AA Computers promises to provide consulting services by extensively customizing the
software to ABS's information technology environment, for a total consideration of P3,456,000. In
this case, AA Computers is providing a significant service by integrating the goods and services (the
license and the consulting service) into one combined item for which ABS has contracted. In addit ion,
the software is significantly customized by AA Computers in accordance with specifications
negotiated by ABS. How many performance obligations exist in the contract?

3
Question 3 2 / 2 points
Use the following information for questions 3 and 4:
An author, Anton D., licenses the images and names of the characters of his series of popular
children's books to a theme park for 10 years. The series is on-going and the characters evolve over
time as they age in the stories. New characters are also introduced in successive books. The theme
park is required to use the latest images of the characters. In exchange for the license Anton D.
receives P10 million per year for 10 years.
When should revenue be recognized
1.
a. Over time
b. Point in time
c. No revenue recognized
d. No performance obligation
___A___(50 %)
How many performance obligations exist in this contract for license?
1.
a. 0
b. 1
c. 2
d. 3

___B___(50 %)
1 / 1 point
Portia Glecildo Nacinopa Pharmaceuticals entered into a licensing agreement with PortGle Lab for a
new drug under development. Portia Glecilda Nacinopa will receive P6,750,000 if the new drug
receives Food and Drug Administration approval. Based on prior approval Portia Glecilda Nacinopa
determines that it is 85% likely that the drug will gain approval. The transaction price of this
arrangement should be:

P6,750,000

P5,737,500

P1,012,500

P0 until approval is received

Question 5 3 / 3 points
Use the following Information for questions 6 to 8:

Rhea Untalan Mansibang Associates sells two licenses to Yvette Tingin-Atienza Company on
September 1, 20x6. First, in exchange for P100,000, Rhea Untalan Mansibang Associates provides
Yvette Tingin-Atienza with a copy of its proprietary investment management software, who Rhea
does not anticipate updating and which Yyette Tingin-Atienza can use permanently. Second, in
exchange for P90,000, Rhea Untalan Mansibang Associates provides Yvette Tirane-Atienza’s with a
three-year right to market Yvette Tingin-Atienza's financial advisory services under the name of Rhea
Associates, which Rhea Untalan Mansibang Associates advertises on an ongoing basis.

The software license is


1.
a. Right of use
b. Right to access
c. Either right to use or access
d. Neither
___A___(33.33 %)
The license to use the name, Rhea Associates is:
5.
a. Right of use
b. Right to access
c. Either right to use or access
d. Neither
___B___ (A)
How much revenue will Rhea recognize in 20x6 under this arrangement?
a.
a. P10,000
b. P90,000
c. P100,000
d. P110,000
___D___(33.33 %)

Hide Feedback

Item #2 B Right to access. Correction to Dayag’s answer key.

1 / 1 point
Amabella Caceres-Saker Engineering licensed software to oil-drilling firms for 5 years. In addition to
providing the software, the company also provides consulting services and support to ensure smooth
operation of the software. The total transaction price is P350,000. Based on standalone values, the
company estimates the consulting services and support have a value of P100,000 and the software
license has a value of P250,000. Assuming the performance obligations are not interdependent, the
journal entry to record the transaction includes

a credit to Sales Revenue for P250,000 and a credit to Unearned Service Revenue
P100,000.
a credit to Service Revenue of P100,000.

a credit to Unearned Service Revenue of P100,000.

a credit to Sales Revenue of P350,000.


Question 7 1 / 3 points
Use the following information for questions 10 to 12: Variable Consideration

Billy Biotech enters into a licensing agreement with Paul Pharmaceutical for a drug under
development. Billy will receive a payment of P20,000,000 if the drug receives a regulatory. Based on
prior experience in the drug-approval process, Billy determines it is 90% likely the drug will gain
approval and a 10% chance of denial. Assuming that regulatory approval was granted on December
20, 20x5, and that Billy received the payment from Paul on January 15, 20x6.

Determine the transaction price of the arrangement for Billy Biotech:


1.
a. Nil
b. P18,000,000
c. P20,000,000
d. No transaction at all
___C___(33.33 %)
On December 20, 20x5, license revenue amounted to C.
9.
a. Nil
b. P18,000,000
c. P20,000,000
d. No transaction at all
___A___(33.33 %)
On January 15, 20x6, license revenue amounted to
8.
a. Nil
b. P10,000,000
c. P18,000,000
d. P20,000,000
___B___(33.33 %)

View Feedback
2 / 2 points
The ReSA Singing Group League (RSGL) licenses its trademark to ReSA Logo. Under the license
arrangement, ReSA Logo pays the RSGL a P1,000,000 initial license fee plus a bonus when annual
sales of ReSA Logo merchandise reach a threshold. The license agreement is for 4 years.

How much of the P1,000,000 initial license fee should the RSGL recognize as revenue in the first year
of the contract?
13.
a. Zero
b. P250,000
c. P1,000,000
d. Cannot tell from information given
___B___(50 %)
Assume that the RSGL anticipates that, in addition to receiving the P1,000,000 million license fee, it
will receive a bonus of P2,000,000 in year 1 of the contract and a bonus of P3,000,000 in years 2 -4 of
the contract based on ReSA Logo's sales. Also assume that the RSGL is convinced that it is probable
there will not be a significant reversal of any revenue recognized with respect to the bonus in
subsequent periods. At the inception of the contract, what is the amount of transaction price that
the RSGL would estimate with respect to this license arrangement?
a. Zero
b. P1,000,000
c. P3,000,000
d. P12,000,000
___B___(50 %)
2 / 2 points
Items 15 and 16 are based on the following Information:

ReSA FoodGroup operates several restaurants around the world through franchise agreements.
• On January 1. 204. ReSA Food Group grants a franchisee the exclusive right to cooperate a
restaurant using the ReSA Food Group brand in Manila for three years, and a license to
operate another branded restaurant in Makati for three years.
• However, because of an existing arrangement with another franchisee, the right in Makati
does not begin Until January 1, 20x5. The license fee is equal to P 15,000,000 for each of
the two licenses.
How many performance obligations exist in this contract for franchise license?
15.
a. 0
b. 1
c. 2
d. 3
___C___(50 %)
The license fee allocated to the right to operate a restaurant in recognizing as revenue:
a. Over time
b. Point in time
c. No revenue recognized
d. No performance obligation
___A___(50 %)
2 / 2 points
One the following information for questions 17 and 18:

Darlene Tolentino Ysmael a franchisor, grants a five-year franchise to an overseas company exchange
for an up-front payment of P1,000,000, to accelerate its global expansion. Darlene Tolentino Ysmael
specializes in manufacturing makeup kit which is normally sold for P1000. As part of the franchise
arrangement, it agrees to sell this product to the franchisee for P700 throughout the franchise period
- a 30% discount on usual market prices to third parties. A stage, no other services will be provided
by the franchisor.

How many performance obligations exist in this contract for franchise?


17.
a. 0
b. 1
c. 2
d. 3
___C___ (A)
The initial franchise fee of P1,000,000 is recognized as revenue:
a. Over time
b. Point in time
c. No revenue recognized
d. No performance obligation
___A___ (B)

View Feedback
4 / 4 points
Use the following information for questions 19 to 22:

Atianzar and Cabalde Computers is a franchisor and provides a range of computing services
hardware/software installation, repairs, dota backup, device syncing, and network solutions on
popular Samsung, Dell, Acer and other PC devices. Each franchise agreement gives a franchisee the
right to open a Atianzar and Cabalde outlet and sell Atianzar and Cabalde products and services in
the area for five (5) years. Under the contract, Atianzar and Cabaldes also provides the franchisee
with a number of services to support and enhance the franchise brand, including:
• advising and consulting on the operations of the store;
• communicating new hardware and software developments and service techniques;
• providing business and training manuals; and
• advertising programs and training.

As on almost entirely service operation (all materials and other supplies are purchased as needed by
customers), Atianzar and Cabalde provides few upfront services to franchisees. Instead, the
franchisee recruits service technicians, who are given Atianzar and Cabaldes’ traning materials
(online manuals and tutorials), which are updated for technology changes on a monthly basis at a
minimum.

Atianzar and Cabalde enters into a franchise agreement on December 15, 20x7, giving a franchisee
the rights to operate a Atianzar and Cabalde franchise in Cagayan Valley for five (5) years. Atianzar
and Cabalde charges an initial franchise fee of P975,000 for the right to operate as a franchisee,
payable upon signing the contract, Atianzar and Cabalde also receive ongoing royalty payments of
7% of the franchisee's annual sales (payable each January 15 of the following year). The franchise
company records P16,575,000 of revenue for the year ended December 31. 20x8.

How many performance obligations exist in this contract for franchise?


19.
a. None
b. 1
c. 2
d. 3
___B___(25 %)
The franchisee revenue should be recognized:
a. No transaction
b. No revenue
c. Point in Time
d. Over Time
___D___(25 %)
The franchisee revenue on December 31, 20x8 should be:
a. None
b. P195,000
c. P975,000
d. P1,160,250
___A___(25 %)
The franchisee revenue on December 31, 20x8 should be:
a. P195,000
b. P975,000
c. P1,160,250
d. P1,355,250
___D___(25 %)
1 / 1 point
Fred Esquillo, Inc charges an initial franchise fee of P90,000 broken down as follows:

Rights to trade name, market area, and proprietary know-how . . . . . . . . . P40,000


Training services . . . . . . . . . . . . . . . . . . . . . . . . . . . . . . . . . . . . . . . . . . . . . . . . 11,500
Equipment (cost of P10,800) . . . . . . . . . . . . . . . . . . . . . . . . . . . . . . . . . . . . . . 38,500
Total initial franchise fee . . . . . . . . . . . . . . . . . . . . . . . . . . . . . . . . . . . . . . . . . P90,000
Upon signing of the agreement, a payment of P40,000 is due. Thereafter, two annual payments of
P30,000 are required. The credit rating of the franchisee is such that it would have to pay interest of
8% to borrow money. The franchise agreement is signed on August 1. 20x7. Assuming that no future
services are required by the franchisor once the franchise begins operations, the entry on November
1, 20x7 would include

A credit to Unearned Franchise Revenue for P40,000.

A debit to Service Revenue for P11,500.

A debit to Sales Revenue for P38,500.

A debit to Unearned Franchise Revenue for P41,000.


Question 13 1 / 1 point
Liezl Sangalang Malalaban, Inc. charges an initial franchise fee of P90,000 broken down as follows.

Rights to trade name, market area, and proprietary know-how . . . . . . . . . P40,000


Training services . . . . . . . . . . . . . . . . . . . . . . . . . . . . . . . . . . . . . . . . . . . . . . . . 11,500
Equipment (cost of P10,800) . . . . . . . . . . . . . . . . . . . . . . . . . . . . . . . . . . . . . . 38,500
Total initial franchise fee . . . . . . . . . . . . . . . . . . . . . . . . . . . . . . . . . . . . . . . . . P90,000
Upon signing of the agreement, a payment of P40,000 is due. Thereafter, two annual payments of
P30,000 are required. The credit rating of the franchisee is such that it would have to pay interest of
8% to borrow money. The franchise agreement is signed on August 1, 20x8, and the franchise
commences operation on November 1, 20x8. Assume that the total training fees includes training
services for the period leading up to the franchise opening (P5,500 value) and for 3 months following
opening. The journal entry on August 1, 20x8 would include:

A credit to Unearned Service Revenue for P11,500

A credit to Unearned Service Revenue for P6,000

A debit to Sales Revenue for P38,500

A debit to Unearned Franchise Revenue for P40,000


Question 14 1 / 1 point
On January 1, 20x4 Ehtelmea Dulva Dairy Treats, Inc. entered into a franchise agreement with a
company allowing the company to do business under Ethelmea Dulva Dairy Treats’s name. Ethelmea
Dulva Dairy Treats had performed all required services by January 1, 20x4, and the franchisee paid
the initial franchisee fee of P560,000 in full on that date. The franchise agreement specifies that the
franchisee must pay a continuing fee of P48,000 annually, of which 20% must be spent on advertising
by Ethelmea Dulva Dairy Treats. What entry should Ethelma Dulva Dairy Treats make on January 1,
Initial franchise fee and the continuing franchise fee for 20x4?

DR Cash 608,000
CR Franchise Fee Revenue 560,000
CR Unearned Franchise revenue 48,000
DR Cash 608,000
CR Unearned franchise revenue 608,000
DR Cash 608,000
CR Franchise revenue 560,000
CR Unearned franchise revenue 38,400
CR Revenue from Continuing Franchise Fees 9,600
DR Prepaid advertising 9,600
DR Notes receivable 608,000
CR Franchise revenue 560,000
CR Revenue from Continuing Franchise Fees 48,000
CR Unearned Franchise Fees 9,600
Question 15 1 / 1 point
Mean Dones and Jeremie Jane Roldan Inc. charges an initial franchise fee of P920,00, with P200,000
paid when the agreement is signed and the balance in five annual payments. The present value of
the future payments, discounted at 10%, is P545,872. The franchise has the option to purchase
P120.000 of equipment for P96,000, franchise operations already stared. The amount of revenue
from franchise fees is:

P200.000
P721.872

P745,872

P920,000
Question 16 2 / 2 points
Use the following information for questions 27 and 28:

KimDrei Group of Companies charges an initial fee of P1,600,000 for a franchise, with P320,000 paid
when the agreement is signed and the balance in four annual payments The present value of the
annual payments, discounted at 10%, is P1,014,000. The franchise has the right to purchase P60,000
of kitchen equipment and supplies for P50,000. An additional part of the initial fee is for advertising
to be provided by Pasta Inn during the next five years. The value of the advertising is P1,000 a
month.

When KimDrei prepares its financial statements, the franchise fee revenue to be reported is:
25.
a. P1,264,000
b. P1,324,000
c. P1,334,000
d. P1,600,000
___A___(50 %)
When KimDrei prepares its financial statements, the unearned franchise fee revenue to be reported
is:
a. Zero
b. P10,000
c. P60,000
d. P70,000
___D___(50 %)
1 / 1 point
Ronella Ocampo sells hairstyling franchises. Ronella Ocampo receives P50,000 from a new franchisee
for providing initial training, equipment and furnishings that have a stand-alone selling price of
P50,000. Ronella Ocampo also receives P30,000 per year for use of the Ronella Ocampo name and
for ongoing consulting services (starting on the date the franchise is purchased). Carlos became a
Ronella Ocampo franchisee on July 1, 20x6, and on August 1, 20x6, had completed training and was
open for business. How much revenue 20x6 will Ronella Ocampo recognize for its arrangement with
Carlos?

Zero

P10,000

P65,000

P70,000
Question 18 3 / 3 points
Use the following information for questions 30 to 32:

Joey Muffler sells franchise arrangements throughout Luzon and Visayas. Under a franchise
agreement, Joey receives P600,000 in exchange for satisfying the following separate performance
obligations:
17.
a.
▪ franchisees have a five-year right to operate as a Joey Muffler retail
establishment in on exclusive sales territory
▪ franchisees receive initial training and certification as a Muffler Mechanic,
and
▪ franchisees receive a Joey Muffler building and necessary equipment
The stand-alone selling price of the initial training and certification is P15,000, and P450,000 for the
building and equipment. Joey estimates the stand-alone selling price of the five-year right to operate
as a Joey Muffler establishment using the residual approach. Joey received P75,000 on July 1, 20x6.
from Althea and accepted a note receivable for the rest of the franchise price. Joey will construct and
equip Altheas' building and train and certify Althea by September 1, and Althea’s five-year right to
operate as a Joey Muffler establishment will commence on September 1 as well.

What amount would Joey calculate as the stand-alone selling price of the five year right to operate as
a Joey Muffler retail establishment?
a. P135,000
b. P250,000
c. P585,000
d. P600,000
___A___(33.33 %)
What journal entry would Joey record on July 1, 20x6, to reflect the sale of a franchise to Althea?
a. DR Cash 600,000
CR Unearned franchise revenue 600,000
b. DR Cash 75,000
DR Notes receivable 525,000
CR Unearned franchise revenue 600,000
c. DR Cash 75,000
DR Notes receivable 525,000
CR Franchise revenue 75,000
CR Unearned franchise revenue 525,000
d. DR Cash 75,000
DR Notes receivable 525,000
CR Franchise revenue 600,000
___B___(33.33 %)
How much revenue would Joey recognize in the year ended December 31, 20x6, with respect to its
franchise arrangement with Althea? (Ignore any interest on the notes receivable.)
a. P 9,000
b. P450,000
c. P465,000
d. P474,000
___D___(33.33 %)
1 / 1 point
On April 1, 20x4 Joy Grigsby, Inc. entered into a franchise agreement with a local business man. The
franchisee paid P240,000 and gave a P160,000, 8%, 3-year note payable with interest due annually
on March 31. Joy Grigsby recorded the P400,000 initial franchise fee as revenue on April 1, 20x4. On
December 30, 20x4, the franchisee decided not to open an outlet under Joy Grigsby's name. Joy
Grigsby canceled the franchisee's note and refunded P128,000, less accrued interest on the note, of
the P240,000 paid on April 1. What entry should Joy Grigsby make on December 30, 20x4?

DR Loss on Repossessed Franchise 128,000


CR Cash 128,000
DR Loss on Repossessed Franchise 118,400
CR Cash 118,400
DR Loss on Repossessed Franchise 278,400
CR Cash 118,400
CR Notes Receivable 160,000
DR Revenue from Franchise Fees 400,000
CR Interest Income 9,600
CR Cash 118,400
CR Notes receivable 160,000
CR Revenue from Repossessed Franchise 112,000
Question 20 1 / 1 point
Ana Cielito Joaquin, Inc, charges an initial franchise fee of P115,000, with P25,000 paid when the
agreement was signed and the balance in five annual payments. The present value of the future
payments, discounted at 10% is P68,234. The franchise outlet already commence operations and the
franchisee has the option to purchase P15,000 of equipment for P12,000. The amount of revenue
from franchise fees is:

P25,000

P 93,234

P90,234

P115,000

Hide Feedback

Choices interchanged. B and C will be considered.

Comita Nocom and Ellen Riofrio Pasta Inn charges an initial fee of P800,000 for a franchise, with
P160,000 paid when the agreement is signed on January 1, 20x5 and the balance in four annual
payments starting December 31, 20x5. The present value of the annual payments, discounted at 10%
is P507,200. The franchisee has the right to purchase P60,000 of kitchen equipment and supplies for
P50,000. An additional part of the initial fee is for advertising to be provided by Comita Nocom and
Ellen Riofrio Pasta Inn during the next five years. The value of the advertising is P1,000 a month.
Franchise commences operations on January 20, 20x5.

The amount of revenue from franchise fee on 20x5.


35.
a. P800,000
b. P667,200
c. P657,200
d. P597,200
___D___(33.33 %)
The amount of unearned franchise fee on December 31, 20x5:
a. P10,000
b. P60,000
c. P70,000
d. P132,800
___C___(33.33 %)
What entry should Carmita Nocom and Ellen Riofrio Pasta Inn make for the franchise fees on January
1, 20x5?

a. DR Cash 160,000
DR Notes receivable 507,200
CR Unearned franchise revenue 667,200

b. DR Cash 160,000
DR Notes receivable 507,200
CR Uneamed franchise revenue 700,000

c. DR Cash 160,000
DR Notes receivable 507,200
CR Franchise revenue 597,200
CR Unearned franchise revenue 70,000

d. DR Cash 160,000
DR Notes receivable 507,200
CR Franchise revenue 597,200
CR Unearned interest income/Discount on N/R 132,800
CR Unearned franchise revenue 70,000
___D___(33.33 %)
3 / 3 points
Use the following information for questions 38 to 40:

Doriedel Salvador and Lia Mendoza-Lagcao Food, Inc., a new fast food chain, sells exclusive
franchises for P25,000,000 for 25 years. For this fee, franchisees receive training, assistance on site
selection, assistance during the construction phase, and promotional considerations for the grand
opening, including a visit by Doriedel Salvador and Lia Mendoza-Lagcao Food, Inc. There is also a
P500,000 per month continuation fee for institutional advertising and accounting services after the
store is open for business. On March 20, 20x5 of the current year, Doriedel Salvador and Lia
Mendoza-Lagcao Food, Inc. signed an agreement and sold a franchise to Ronald Chua and Jackson
Ongsip for the standard fee. The franchisor received a 20% down payment and a 10%, four-year note
for the balance. On June 15, 20x5 the franchise outlet of Ronald Chua and Jackson Ongsip had his
grand opening and Doriedel Salvador and Lia Mendoza-Lagcao Food, Inc. had met all requirements
for the services rendered such as performance obligations related to the franchise rights, training
and equipment. On December 15,20x5 Doriedel Salvador and Lia Mendoza-Lagcao Food, Inc.
received P500,000 annually for the the continuous services rendered.

The amount of revenue from franchise fee on March 20, 20x5:


a. Nil
b. P500,000
c. P 1,000,000
d. P25,000,000
___A___(33.33 %)
The amount of revenue from franchise fee on June 15, 20x5:
a. Nil
b. P500,000
c. P 1,000,000
d. P25,000,000
___D___(33.33 %)
The amount of revenue from continuing franchise fee-services on December 15, 20x5:
35.
a. Nil
b. P500,000
c. P1,000,000
d. P25,000,000
___B___(33.33 %)
3 / 3 points
Gina Lopez signs a 1-year contract with Gary, Butch, Charlie and Ronald Health Club. The terms of the
contract are that Gina Lopez is required to pay a non-refundable initiation fee of P3,600 and a
membership fee of P900 per month. Gary, Butch, Charlie and Ronald Health Club determine that its
customers, on average, renew their annual membership two times before terminating their
membership.

How many performance obligations exist in this contract for membership fee arrangement?
a. 0
b. 1
c. 2
d. 3
___B___(33.33 %)
The transaction price of this arrangement - non-refundable upfront fee should be:
a. Nil
b. P 3,600
c. P32,400
d. P36,000
___D___(33.33 %)
The amount of revenue reported each year:
a. Nil
b. P3,600
c. P6,300
d. P12,000
___D___(33.33 %)
1 / 1 point
A health club enters into a one-year contract with a customer for an unlimited health club access for
P150 per month. The contract requires the customer to pay a non-refundable upfront fee of P240.
The upfront fee does not transfer a good or service to the customer and & in effect, an advance
payment for health club access. The upfront fee will be recognized as revenue on a straight -line basis
throughout the year. As the health club recognizes the revenue from the contract with customer.
The monthly revenue from the contract with this customer will be

P150

P170

P240

P390
Question 25 4 / 4 points
Use the following information for questions 45-48 - With Prepayments, Coupons, Customer
Options and Material Right

The Rink offers annual P2,000 memberships that entitle members to unlimited use of ice -skating
boilies and locker rooms. Each new membership also entitles the member to receive ten "20% off a
P50 meal" coupons that are redeemable at the Rink's snack bar. The Rink estimates that
approximately 80% of the coupons will be redeemed, and that, if the coupons weren't redeemed,
P50 meals still would be discounted by 5% because of ongoing promotions.

How many performance obligations exist in this contract for membership fee arrangement?
a. 0
b. 1
c. 2
d. 3
___C___(25 %)
Calculate how much of the transaction price should be allocated?
a. P 58.25
b. P 60.00
c. P2,000
d. P2.060
___C___(25 %)
How much of the contract price would be allocated to each performance obligation (stand -alone
selling price of meals coupon voucher and membership fee. respectively)?
a. P 0; P2,060
b. P58.25;P2,060
c. P58.25; P2,000
d. P60.00; P2,000
e. P58.30; P1,941.7
___E___(25 %)
The journal entry to recognize sale of a new membership. Clearly deferred revenue associated with
each performance obligation:

a. DR Cash 2,000
CR Deferred revenue - membership fees 1,941.75
CR Deferred revenue - meals coupons 58.25

b. DR Cash 2,000
CR Deferred revenue - membership fees 2,000

c. DR Cash 2,000
CR Deferred revenue – meals coupons 2,000

d. DR Cash 2,000
CR Revenue – membership fees 1,941.75
CR Revenue – meals coupons 58.25
___A___(25 %)
3 / 3 points
Use the following information for questions 49-51:

Assume that a customer enrolls in AAA's Premier Membership, which provides 12 months of
roadside assistance for P1,200. On August 1, 20x6, a customer purchases a contract that that runs
from that date through July 31, 20x7. Given that roadside assistance requests occur equally
throughout the contract period, AAA uses "proportion of time" as its measure of progress toward
completion.

The journal entry on August 1, 20x6:


a. DR Cash 1,200
CR Deferred revenue 1,200
b. DR Cash 100
CR Deferred revenue 100
c. DR Cash 1,200
CR Revenue 1,200
d. DR Cash 100
CR Deferred revenue 100
___A___(33.33 %)
The journal entry on December 31, 20x6:
a. DR Cash 500
CR Deferred revenue 500
b. DR Deferred revenue 500
CR Sales revenue 500
c. DR Deferred revenue 1,200
CR Sales revenue 1,200
d. DR Cash 1,200
CR Deferred revenue 1,200
___B___(33.33 %)
The current liability section of the balance sheet on December 31, 20x6 amounted to:
a. Nil
b. P500
c. P700
d. P1,200
___C___(33.33 %)
3 / 3 points
Use the following information for questions 52 to 54:

On June 1, DD Company shipped twenty five DVD to BB View Store on consignment. The DVD is to be
sold at an advertised price of P200 per item. The cost of each DVD to the consignor is P100. The
consignor paid P75 to ship the merchandise. Commission is to be 25% of sales price. During the
month, two DVD were returned.
On June 30, BB View Store remitted the amount due to consignor after deducting commission of
P400.

The amount remitted by BB View Store is:


52.
a. P1,100
b. P1,600
c. P1,200
d. P2,000
___C___(33.33 %)
The consignment profit is:
a. P370
b. P415
c. P720
d. P800
___A___(33.33 %)
The cost of inventory on consignment amounted to:
a. P1,400
b. P1,550
c. P1,545
d. P1,500
___C___(33.33 %)
1 / 1 point
On August 5, 20x5, Famous Furniture shipped 20 dining sets on consignment to Furniture Outlet, Inc.
The cost of each dining set was P350. The cost of shipping the dining se amounted to P1,800 and was
paid for by Famous Furniture. On December 30, 20x5 the consignee reported the sale of 15 dining
sets at P850 each. The consignee remitted payment for the amount due after deducting a 6%
commission, advertising expense of P300, and installation and setup costs of P390. The amount cash
received by Famous Furniture is

P12,750

P11,985

P11,295

P11,685
Question 29 0 / 1 point
On August 5, 20x5, Famous Furniture shipped 20 dining sets on consignment to Furniture Outlet, Inc.
The cost of each dining set was P350 each. The cost of shipping the dining sets amounted to P1,800
and was paid for by Famous Furniture. On December 30, 20x5, the consignee reported the sale of 15
dining sets at P850 each. The consignee remitted payment for the amount due after deducting a 6%
commission, advertising expense of P300, and installation and setup costs of P390. The total profit
on units sold for the consignor is

P11,295

P4,695

P6,045

P9,945

View Feedback
3 / 3 points
TS Trading consigned 100 beds costing P600 each to PP Company. The advertised selling price is
P1,000 each bed. The consignment agreement provides that the consignee is to be allowed a
commission of 15% of the selling price. Furthermore, PP Company has to a draw a sight draft for 60%
of the cost of the beds; the advance is to be recovered periodically by monthly deductions in
proportion to units sold) from the remittances which accompany the account sales. All expenses of
the consignee are to be deducted month as incurred.

At the end of the first month, the consignee rendered an account sales showing among others the
following charges: Commission, P2,250; Advertising; P1,500; and Delivery Expense. P750.

The number of units sold by PP Co. is:


57.
a. 10
b. 15
c. 20
d. 25
___B___ (C)
The amount remitted to TS Co. for the month is
a. P1,500
b. P4,500
c. P5,100
d. P5,500
___C___(33.33 %)
The consignment profit (loss) of TS Co. is:
a. (P1,500)
b. P2,137.50
c. P3,412.50
d. None of the above
___D___ (B)

View Feedback
1 / 1 point
On November 30, Northrup Company consigned 90 freezers to Watson Company for sale at P1,600
each and paid P1.200 in transportation costs. A report of sales was received on December 30 from
Watson reporting the sale of 20 freezers, together with a remittance of the P27,200 balance due. The
remittance was net of the agreed 15% commission. How much, and in what month, should Northrup
recognize as consignment sales revenue?

November P0; December P32,000

November P0; December P27,200

November P144,000; December P0

November P142,800; December P0

Question 32 2 / 2 points
On October 1, 20x4, the NN Company consigned one hundred wall clocks to P&G Retailers, Inc. Each
wall clock had a cost of P150. Freight on the shipment was paid by NN Company for P200. On
December 1, 20x4, P&G submitted an account sales stating that it had sold sixty pieces and it was
remitting the P12,840 balance due. The remittance was net of the following deductions from the
sales price of the wall clocks sold:
Commission (20% of sales price) . . . . . . . . . . . ?
Advertising . . . . . . . . . . . . . . . . . . . . . . . . . . . . .P500
Delivery and installation charges . . . . . . . . . . .P100
What was the total sales price of the wall clocks sold by P&G Retailers, Inc.
61.
a. P13,440
b. P15,000
c. P16,800
d. P17,000
___C___(50 %)
What was the cost of inventory on consignment?
a. P6,000
b. P6,080
c. P6,280
d. P6,320
___B___ (A)

View Feedback
3 / 3 points
Use the following information for questions 63 to 65:

The CC Manufacturing Company delivered ten DVD players to CLTV Company on consignment. These
DVD player cost P3,000 each and are to be sold at P5,000 each. The CC Manufacturing Co paid
shipment cost of P2,500. CLTV Co. submitted an account sales stating that it had returned one unit
and was remitting P21,900. This amount represents the total amount due to C Manufacturing Co.
after deducting the following from the selling price of the DVD player sold:

Commission . . . . . . . . . . . . . . . . . . . . . . . . . . . . . . . . . 20% of selling price


Advertising . . . . . . . . . . . . . . . . . . . . . . . . . . . . . . . . . . P1,000
Delivery and installation . . . . . . . . . . . . . . . . . . . . . . . P 600
Cartage on consigned goods . . . . . . . . . . . . . . . . . . . . . P 500

The number of sold by TV Co. is:


a. 4
b. 5
c. 6
d. None of the above
___C___(33.33 %)
The profit (loss) on consignment realized by CC Manufacturing Company is:
a. P2,300
b. P2,480
c. (P2,550)
d. None of the above
___A___(33.33 %)
The cost of inventory in the hands of CLTV Company is:
a. P10,080
b. P10,150
c. P10,200
d. None of the above
___D___(33.33 %)
2 / 2 points
Use the following information for questions 66 and 67:

On May 1, 20x4, TV Inc. consigned 80 VCD players to Ed's TV. The VCD player cost P270. Freight on
the shipment paid by Ed's TV was P600. On July 10, TV Inc. received an account sales and P12900
from Ed's TV. Thirty VCD players had been sold and the following expenses were deducted

Freight . . . . . . . . . . . . . . . . . . . . . . . . . . . . . . . . . . . . . 20% of selling price


Commission (20% of sales price). . . . . . . . . . . . . . . . . P600
Advertising . . . . . . . . . . . . . . . . . . . . . . . . . . . . . . . . . . P390
Delivery . . . . . . . . . . . . . . . . . . . . . . . . . . . . . . . . . . . . . P210

The total sales price of the VCDs sold by Ed's TV was:


a. P15,375
b. P16,125
c. P16,388
d. P17,625
___D___(50 %)
The inventory of VCDs will be reported on whose balance sheet and at what amount
a. Balance Sheet of TV Inc.; Amount of P13,875
b. Balance Sheet of TV Inc.; Amount of P13,500
c. Balance Sheet of Ed TV; Amount of P13,875
d. Balance Sheet of Ed TV; Amount of P13,500
___A___(50 %)
3 / 3 points
Use the following information for questions 68 to 70:

On May 15, 20x4. AA Sales Company received a shipment of merchandise with a selling price P15,000
from PC Company. The consigned goods cost PC Company P10,000 and freight charges of P120 had
been paid to ship the goods to AA Sales Company.

The consignment arrangement provided for a sale of merchandise on credit with terms of 2/10 n30.
The 15% commission is to be based on the accounts receivable collected by the consignee. Cash
discounts taken by customers, expenses applicable to goods on consignment and any cash advanced
to the consignor are deductible from the remittance by the consignee.

AA Sales Company advanced P6,000 to PC Company upon receipt of the shipment An expense of
P800 was paid by AA. By June, 20x4, 70% of the shipment had been sold, and 80% of the resulting
accounts receivable had been collected, all within the discount period. Remittance of the amount
due was made on June 30, 20x4.
The cash remitted by AA Sales Company is:
68.
a. 172
b. 800
c. 972
d. 2,340
___A___(33.33 %)
The profit on consignment is:
a. 750
b. 873
c. 1,188
d. 1,428
___B___(33.33 %)
The cost of unsold units in the hands of AA is:
a. 3,186
b. 3,036
c. 1,500
d. None of the above
___B___(33.33 %)
3 / 3 points
Use the following information for questions 71 to 73:

On May 1, RR Products Company ships five (5) of its appliances to SZ Company on consignment. The
cost of the appliances shipped is P155 per unit. The consignor paid shipping costs totaling P5O. Each
unit is to be sold at P250 payable P50 in the month of purchase and P10 per month thereafter. The
consignee is entitled to 20% of all amounts collected on consignment sales.

SZ Company was able to sell 3 appliances in May and 1 in June. Regular monthly collections by the
consignee, and appropriate cash remittances have been made to the consignor at the end of each
month.

The total amount remitted to consignor as of June is:


a. P0
b. P64
c. P184
d. P200
___C___(33.33 %)
The profit on consignment is:
a. P294
b. P150
c. P150
d. None of the above
___D___(33.33 %)
The cost of inventory on consignment is:
a. P155
b. P165
c. P245
d. None of the above
___B___(33.33 %)
3 / 3 points
Use the following information for questions 74 to 76:

AL Company consigned five calculators, with cost of P800 each, to the OO Company which was to sell
these goods for the account and ink of the former for a commission of 15% of selling price. The AL
Company paid shipping costs of P200 on the shipment. Correspondingly, OO Company paid P320 on
the freight of the shipment.

On the last day of the year, OO Company reported that it had sold three of the calculators, two for
cash at P1,500 each and one on credit at P1,800, of which 25% was collected as down payment. OO
Company remitted all the cash due.

The amount of cash remitted by OO Company is:


74.
a. P3,760
b. P2,410
c. P1,350
d. None of the above
___B___(33.33 %)
The consignment profit (loss) is:
a. P1,368
b. P1,160
c. P1,040
d. None of the above
___A___(33.33 %)
The amount of inventory on consignment of AL Company is:
a. P1,720
b. P1,808
c. P2,712
d. None of the Above
___B___(33.33 %)
3 / 3 points
Use the following information for questions 77 to 79:
On October 5, 20x4, the PPG Trading Co. consigned 30 computer units, costing P8,000 each, to
Pampanga, Inc. The units were to be sold on either cash or credit basis at a com mission of 15%
consignee received the goods. Sales were made as follows:

October 15: 10 units for cash at P13,000 each


October 28: 12 units on account at P14,000 each

On October 31, 20x4, collections on accounts amounted to P95,000, and an allowance of P2,000 was
given to a charge customer for a defective unit. On November 15, 2014. On October 31, 20x4, a
receivable balance of P7,000 was determined to be uncollectible. On December 21, 20x4, the
consignee made the proper remittance.

The amount due from BB, Inc. is:


a. P64,000
b. P71,000
c. P80,000
d. None of the above
___D___(33.33 %)
The consignment profit is:
a. P46,400
b. P48,400
c. P55,400
d. None of the above
___D___(33.33 %)
The cost of inventory on consignment is:
a. P70,800
b. P66,800
c. P66,800
d. None of the above
___D___(33.33 %)
1 / 1 point
NN Company consigns sign pens to retailers, debiting Accounts Receivable for the retail sales price of
the sign pens consigned and crediting Sales. All costs relating to the consigned sign pens are debited
to expenses of the current accounting period. Net remittances of the consignees are credited to
Accounts Receivable. In December, 800 sign pens costing P60 each and retailing for P100 a unit were
consigned to SS Store. Freight cost of P800 was debited to Freight Expense by the consignor. On
December 31, SS Store remitted P35,505 to NN Company in full settlement of the balance due.
Accounts Receivable was credited for this amount. The consignee deducted a commission of P10 on
each sign pens sold and P45 for delivery expense. The number of sign pens sold by SS Store is:

355

395

400
None of the above
Question 40 1 / 1 point
Berry Farm produces organic tomatoes and strawberries. In June 20x6, it transported 100 boxes of
strawberries with a price of P20 per box to the Bay Farmers' Market. Berry Farm paid an upfront fee
of P100 to present its products at the market for one week, and the market earns a 25% profit
margin on each item sold, but Berry Farm is responsible for any items that remain unsold at the end
of the week. The market was able to sell 65 boxes of strawberries to customers. How much revenue
should Berry Farm recognize with respect to this transaction?

None

P1,300

P2,000

Incomplete data
Question 41 1 / 1 point
Holmgren Seafoods, Inc. catches and processes salmon and tuna caught off the coast of Maine. In
May 20x6, it placed 100 freshly caught wild salmon with a retail price of P75 each in Joe's Fish Shop.
Holmgren's contract with the shop stipulates that the shop will earn a 15% commission on each
salmon sold. Joe's is responsible for purchasing any fish that remain unsold at the end of a three -day
period. During the three-day period, Joe's Fish Shop was able to sell 88 of the 100 salmon. How much
revenue should Holmgren recognize with respect to this transaction?

None

P75

P7,500

Incomplete data

Use the following information for questions 83 and 84:

Information relating to regular sales and consignment sales of EE Products for the year ended June
30, 20x4 follows:

Regular Sales Consignment Sales Total


Sales P120,000 P30,000 P150,000
Cost of Sales 84,000 26,000 110,000
Operating Expenses ? 1,760 16,910

You ascertain that merchandise costing P6,500 are in the possession of consignees and are included
in the cost of consigned merchandise sold. Operating expenses of P15,150 (more than half of which
are fixed) are to be allocated to regular sales and to consignment sales on the basis of volume. The
P1,760 operating expenses relating to consignment sales include a commission of 5% and P260 Costs
incurred by consignees relating to the entire shipment of merchandise worth P26,000.
The net income on regular sales is:
a. P30,280
b. P23,880
c. P17,380
d. None of the above
___B___(50 %)
The net income on consignment sales is
a. P8,740
b. P5,710
c. P2,240
d. None of the above
___D___(50 %)
1 / 1 point
Seahawks, Inc. had the following consignment transactions during December:

Inventory shipped on consignment to Ashe Company . . . . . P18,000


Freight paid by Seahawks . . . . . . . . . . . . . . . . . . . . . . . . . . . . . 900
Inventory received on consignment from Fenn Company . . . 12,000
Freight paid by Fenn . . . . . . . . . . . . . . . . . . . . . . . . . . . . . . . . . 500

No sales of consigned goods were made through December 31. Seahawks' December 31 balance
sheet should include consigned inventory at

P18,900

P18,000

P12,500

P12,000
CORPORATE LIQUIDATION
1 / 1 point
Port Corporation is a parent, having purchased 80% of Sand Company’s common stock at par value
for P800,000. Sand Company is in financial difficulty The parent granted an unsecured loan of
P400,000 to the subsidiary A accounting statement of affairs for Sand Company shows a dividend of
40%. Port Corporation can expect to receive payments for its investment in Sand Company at
approximately:

P640,000

P320,000

P160,000

P0

Question 2 1 / 1 point
The following items were displayed in the statement of affairs for Lubbock Company:

Unsecured liabilities without priority P90,000


Stockholders’ equity 36,000
Loss on realization of assets 45,000
Estimated administrative costs that have not 4,500
been entered in the accounting records
Unsecured liabilities with priority 10,000

Based on the foregoing information, what percentage of their claims should unsecured, non -priority
creditors expect to receive on the liquidation of Lubbock Company:

85%

90%

86.50%

100%

Question 3 1 / 1 point
Lakeside Bank holds a P100,000 note secured by a building owned by Fly-By-Night Manufacturing,
which has filed for bankruptcy. If the property has a book value of P120,000 and a fair market value
of P90,000, what is the best way to describe the note held by Second Bank and Trust Company? The
bank has a(n)

Secured claim of P100,000.


Unsecured claim of P100,000.

Secured claim of P90,000 and an unsecured claim of P10,000.

Secured claim of P100,000 and an unsecured claim of P20,000.

Question 4 1 / 1 point
Splat Company filed a voluntary bankruptcy petition, and the statement of affairs reflected the
following amounts:

Assets Estimated Current Value


Book Value
Assets pledged with fully secured creditors P900,000 P1,110,000
Assets pledged with full secured creditors 540,000 360,000
Free assets 1,260,000 960,000
P2,700,000 P2,430,000
Liabilities
Liabilities with priority P210,000
Fully secured creditors 780,000
Partially secured creditors 600,000
Unsecured creditors 1,620,000
P3,210,000
Assume the assets are converted to cash at their estimated current values. What amount of cash will
be available to pay unsecured non-priority claims?

P720,000

P840,000

P960,000

P1,080,000

Question 5 1 / 1 point
Dobby Corporation was forced into bankruptcy and is in the process of liquidating assets and paying
claims. Unsecured claims will be paid at the rate of thirty cents on the peso. Carson holds a note
receivable from Dobby for P75,000 collateralized by an asset with a book value of P50,000 and a
liquidation value of P25,000. The amount to be realized by Carson on this note is:

P25,000

P40,000

P50,000
P75,000

Question 6 1 / 1 point
Poor Company filed a voluntary bankruptcy petition, and the settlement of affairs reflected the
following amounts:

Assets Estimated Current Value


Book Value
Assets pledged with fully secured creditors P450,000 P555,000
Assets pledged with full secured creditors 270,000 180,000
Free assets 630,000 480,000
P1,350,000 P1,215,000
Liabilities
Liabilities with priority P105,000
Fully secured creditors 390,000
Partially secured creditors 300,000
Unsecured creditors 810,000
P1,605,000
Assume the assets are converted to cash to their estimated current values. What amount of cash will
be available to pay unsecured non-priority claims?

P360,000

P420,000

P480,000

P540,000

Question 7 1 / 1 point
Target Corporation was forced into bankruptcy and is in the process of liquidating assets and paying
claims. Unsecured claims will be paid at the rate of thirty cents on the peso. Arrow holds a note
receivable from Target for P90,000 collateralized by an asset with a book value of P60,000 and a
liquidation value of P30,000. The amount to be realized by Arrow on this note is:

P30,000

P48,000

P60,000

P90,000

Question 8 1 / 1 point
Sparkman Co. filed a bankruptcy petition and liquidated its noncash assets. Sparkman was paying
forty cents on the dollar for unsecured claims. Bailey Co. held a mortgage of P150,000 on land that
was sold for P110,000. The total amount of payment that Bailey should have received is calculated to
be

P110,000

P44,000

P126,000

P134,000

P60,000

Question 9 1 / 1 point
Equipment with a book value of P120,000 is sold in a liquidation process for cash of P110,000. This
equipment was security for a P150,000 bank loan. Any remainder is considered unsecured, how
would this transaction be reported on the Statement of Realization and Liquidation?

A reduction in non-cash assets of P120,000

A loss reported to owners’ equity of P10,000

A disbursement of cash to the bank of P110,000, a reduction in partially secured liability


of P150,000, and an increase in unsecured without priority liability of P40,000

All of the above would occur

Question 10 1 / 1 point
Use the following information for questions 10 to 12:
Orville Company recently petitioned for bankruptcy and is now in the process of preparing a
statement of affairs. The carrying values and estimated fair values of the assets of Orvile Company
are as follows:

Carrying Value Fair Value


Cash P20,000 P20,000
Accounts receivable 45,000 30,000
Inventory 60,000 35,000
Land 75,000 70,000
Building (net) 180,000 100,000
Equipment (net) 170,000 80,000
Total P550,000 P335,000

Debts of Orville are as follows:

Accounts payable P60,000


Wages Payable (all have priority) 10,000
Taxes payable 10,000
Notes payables (secured by receivable and 120,000
inventory)
Interest on Notes Payable 6,000
Bonds Payable (secured by land and building) 150,000
Interest on bonds payable 7,000
Total P363,000

What is the total amount of unsecured claims?

P93,000

P113,000

P121,000

P126,000

Question 11 1 / 1 point
What estimated amount will be available for general unsecured creditors upon liquidation?

P28,000

P93,000

P113,000

P121,000

Question 12 1 / 1 point
What is the estimated dividend percentage?

23%

93%

77%

68%

Question 13 1 / 1 point
Eagle Company recently petitioned for bankruptcy and is now in the process of preparing a
statement of affairs. The following information has been assembled for the statement:

Assets Book Value Estimated Current Value


Cash P70,000 P70,000
Other current assets 240,000 230,000
Building 600,000 700,000
Land 200,000 300,000

Liabilities
Liabilities with priority P140,000
Mortgage payable (secured by building) 300,000
Note Payable (secured by land) 400,000
Unsecured liabilities 600,000

What amount will be paid to the fully secured creditors and the creditors with priority?

Fully Secured Creditors P300,000; Creditors with Priority P140,000

Fully Secured Creditors P300,000; Creditors with Priority P92,000

Fully Secured Creditors P600,000; Creditors with Priority P92,000

Fully Secured Creditors P700,000; Creditors with Priority P140,000

Question 14 1 / 1 point
Mandich Co. had the following amounts for its assets, liabilities and stockholders’ equity accounts
just before filing a bankruptcy petition and requesting liquidation:

Book Value Net Realizable Value


Cash P10,000 P10,000
Accounts Receivable 100,000 60,000
Inventory 350,000 350,000
Land 110,000 75,000
Building and Equipment 700,000 300,000
Accounts Payable 100,000
Salaries Payable 75,000
Notes Payable (secured by 300,00
inventory)
Employees’ claims for 10,000
distribution to pension plans
Taxes Payable 80,000
Liability for accrued expenses 20,000
Bonds Payable 500,000
Common Stock 200,000
Additional Paid-In Capital 100,000
Retained Earnings Deficit (115,000)
Of the salaries payable, P35,000 was owed to an officer of the company. The remaining amount was
owed to salaried employees who had not been paid within the previous 80 days: John Webb was
owed P2,500. The maximum amount owed for any one employee’s claims for contributions to
benefit plans was P800. Estimated expense for administering the liquidation amounted to P40,000.
What was the total amount of unsecured liabilities with priority?

P75,000

P155,000

P165,000

P170,000

P205,000

Question 15 0 / 1 point
Use the following information for questions 15 to 26:
On June 1, 20x5, the books of Dremer Corporation show assets with book values and realizable
values as follows:

Book Value Realizable Value


Cash P1,850 P1,850
Accounts Receivable (net) 21,200 17,000
Note Receivable 15,000 15,000
Inventory 41,000 20,000
Investment in Calandir Stock 5,800 15,000
Land and Building (net) 98,500 92,800
Equipment (net) 43,000 8,000
Totals P226,350 P169,650

Dremer’s books show the following liabilities:

Book Value
Accounts payable (50,000 secured by inventory and equipment) P 90,625
Wages payable (eligibility for priority) 3,775
Other Accrued Liabilities 10,000
Accrued interest on notes payable 375
Accrued interest on mortgage payable 600
Notes payable (secured by Investment in Calandir Stock) 10,000
Mortgage payable (secured by land and building) 70,000
Total P185,375

Compute the estimated amount available to unsecured creditors (with and without priority)/total
free assets:

___60,675___ (440,000)
.
1 / 1 point
Compute the estimated amount available to unsecured creditors without priority (net free assets):
___56,900___
.
1 / 1 point
Compute the amount of unsecured creditors without priority
___72,625___
.
0 / 1 point
Compute the dividend to unsecured creditors without priority (express answer in decimal, round up
to two decimal places)
___.78___ (0.78)
.
1 / 1 point
Compute the estimated payment to fully secured creditors
___80,975___
.
1 / 1 point
Compute the estimated payment to partially secured creditors
___45,160___
.
1 / 1 point
Compute the estimated payment to unsecured creditors with priority
___3,775___
.
1 / 1 point
Compute the estimated payment to unsecured creditors without priority
___39,487.50___
.
1 / 1 point
Compute the estimated payment to creditors (sum of previous four answers)
___169,397.50___
.
1 / 1 point
Compute the estimated deficiency to unsecured creditors
___15,725___
.
0 / 1 point
Compute the estimated gain or loss on realization of assets (if loss, write your answer as -xx,xxx)
___-52,500___ (-56,700)
.
1 / 1 point
Compute the estimated net (total) loss (write your answer as -xx,xxx)
___-56,700___
.
1 / 1 point
Use the following information from Dremer Corporation (refer to previous problem) and the
following information, for the period of 6/1/x5 and 6/30/x5.
No subsequent discoveries
Sale of Calandir Securities to a market value of P16,000
Collection of Note Receivable into cash P15,000
Sale of Equipment at P7,000
Sale of Inventory at P22,000
Partial Payment of Accounts Payable P29,000
Payment of Note Payable P10,375
Compute the cash balance on June 30, 20x5:

___22,475___
.
1 / 1 point
Compute the noncash balance on June 30, 20x5:
___119,700___
.
1 / 1 point
Compute the balance of fully secured liabilities on June 30, 20x5
___70,600___
.
1 / 1 point
Compute the balance of partially secured liabilities on June 30, 20x5
___0___
.
1 / 1 point
Compute the balance of liabilities with priority on June 30, 20x5:
___3,775___
.
0 / 1 point
Compute the balance of liabilities without priority on June 30, 20x5:
___71,625___ (71,265)
.
1 / 1 point
Use the following information for questions 33 and 34:
Zero Na Corp. has been undergoing liquidation since January 1. As of March 31, its condensed
statement of realization and liquidation is presented below:

Assets:
Assets to be realized P1,375,000
Assets acquired 750,000
Assets realized 1,200,000
Assets not realized 1,375,000
Liabilities:
Liabilities liquidated P1,875,000
Liabilities not liquidated 1,700,000
Liabilities to be liquidated 2,250,000
Liabilities assumed 1,625,000
Revenue and Expenses:
Supplementary charges/debits P3,125,000
Supplementary credits 2,800,000

The net gain (loss) for the three-month period ending March 31 is:

P250,000
(P325,000)

P425,000

P750,000

Question 34 1 / 1 point
Compute the ending cash balance of cash account assuming that common stock and deficits are
P1,500,000 and P500,000, respectively:

P425,000

P575,000

P1,325,000

P1,375,000

Question 35 1 / 1 point
When is a “statement of affairs” used?

Only in liquidations.

Only in reorganizations.

In both liquidations and reorganizations.

In preparing a statement of realization and liquidation.

None of the above.

Question 36 1 / 1 point
In a “statement of affairs,

Assets pledged with partially secured creditors are shown on the asset side of the
statement and as a deduction on the liability side of the statement.

Assets pledged with fully secured creditor are shown only on the liability side of the
statement.

Liabilities owed to fully secured creditors are shown only on the asset side of the
statement.

Liabilities owed to partially secured creditors are shown on the asset side of the balance
sheet and as a deduction on the liability side of the statement.
None of the above.

Question 37 1 / 1 point
In a “statement of affairs”,

Liabilities with priority are shown on the liability side of the statement and as a
deduction on the asset side of the statement.

Assets pledged with fully secured creditors are shown on the liability side of the
statement and as a deduction on the asset side of the statement.

Liabilities owed to fully secured creditors are shown on the asset side of the statement
and as a deduction on the liability side of the statement.

Liabilities owed to partially secured creditors are shown on the asset side of the balance
sheet and not as a deduction on the liability side of the statement.

None of the above.

Question 38 1 / 1 point
A debtor filing a debtor’s bankruptcy petition will not be discharged if the debtor had received a
prior discharge in bankruptcy within the past:

Two years

Four years

Six years

Eight years

Question 39 1 / 1 point
Typically, the estimated amount available for short-term prepayments in a statement of affairs
(financial statement) is:

Zero

Carrying amount

Current fair value

Net realizable value

Question 40 1 / 1 point
William Bautista is starting a new business, Bautista Enterprises, which will be single proprietorship
selling retail novelties. Bautista recently received a discharge in bankruptcy, but certain proved
claims were unpaid because of insufficient funds. Which of the following is still a claim against
Bautista?

The unpaid amounts owed to secured creditors who received less than the full amount
after resorting to their security interest and receiving their bankruptcy cash payments

The unpaid amounts owed to trade creditors for merchandise purchased and sold by
Bautista in the ordinary course of his prior business enterprise

A personal loan to Bautista by his father made in an attempt to avoid bankruptcy

The unpaid amount of income taxes payable to the United Sates that became due
within three years preceding Bautista’s insolvency

Question 41 1 / 1 point
What is defined as a condition in which a company is unable to meet debts as the debts mature?

Deficit

Liability

Insolvency

Credit squeeze

Question 42 1 / 1 point
What type of ledger account is the Estate Deficit account used in the trustee’s accounting records for
a debtor in a bankruptcy liquidation?

Asset

Liability

Equity

Revenue

None of the above

Question 43 1 / 1 point
Which of the following is first-ranked of the unsecured liabilities with priority in bankruptcy
liquidation?

Claims of governmental entities for various taxes or duties

Administrative costs
Claims for wages, salaries, and commissions, subject to limitations of amount and time

None of the foregoing

Question 44 1 / 1 point
The accounting equation for a trustee in bankruptcy liquidation is:

Assets equal liabilities plus owners’ equity

Assets equal accountability

Assets equal liabilities minus estate deficit

Assets minus liabilities equals accountability

Question 45 1 / 1 point
Nimbus Company has incurred large nee losses for the past two years. Because of its inability to pay
current liabilities, Nimbus has filed a petition for reorganization under the Bankruptcy Law. The
reorganization provisions of the Bankruptcy Law:

Require that the bankruptcy court appoint a trustee in all cases

Permit Nimbus management to remain in possession of its assets

Apply only to creditors’ bankruptcy petitions

Will apply to Nimbus only if Nimbus is required to register with the Securities and
Exchange Commission pursuant to the Philippine securities laws

Question 46 1 / 1 point
In the journal entry to open the accounting records of a trustee in Chapter 7 bankruptcy liquidation,
the debit to the Estate Deficit ledger account is in the statement of affairs amount of the:

Estimated deficiency to unsecured, non-priority creditors

Total estimated amount available

Estimated amount available for unsecured, non-priority creditors

Stockholders’ equity of the debtor corporation

Question 47 1 / 1 point
Under the Bankruptcy Code, do creditors having priority include

Partially Secured Creditors - Yes; Specified Unsecured Creditors - Yes

Partially Secured Creditors - Yes; Specified Unsecured Creditors - No


Partially Secured Creditors - No; Specified Unsecured Creditors - Yes

Partially Secured Creditors - No; Specified Unsecured Creditors - No

Question 48 1 / 1 point
What are the objectives of bankruptcy laws in the Philippines?

Provide relied for the court system and ensure that all debtors are treated the same.

Distribute assets fairly and discharge honest debtors from their obligation.

Protect the economy and stimulate growth.

Prevent insolvency and protect shareholders.

Question 49 1 / 1 point
In a bankruptcy, which of the following statements is true?

An order for relief results only from a voluntary petition.

Creditors entering an involuntary petition must have debts totaling at least P20,000.

Secured notes payable are considered liabilities with priority on a statement of affairs.

None

Question 50 1 / 1 point
In reporting a company that is to be liquidated, assets are show at:

Present value calculated using an appropriate effective rate.

Net realizable value.

Historical cost.

Book value.

Question 51 1 / 1 point
An involuntary bankruptcy petition must be filed by:

The insolvent company’s attorney.

The holders of the insolvent company’s debenture bonds.


Unsecured creditors with total debts of at least P13,475.

The company’s management.

2 1 / 1 point
An order for relief

Prohibits creditors from taking action to collect from an insolvent company without
court approval.

Calls for the immediate distribution of free assets to unsecured creditors.

Can be entered only in an involuntary bankruptcy proceeding.

Gives an insolvent company time to file a voluntary bankruptcy petition.

Question 53 1 / 1 point
On a statement of financial affairs, how are liabilities classified?

Current and noncurrent.

Secured and unsecured.

Monetary and nonmonetary.

Historic and futuristic.

Question 54 1 / 1 point
What is a debtor in possession?

The holder of a note receivable issued by an insolvent company prior to the granting of
an order for relief.

A fully secured creditors.

The ownership of an insolvent company that continues to control the organization


during a bankruptcy reorganization.

The stockholders in a bankruptcy proceeding.

Question 55 1 / 1 point
How are anticipated administrative expenses reported on a statement of financial affairs?

As a footnote until actually incurred.


As a liability with priority.

As a partially secured liability.

As an unsecured liability.

Question 56 1 / 1 point
What is an inherent limitation of the statement of financial affairs?

Many of the amounts reported are only estimations that might prove to be inaccurate.

The statement is applicable only to bankruptcy.

The statement covers only a short time, whereas a bankruptcy may last much longer.

The figures on the statement vary as to a voluntary and an involuntary bankruptcy.

Question 57 1 / 1 point
On a balance sheet prepared for a company during its reorganization, at what balance are liabilities
reported?

As current and long term

As monetary and nonmonetary

As subject to compromise and not subject to compromise

As equity related and debt related

Question 58 1 / 1 point
On a balance sheet prepared for a company during its reorganization, at what balance are liabilities
reported?

At the expected amount of the allowed claims.

At the present value of the expected future cash flows.

At the expected amount of the settlement.

At the amount of the anticipated final payment.

Question 59 1 / 1 point
A corporation that is unable to pay its debts as they become due is:

Bankrupt
Overdrawn

Insolvent

Liquidating

Question 60 1 / 1 point
To assist the trustee, a debtor must

Collect and reduce to money any non-exempt property

File progress reports with the court

File a statement of affairs, consisting of answers to a series of questions regarding


debtor’s financial condition

Pay dividends to creditors with regards to priorities

Question 61 1 / 1 point
Which of the following statements is true?

Certain debts are not dischargeable.

The goal of liquidation is to give the company a new start.

All secured claims are paid in full.

The expenses to administer the estate are paid last because they are unsecured.

Question 62 1 / 1 point
Which of the following does not describe the accounting statement of affairs?

The emphasis is on asset net realizable value, not historical cost

The statement of affairs is concerned only with the assets of the debtor organization,
not the claims

The statement can also be used in a reorganization

The statement of affairs is based on estimated values; actual realized values may be
different

Question 63 1 / 1 point
The document used to estimate amounts available to each class of claims is called a(n)
Statement of Assets and Liabilities.

Legal Statement of Affairs.

Accounting Statement of Affairs.

Statement of Realization and Liquidation.

Question 64 1 / 1 point
The document used by a trustee to report periodically on the status of fiduciary activities is

Statement of Assets and Liabilities.

Legal Statement of Affairs.

Accounting Statement of Affairs.

Statement of Realization and Liquidation.

Question 65 1 / 1 point
After eliminating the deficit in a reorganization plan, a balance may remain in Reorganization Capital.
On the balance sheet, where would this account appear?

Part of the Paid-In Capital

Part of the dated balance in Retained Earnings

An Intangible Asset if the balance is a debit

A deferred credit amortized over a period not to exceed 40 years

Question 66 1 / 1 point
The ratio called “dividend to general unsecured creditors” is calculated by which of the following
formulas?

Estimated amount available for unsecured creditors with/without priority divided by


the Total claims of all unsecured creditors with/without priority

Estimated realizable value of all debtor assets divided by book value of debtor assets

Estimated gain/loss on liquidation divided by Total estimated net realizable value of


debtor assets

Net estimated proceeds available to unsecured creditors divided by Total claims of


unsecured creditors
Question 67 1 / 1 point
In the accounting statement of affairs, the gains or losses upon liquidation would equal

Net book value of assets minus book value of liabilities.

The book value of assets minus their realizable value.

Total estimated realizable value of assets minus the amount assigned to secured
creditors.

Total estimated realizable value of assets minus the amount remaining for unsecured
creditors

Question 68 1 / 1 point
A corporation’s accounting statement of affairs shows a dividend of 40%. The dividend means that

All creditors and stockholders will receive approximately 40% of the book value of their
respective interests.

All creditors will receive an amount approximately equal to 40% of the book value of
their claims, but stockholder will receive nothing.

Class 1-6 unsecured claims will receive 40% of the book value of their respective claims.

Class 7 unsecured claims will receive 40% of the book value of their respective claims.

Question 69 1 / 1 point
A corporation’s accounting statement of affairs shows a dividend of 115%. The dividend means that

Secured creditors will receive an amount in excess of the book value of their claims.

Unsecured creditors will receive an amount in excess of the book value of their claims.

Stockholders may expect some return on their interests.

An error was made in the preparation of the statement.

Question 70 1 / 1 point
The Statement of Realization and Liquidation differs from the Statement of Affairs because

The Statement of Realization and Liquidation reports estimated realizable values rather
than actual liquidation results

The Statement of Realization and Liquidation is a summary of secured debt activity only

The Statement of Realization and Liquidation is prepared only at final completion of the
liquidation process
The Statement of Realization and Liquidation reports actual liquidation results rather
estimated realizable values.

Question 71 1 / 1 point
When a business become insolvent, it generally has three possible courses of action. Which of the
following is not one of the three possible courses of action?

The debtor and its creditors may enter into a contractual agreement, outside of formal
bankruptcy proceedings.

The debtor continues operating the business in the normal course of the day-to-day
operations.

The debtor or its creditors may file a bankruptcy petition, after which the debtor is
liquidated.

The debtor or its creditors may file a petition for reorganization.

Question 72 1 / 1 point
A composition agreement is an agreement between the debtor and its creditors whereby the
creditors agree to

Accept less than the full amount of their claims.

Delay settlement of the claim until a later date.

Force the debtor into a liquidation.

Accrue interest at a higher rate.

Question 73 1 / 1 point
A debtor ma file which type of petition when seeking judicial protection under the Bankruptcy Law?
I. Voluntary
II Involuntary

I only

II only

Either I or II

Neither I or II

Question 74 1 / 1 point
The duties of the trustee include:
Appointing creditor’s committees in liquidation cases.

Approving all payments for debts incurred before the bankruptcy filing.

Examining claims and disallowing any that are improper.

Calling a meeting of the debtor’s creditors.

FOREIGN CURRENCY TRANSACTIONS


1 / 1 point
The impetus behind the move to upgrade international accounting standards has recently come from
The Securities and Exchange Commission

The Securities and Exchange Commission

The International Organization of Securities Commissioners

The International Accounting Standards Board

The Financial Accounting Standards Board

The United Nations

Question 2 1 / 1 point
Which of the following is false concerning Philippine GAAP relative to US GAAP countries in general?

Philippine GAAP requires more disclosure of lines of business than US GAAP.

US GAAP requires more detailed information for interim financial reporting.

US GAAP is largely based on tax laws.

Some Western European countries allow wide latitude in smoothing out earnings.

None of the above.

Question 3 1 / 1 point
Actually changing one currency into another currency is called

Translation
Denominating

Measuring

Conversion

None of the above

Question 4 1 / 1 point
If one foreign currency units (FCU) can be exchanged for P1.50 of Philippine currency, what fraction
should be used to compute indirect quotation of the exchange rate expressed in FCU?

P1.50/1

1/P1.50

1/.667

.667/1

None of the above

Question 5 1 / 1 point
To express 1000 Foreign Currency Units (FC) in pesos, it is necessary to

Divide the indirect exchange rate by 1,000 FCUs.

Multiply the indirect exchange rate by 1,000 FCUs.

Divide the 1,000 FCUs by the direct exchange rate.

Multiply the 1,000 FCUS by the direct exchange rate.

None of the above.

Question 6 1 / 1 point
The process of expressing amounts stated in one currency in terms of another currency using
appropriate currency exchange rates is called

Measurement.

Conversion.

Translation.

Denominating.

None of the above.

Question 7 1 / 1 point
Floating exchange rates are also referred to as

Free rates.

Fixed rates.

Spot rates.

Direct rate.

None of the above.

Question 8 1 / 1 point
Exchange rates determined by market conditions are commonly referred to as

Floating rates.

Direct rates.

Spot rates

Official rates.

None of the above.

Question 9 1 / 1 point
Which of the following exchange rates does not fit under the floating exchange rate system?

Future rates.

Official rates.

Spot rates.

Forward rates.

None of the above.

Question 10 1 / 1 point
In the long run, changes in exchange rates can be best attributed to

Foreign trade deficits or surpluses.

Foreign investment deficits or surpluses.

Differential trade deficits or surpluses.

Differential rates of inflation.


None of the above.

Question 11 1 / 1 point
Which of the following items is not a cause that affects the price of a currency in either the short run
or the long run?

Interest rates.

A foreign trade deficit or surplus.

Foreign investment.

Purchasing power parity theory.

None of the above.

Question 12 1 / 1 point
A domestic company having importing and exporting transactions settlement in foreign currency will
hope that the direct exchange rate

Increases for both types of transactions.

Decreases for both types of transactions.

Increases for exporting transactions and decreases for importing transactions.

Decreases for exporting transactions and increases for importing transactions.

Question 13 1 / 1 point
In unhedged importing or exporting transactions involving, which of the following dates is not a date
having any accounting significance insofar as amounts reportable to stockholders?

The intervening financial reporting date.

The settlement date.

The transaction date.

The commitment date.

None of the above.

Question 14 1 / 1 point
For unhedged importing and exporting transactions involving credit and requiring settlement in
foreign currency, which of the following dates would never be of concern or have accounting
significance?

The forward rate date.


The transaction date.

The settlement date.

The commitment date.

None of the above.

Question 15 1 / 1 point
For importing transactions denominated in a foreign currency, any change in the exchange between
the transaction date and any intervening financial reporting date(s) is reported as

An adjustment to the foreign currency receivable.

A gain or loss to be added to be added to or subtracted from the initially recorded cost
of inventory.

A gain or loss in the current income statement.

A deferred gain or loss in the balance sheet pending settlement.

None of the above.

Question 16 1 / 1 point
For importing and exporting transactions, recognizing in the income statement FX transaction gain or
losses resulting from adjustments made at intervening financial reporting dates is not

A disregarding of the realized versus unrealized concept.

Essentially current-value accounting.

Consistent with the one-transaction perspective.

Allowed unless there is an offsetting loss or gain from a related hedging transaction.

None of the above.

Question 17 1 / 1 point
A domestic exporter has foreign currency receivables. The exporter's risk exposure is that the

Foreign currency will strengthen.

Direct exchange rate will decrease.

Peso will weaken.

Indirect exchange rate will decrease.

None of the above.


Question 18 1 / 1 point
A domestic importer whose transactions are in foreign currency has risk exposure that the

Foreign currency will strengthen.

Direct exchange rate will decrease.

Peso will weaken.

Indirect exchange rate will decrease.

None of the above.

Question 19 1 / 1 point
In a bank wire transfer, which of the following occurs?

Currency physically changes hands between the banks involved when the wire transfer
occurs.

Currency is physically moved between countries by the banks involved at the wire
transfer date.

The result is that the two banks involved create a payable and receivable between each
other.

Both b and c

Both a and b

Question 20 1 / 1 point
Concerning importing and exporting transactions, which of the following statements is false?

Gains and losses on adjustments to foreign currency receivables and payables may be
reported net in the income statement.

Gains and losses on adjustments to foreign currency receivables and payables are
unrealized in nature.

When a domestic company has a gain or loss as a result of adjusting a foreign currency
receivable or payable, the foreign company will have the opposite result.

FX transaction gains are taxable when realized.

None of the above.

Question 21 1 / 1 point
A foreign currency transaction loss occurs on an open-account purchase from a foreign supplier
denominated in local currency units (LCU) of the foreign supplier's country if the:
Buying spot rate for the LCU decreases between the purchase date and the payment
date.

Selling spot rate for the LCU decreases between the purchase date and the payment
date

Buying spot rate for the LGU increases between the purchase date and the payment
date.

Selling spot rate for the LCU increases between the purchase date and the payment
date.

Question 22 1 / 1 point
A foreign currency transaction gain or loss is:

A change in the exchange rate quoted by a foreign currency dealer.

A term synonymous with translation of a foreign currency to pesos.

The difference between the recorded pesos amount of a trade account receivable or a
trade account payable denominated in a foreign currency and the amount of pesos
ultimately received or paid.

А change from the current/noncurrent method to the monetary/nonmonetary method


of remeasuring a foreign investee's financial statements to the pesos functional
currency

Question 23 1 / 1 point
A transaction gain or loss at the settlement date is:

a change in the exchange rate quoted by a foreign currency trader.

synonymous with the translation of foreign currency financial statements into dollars.

the difference between the recorded dollar amount of an account receivable


denominated in a foreign currency and the amount of dollars received.

the difference between the buying and selling rate quoted by a foreign exchange trader
of the settlement date.

Question 24 1 / 1 point
From the viewpoint of a Philippine company, a foreign currency transaction is a transaction:

measured in a foreign currency

denominated in a foreign currency

measured in Philippine currency


denominated in Philippine currency

Question 25 1 / 1 point
The exchange rate quoted for future delivery of foreign currency is the definition of a(n):

direct exchange rate.

indirect exchange rate.

spot rate.

forward exchange rate.

6 1 / 1 point
Company X denominated a December 1, 20x4, purchase of goods in a currency other than its
functional currency. The transaction resulted in a payable fixed in terms of the amount of foreign
currency, and was paid on the settlement date, January 10, 20x5. Exchange rates moved unfavorably
at December 31, 20x4 resulting in a loss that should:

Be included as a separate component of stockholders’ equity at December 31, 20x4.

Be included as a component of income from continuing operations for 20x4.

Be included as a deferred charge at December 31, 20x4.

Not be reported until January 10, 20x5, the settlement date.

Question 27 1 / 1 point
Pigskin Co., a Philippine corporation, sold inventory on credit to a foreign company on April 8, 20x4.
Pigskin received payment of 35,000 foreign currencies (FCs) on May 8, 20x4. The exchange rate was
P1 = 0.65 FC on April 8 and P1 = 0.70 FC on May 8. What amount of foreign exchange gain or loss
should be recognized? (round to the nearest peso)

P10,500 loss

P10,500 gain

P1,750 loss

P3,846 loss

No gain or loss should be recognized

Question 28 1 / 1 point
Use the following information for questions 28 to 30:
Norton Co., a Philippine corporation sold inventory on December 1, 20x4 with payment of 10,000
foreign currencies (FC) to a foreign customer to be received in sixty days. The pertinent exchange
rates were as follows:

Dec 1 Spot rate: P1.7242


Dec 31 Spot rate: P1.8182
Jan 30 Spot rate: P1.6666

For what amount should Sales be credited on December 1?

P5,500

P16,949

P18,182

P17,241

P16,667

Question 29 1 / 1 point
What amount of foreign exchange gain or loss should be recorded on December 31?

P300 gain

P300 loss

Zero

P941 loss

P941 gain

Question 30 1 / 1 point
What amount of foreign exchange gain or loss should be recorded on January 30?

P1,516 gain

P1,516 loss

P575 loss

P500 loss

P500 gain

Question 31 1 / 1 point
Use the following information for questions 31 to 33:
Brisco Bricks purchases raw materials from its foreign supplier on May 8. Payment of 2,000,000
foreign currency units (FC) is due in 30 days. May 31 is Brisco’s fiscal year-end. The pertinent
exchange rates were as follows:

Dec 1 Spot rate: P1.25


Dec 31 Spot rate: P1.26
Jan 30 Spot rate: P1.20

For what amount should Brisco’s Accounts Payable to be credited on May 8?

P2,500,000

P2,440,000

P1,600,000

P1,639,344

P1,666,667

Question 32 1 / 1 point
How much Foreign Exchange Gain or Loss should Brisco record on May 31?

P2,520,000 gain

P20,000 gain

P20,000 loss

P80,000 gain

P80,000 loss

Question 33 1 / 1 point
How much peso will it cost Brisco to finally pay the payable on June 7?

P1,666,667

P2,440,000

P2,520,000

P2,500,000

P2,400,000

Question 34 1 / 1 point
Alpha, Inc., a Philippine company, had a receivable from a customer that was denominated in foreign
currency (FC). On December 31, 20x4, this receivable for 75,000 FC was correctly included in Alpha’s
balance sheet at P8,000. The receivable was collected on March 2, 20x5, when the peso equivalent
was P6,900. How much foreign exchange gain or loss will Alpha record on the income statement for
the year ended December 31,20x5?

P1,100 loss

P1,100 gain

P6,900 loss

P6,900 gain

P8,000 gain

Question 35 1 / 1 point
Use the following information for questions 35 and 36:
On April 1, 20x3, Shannon Company, a Philippine company, borrowed 100,000 FC (foreign currencies)
from a foreign lender by signing an interest-bearing note due April 1, 20x4. The peso value of the
loan was as follows:

Date Amount
April 1, 20x3 P97,000
December 31, 20x3 103,000
April 1, 20x4 105,000

How much foreign exchange gain or loss should be included in Shannon’s 20x3 income statement?

P3,000 gain

P3,000 loss

P6,000 gain

P6,000 loss

P7,000 gain

Question 36 0 / 1 point
How much foreign exchange gain or loss should be included in Shannon’s 20x4 income statement?

P1,000 gain

P1,000 loss

P2,000 gain

P2,000 loss
P8,000 loss

Question 37 1 / 1 point
On 11/4/x6, a domestic exporter sold inventory to a foreign firm for 100,000 FCUs (foreign currency
units). On that date, the direct exchange rate was P.70. At 12/31/x6, the direct exchange rate was
P.67. On 1/7/x7, when the direct exchange rate was P.71, the domestic exporter received full
payment of 100,000 FCUs. In the exporter’s 20x6 financial statements, what should be reported as an
FX gain or loss?

A P1,000 gain.

A P1,000 loss.

A P3,000 gain.

A P3,000 loss.

A deferred FX gain or loss.

Question 38 1 / 1 point
On 10/5/x6, a domestic importer acquired inventory from a foreign firm for 100,000 FCs (foreign
currency units). On that date, the direct exchange rate was P.80. At 12/31/x6, the direct exchange
rate was P.84. On 1/10/x7, when the direct exchange rate was P.85, the domestic importer made full
payment of 100,000 FCUs. In the importer’s 20x6 financial statements, what should be reported as
an FX gain or loss?

A P4,000 gain.

A P4,000 loss.

A P5,000 gain.

A P5,000 loss.

A deferred FX gain or loss.

Question 39 1 / 1 point
On 10/19/x6, Dell, Inc., ordered inventory from a foreign firm. The terms specified FOB shipping
point and payment of 1,000,000 FCUs (foreign currency units). On 12/20.x6, the foreign vendor
shipped the inventory and invoiced Dell. Dell paid the invoice on 1/7/x7. The direct spot exchange
rates for the euro on the respective dates follow:

10/19/x6 P.802
12/20/x6 P.798
12/31/x6 P.795
1/7/x7 P.793
In Dell’s 20x6 financial statements, what should be reported for the following items? (1) FX Gain in
the Income Statement; (2) Inventory in the Balance Sheet

(1) P3,000; (2) P795,000

(1) P3,000; (2) P798,000

(1) P7,000; (2) P795,000

(1) P7,000; (2) P798,000

(1) P5,000; (2) P198,000

Question 40 1 / 1 point
On 11/4/x6, Patt, Inc. received an inventor order from a foreign firm. The terms specified FOB
shipping point and payment of 1,000,000 FCUs (foreign currency units) 45 days after delivery. On
12/15/x6, Patt shipped the inventory and billed the customer. Patt received the customer’s
remittance in full on 1/20/x7. The direct spot exchange rates for the FCUs on the respective dates
follow:

11/4/x6 P.183
12/15/x6 P.181
12/31/x6 P.180
1/20/x7 P.175

In Patt’s 20x6 financial statements, what should be reported for the following items? (1) FX Loss in
the Income Statement; (2) Sales

(1) P1,000; (2) P181,000

(1) P3,000; (2) P181,000

(1) P3,000; (2) P180,000

(1) P3,000; (2) P183,000

(1) P2,000; (2) P181,000

Question 41 1 / 1 point
Use the following information for questions 41 to 45.
White Corporation, a Philippine company, purchased inventory from Gold Enterprises, a foreign
supplier on November 16 for 70,000 FCUs (foreign currency units). Payment is due on January 16 and
White’s accounting period ends on December 31. The spot rates on November 16, December 31, and
January 16 are 1 FCU = P.65, P.72, and P.69, respectively. What is the peso amount of inventory
recorded in the accounting records on November 16?

P50,400
P45,500

P48,067

P48,300

Question 42 1 / 1 point
What is the peso amount of inventory on the December 31 balance sheet?

P50,400

P45,500

P48,067

P48,300

Question 43 1 / 1 point
What is the peso amount of accounts payable on the December 31 balance sheet?

P50,400

P45,500

P48,067

P48,300

Question 44 1 / 1 point
What is the peso amount of the exchange loss or gain recorded on the income statement at
December 31?

P2,100 loss

P2,100 gain

P4,900 loss

P4,900 gain

Question 45 1 / 1 point
What is the peso amount of the exchange loss or gain recorded on the income statement at January
16?

P2,100 loss

P2,100 gain

P4,900 loss
P4,900 gain

Question 46 1 / 1 point
Use the following information for questions 46 to 49.
Echo, Inc., a Philippine company, sold materials to Radar Corporation, a foreign customer on October
26 for 25,000 FCUs (foreign currency units). Payment is due on March 1 and Echo’s accounting period
ends on December 31. The spot rates on October 26, December 31, and March 1 are 1 FCU = P1.14,
P1.06, and P1.09, respectively. What is the peso amount of sales recorded in the accounting records
on October 26?

P27,250

P26,500

P27,417

P28,500

Question 47 1 / 1 point
What is the peso amount of accounts receivable on the December 31 balance sheet?

P27,250

P26,500

P27,417

P28,500

Question 48 1 / 1 point
What is the peso amount of the exchange loss or gain recorded on the income statement at
December 31?

P2,000 loss

P2,000 gain

P750 loss

P750 gain

Question 49 1 / 1 point
What is the peso amount of the exchange loss or gain recorded on the income statement at March
1?

P2,000 loss

P2,000 gain
P750 loss

P750 gain

Question 50 1 / 1 point
Use the following information for questions 50 to 55.
On July 1, 20x2 CDN purchased inventory from its foreign supplier RNB Enterprises at a cost of 1,000
FCUs. CDN’s year end is on July 31. Some important dates regarding this transaction, as well as the
exchange rates in effect at each of these dates are shown below:

Transaction Date: July 1, 20x2: 1 FCU = P 0.82


Year End: July 31, 20x2: 1 FCU = P 0.81
Settlement Date: July 31, 20x2: 1 FCU = P0.8050

What was the amount paid to RNB by CDN at the settlement date?

820 FCU

P820

P810

P805

1 1 / 1 point
At what amount would CDN record its inventory purchase from RNB at the time of purchase?

820 FCU

P820

P810

P805

Question 52 1 / 1 point
At what amount would CDN record its Liability to RNB at the time of purchase?

820 FCU

P820

P810

P805

Question 53 1 / 1 point
What would be the amount of the foreign exchange gain or loss recorded at the balance sheet date
under the one-transaction approach?

Nil.

A P10 Exchange Loss.

A P10 Exchange Gain.

A P15 Exchange Loss.

Question 54 1 / 1 point
What would be the amount of the foreign exchange gain or loss recorded at the balance sheet date?

Nil.

A P10 Exchange Loss.

A P10 Exchange Gain.

A P15 Exchange Loss.

Question 55 1 / 1 point
What would be the amount of the foreign exchange gain or loss recorded at the settlement date?

A P5 Exchange Gain.

A P10 Exchange Loss.

A P10 Exchange Gain.

A P15 Exchange Loss.

Question 56 1 / 1 point
Use the following information for questions 56 to 59:
XYZ Corp has a calendar year end. On January 1, 20x0, the company borrowed 5.000.00 Foreign
Currency Units (FCUS) from a foreign bank. The loan is to be repaid on December 31 20x3 and requires
interest at 5% to be paid every December 31. The loan and applicable interest are both to be repaid in
pesos. XYZ does not hedge to minimize its foreign exchange risk. The following exchange rates were in
effect throughout the term of the loan:

January 1, 20x0 FCU 1=P1.1500


December 31, 20x0 FCU 1=P1.1490
December 31, 20x1 FCU 1=P1.1485
December 31, 20x2 FCU 1=P1.1483
December 31, 20x3 FCU 1=P1.1487
The average rates in effect for 2010 and 2011 were as follows:

20x0: FCU 1=P1.1493


20x1: FCU 1=P1.1487

At would amount would XYZ record its initial Loan Liability on January 1, 20x0.

P5,476,500

P5,750,000

P5,747,500

P5,471,500

Question 57 1 / 1 point
What is the amount of interest paid during 20x0?

P250,000

P372,500

P287,330

P287,250

Question 58 1 / 1 point
What is the amount of the foreign exchange gain or loss on the principal recognized on the 20x0 Income
Statement?

A P5,000 gain.

A P5,000 loss.

A P10,000 gain.

A P10,000 loss.

Question 59 1 / 1 point
What is the amount of interest paid during 20x1?

P250,000

P372,500

P287,330

P287,125
Question 60 1 / 1 point
Use the following information for Questions 60 and 61:
On November 1, 20x3, Magpie Corporation sold merchandise to William Tell Corporation foreign firm.
Magpie measured and recorded the account receivable from the sale of P78,000. William Tell paid for
this account on November 30, 20x3. Spot rates for FCU on November 1 and November 30, respectively,
were P0.80 and P0.78.
If the sale of the merchandise was denominated in FCU, the November 30 entry to record the receipt of
payment from William Tell included a

credit to Accounts Receivable for P76,050

credit to Exchange Gain for P1,950

debit to Cash for P78,000

debit to Exchange Loss for P1,950

Question 61 1 / 1 point
If the sale of merchandise is denominated in pesos, the November 30 entry to record of the payment
from William Tell included a

debit to Cash for P78,000

debit to Cash for P76,050.

credit to Exchange Gain for P1,950

credit to Accounts Receivable for P76,050

No entry required

Question 62 1 / 1 point
Use the following Information for Questions 62 to 64:
Suppose the foreign exchange rates are
1 Singapore dollar = P.7025
1 HK dollar = P2.5132
Based on the information given above, the indirect exchange rates for the Singapore dollar and the HK
dollars are:

1.7655 Singapore dollars and 1.4235 HK dollars respectively.

0.2975 Singapore dollars and 1.5132 HK dollars respectively.

2.1622 Singapore dollars and 0.4625 HK dollars respectively.


1.4235 Singapore dollars and 0.3979 HK dollars respectively.

Question 63 1 / 1 point
Based on the information given above, how many Philippine pesos must be paid for a purchase of citrus
fruits costing 10,000 HK dollars?

P25,132

P15,132

P3,979

P35,775

Question 64 1 / 1 point
Based on the information given above, how many Singapore dollars are required purchase goods costing
10,000 Philippines pesos?

7,025

14,235

17,655

2,975

Question 65 1 / 1 point
Use the following information for Questions 65 and 66:
A Philippine company purchases a 90-day certificate of deposit from a foreign bank on September 15.
The certificate has a face value of 1,000,000 foreign currencies (FC), costs FC 500,000, and pays interest
at an annual rate of 2 percent. On December 13, the certificate of deposit matures and the company
receives principal and interest of FC 1,005,000. The spot rate on December 14 is P0.77/FC1. The average
spot rate for the period September 15- December 13 is P0.79/FC1.
The exchange gain or loss on this investment is

P30,000 gain

P30,000 loss

P10,000 loss

P10,000 gain

Question 66 1 / 1 point
Interest income on the investment is reported at

P15,400
P 3,950

P4,000

P3,850

Question 67 1 / 1 point
A Philippine manufacturer has sold computer services to a foreign firm and received 200.000 foreign
currency units (FCs). The exchange rates were 1 FC = P.75 on the date of the sale and 1 FC = P.80 when
the receivable was settled. On the transaction date, the settlement exchange rate is estimated to be 1
FC = P.72. By the settlement date, what is the total exchange gain or loss recorded for the transaction if
the two-transaction method is used?

P10,000 exchange gain

P6,000 exchange loss

P10,000 exchange loss

No gain or loss

Question 68 1 / 1 point
August 1, 20x1, a Philippine firm purchased a machine costing 200,000,000 foreign currency (FC) from a
foreign firm to be paid for on October 1, 20x1. Also on August 1, 20x1, the Philippine firm entered into a
contract to purchase 200,000,000 FC to be delivered on October 1, 20x1, at a forward rate of 1 FC =
P0.00783. The exchange rates were as follows:

Spot
August 1, 20x1 1 FC = P0.00781
August 31, 20x1 1 FC = P0.00777
October 1, 20x1 1 FC = P0.00779

Which of the following statements is incorrect concerning the accounting treatment of these
transactions?

The machine’s final recorded value was P1,558,000

The beginning balance in the accounts payable was P1,562,000.

An exchange gain on the accounts payable of P4,000 was recognized on October 1, 20x1.

The value of the accounts payable just before payment, on October 1, 20x1, was P1,558,000.
FOREIGN CURRENCY TRANSLATIONS
1 / 1 point
When translating foreign currency financial statements for a company whose functional currency is
the peso, which of the following accounts is translated using historical change rates? (1) Notes
Payable; (2) Equipment

(1) Yes; (2) Yes

(1) Yes; (2) No

(1) No; (2) No

(1) No; (2) Yes

Question 2 1 / 1 point
Under the temporal method, monetary assets and liabilities are translated by using the exchange
rate existing at the:

beginning of the current year.

date the transaction occurred.

balance sheet date.

None of these.

Question 3 1 / 1 point
The process of translating the accounts of a foreign entity into its functional currency when they are
stated in another currency is called:

verification.

translation.

remeasurement.

None of these.

Question 4 1 / 1 point
Which of the following would be restated using the average exchange rate under the temporal
method?

cost of goods sold

depreciation expense

amortization expense
None of these

Question 5 1 / 1 point
Paid-in capital accounts are translated using the historical exchange rate under:

the current rate method only.

the temporal method only.

both the current rate and temporal methods.

neither the current rate nor temporal methods.

Question 6 1 / 1 point
Which of the following would be restated using the current exchange rate under the temporal
method?

Marketable securities carried at cost.

Inventory carried at market.

Common stock.

None of these.

Question 7 1 / 1 point
The translation adjustment that results from translating the financial statements of a foreign
subsidiary using the current rate method should be:

included as a separate item in the stockholders’ equity section of the balance sheet.

included in the determination of net income for the period it occurs.

deferred and amortized over a period not to exceed forty years.

deferred until a subsequent year when a loss occurs and offset against that loss

Question 8 1 / 1 point
Average exchange rates are used to translate certain items from foreign financial statements into
pesos. Such averages are used to:

smooth out large translation gains and losses.

eliminate temporary fluctuation in exchange rates that may be reversed in the next
fiscal period.

avoid using different exchange rates for some revenue and expense accounts.

approximate the exchange rate in effect when the items were recognized.
Question 9 1 / 1 point
When the functional currency is identified as the peso, land purchased by a foreign subsidiary after
the controlling interest was acquired by the parent company should be translated using the:

historical rate in effect when the land was purchased.

current rate in effect at the balance sheet date.

forward rate.

average exchange rate for the current period.

Question 10 1 / 1 point
The appropriate exchange rate for translating a plant asset in the balance sheet of a foreign
subsidiary in which the functional currency is the peso is the:

current exchange rate.

average exchange rate for the current year.

historical exchange rate in effect when the plant asset was acquired or the date of
acquisition, whichever is later.

forward rate.

Question 11 1 / 1 point
A foreign subsidiary's functional currency is its local currency which has not experienced significant
inflation. The weighted average exchange rate for the current year would be the appropriate
exchange rate for translating (1) Wages expense; (2) Sales to customers

(1) Yes; (2) Yes

(1) Yes; (2) No

(1) No; (2) No

(1) No; (2) Yes

Question 12 1 / 1 point
If the functional currency is determined to be the peso and its financial statements are prepared in
the local currency, PAS 21, requires which of the following procedures to be followed?

Translate the financial statements into pesos using the current rate method.

Remeasure the financial statements into pesos using the temporal


method.

Translate the financial statements into pesos using the temporal method.
Remeasure the financial statements into pesos using the current rate method.

Question 13 1 / 1 point
P Company acquired 90% of the outstanding common stock of 5 Company which is a foreign
company. The acquisition was company d for using the acquisition method. In preparing consolidated
statements, the paid-in capital of S Company should be converted at the:

exchange rate effective when S Company was organized.

exchange rate effective on the date of purchase of the stock of S Company by P


Company.

average exchange rate for the period S Company stock has been upheld by P Company.

current exchange rate.

Question 14 1 / 1 point
In preparing consolidated financial statements of a Philippine parent company subsidiary's functional
currency is the currency:

of the country the parent is located.

of the country the subsidiary is located.

in which the subsidiary primarily generates and spends cash.

in which the subsidiary maintains its accounting records.

Question 15 1 / 1 point
Gains from remeasuring a foreign subsidiary's financial statements from the local currency, which is
not the functional currency, into the parent company's currency should be reported as a(n):

other comprehensive income item.

extraordinary item (net of tax).

part of continuing operations.

deferred credit

Question 16 1 / 1 point
Assuming no significant inflation, gains resulting from the process of translating a foreign entity's
financial statements from the functional currency to peso should be included as a(n)

other comprehensive income item.

extraordinary item ( net of tax)


part of continuing operations.

deferred credit.

Question 17 1 / 1 point
A foreign subsidiary's functional currency is its local currency and inflation of over 100 percent has
experienced over a three-year period. For consolidation purposes, PAS 29 requires the use of:

the current rate method only.

the temporal method only.

both the current rate and temporal methods.

neither the current rate or the temporal method.

Question 18 1 / 1 point
The objective of remeasurement is to:

produce the same results as if the books were maintained in the currency of the foreign
entity's largest customer.

produce the same results as if the books were maintained solely in the local currency.

produce the same results as if the books were maintained solely in the functional
currency.

None of the above.

Question 19 1 / 1 point
The functional currency approach adopted by PAS 21 requires:

separate statements be maintained by the domestic parent company and the foreign
branch both in their own currencies.

separate statements be maintained by the domestic parent company and the foreign
branch with the foreign branch translated into the functional currency.

results from foreign currency changes to be ignored.

a focus on whether the domestic reporting entity's cash flows wit be indirectly or
directly affected by changes in the exchange rates of the foreign entity's currency.

Question 20 1 / 1 point
In which of the following circumstances surrounding a foreign subsidiary of a Philippine parent,
wherein peso is the most likely to be considered the functional currency?
Sales are made globally and collected in pesos. Plant uses local materials and labor and
pays in foreign currency. Intercompany transaction volume is high.

The foreign subsidiary sells product only in their country and receives their own
currency. The materials and labor are also secured in foreign country and paid for with
foreign currencies.

The foreign subsidiary receives their debt capital from a Philippine bank pesos and
products are sold globally for pesos.

Raw materials are acquired from the parent and paid for in pesos. Labor is acquired
locally and paid in foreign currencies. Financing is secured from the parent in pesos.

Question 21 1 / 1 point
A Philippine firm owns 100% of a Japanese automobile manufacturer. The cost of automobile ports is
paid typically 75% of the firm's total product. In which of the following circumstances would neither
the peso Japanese yen be considered the functional currency?

The Japanese firm buys German automobile parts with marks to produce cars sold in
Latin America for pesos.

The Japanese firm buys German automobile parts with pesos to produce cars sold in
Latin America for pesos.

The Japanese firm buys German automobile parts with marks to produce cars sold in
Latin America for marks.

The IASB requires that either the parent's or the subsidiary’s local currency be used as
the functional currency.

Question 22 1 / 1 point
When the functional currency is the foreign entity's currency:

exchange rate changes do not affect the economic well being of the parent

the subsidiary operates as an entity, independent of the parent

Exchange rate changes do not have immediate impact on the cash flows of the parent

All of the above are correct

Question 23 1 / 1 point
The translation (remeasurement) adjustment reported in a translation when the functional currency
is not the foreign currency is included

as a separate component of other comprehensive income

the current liability section of the balance sheet as deferred revenue


the calculation of net income

none of the above

Question 24 1 / 1 point
Assuming that a foreign entity is deemed to be operating in an environment dominated by the local
currency, the entity's assets are translated using

the current rate.

a simple average rate.

a weighted average rate.

a historical rate.

Question 25 1 / 1 point
Assuming that a foreign entity is deemed to be operating in an environment dominated by the local
currency, the entity's capital stock is translated using

the current rate.

a simple average rate.

a weighted average rate.

a historical rate.

Question 26 1 / 1 point
If the functional currency is determined to not be the foreign entity's local currency, translation is
done using

the current rate method

the functional method

the remeasurement method

the derivative method

7 1 / 1 point
In most cases, which of the following is NOT a component of translated retained earnings?

Translated retained earnings at the end of the prior period

Income from the period translated at the historical rate

The value of dividends translated at the exchange rate on the date of declaration
All are components of translated retained earnings

Question 28 1 / 1 point
Which of the following is NOT true regarding foreign statement translation using the current or
temporal method?

All assets and all liabilities are translated at the current exchange rate at the date of
translation.

Monetary assets and liabilities are translated at the current exchange rate at the date of
translation.

Equity accounts other than retained earnings are translated at the historic rate in effect
on the date of the investment.

Elements of income can be translated at a weighted average rate for the period.

Question 29 1 / 1 point
Which of the following is NOT considered when directly computing the translation adjustment for
foreign financial statements?

Beginning amount of net assets held by the domestic investor

Increase or decrease in net assets for the period excluding capital transactions

Increase or decrease in net asset as a result of capital transactions

All are considered when directly computing the translation adjustment

Question 30 1 / 1 point
Exchange rates will not usually directly affect the cash flows of the parent entity in which the
following cases?

The foreign entity operates in a currency other than its own.

The foreign entity operates in its local currency.

The foreign entity functions in a currency other than its local currency.

The foreign entity functions in the parent’s currency.

Question 31 1 / 1 point
The eliminations and adjustment entries necessary to consolidate the parent and subsidiary for
statements are translated as follows:

all balances profits and losses at the current exchange rate on the consolidation date
intercompany balances translate at the rates used for other accounts, profits and losses
translate at an average rate

intercompany balances translate of the current rates, profits and losses translate at an
average rate

none of the above are correct

Question 32 1 / 1 point
A Philippine parent purchased a foreign subsidiary last year at a price in excess of the subsidiary's
book value. This excess is assumed to be traceable to undervalued equipment. When the parent
company prepares its elimination entries for the excess, which of the following combinations of
exchange rates should be used? (1) Equipment; (2) Depreciation Expense

(1) Historical; (2) Current

(1) Current; (2) Historical

(1) Historical; (2) Average

(1) Current; (2) Average

Question 33 1 / 1 point
Which of the following is true concerning the accounting for a foreign investment under the cost
method?

Investment income is translated at the exchange rate on the dividend declaration date

Investment income is translated using the average exchange rate for the year.

Investment income is based on the investee's net income adjusted for the excess of
purchase price over book value

Investment income is based on the investee's net income without adjusting for the
excess of purchase price over book value.

Question 34 1 / 1 point
A debit balance in a parent's cumulative translation adjustment after the first year of owning a
foreign subsidiary suggests which of the following is true?

The exchange rate has strengthened relative to the peso.

The exchange rate has weakened relative to the peso.

The foreign entity had net income but there was not a change in exchange rates.

The foreign entity had a net loss but there was not a change in exchange rates.

Question 35 1 / 1 point
Which of the following procedures would be necessary when a Swiss subsidiary maintains its books
in euros and its functional currency is Japanese Yen and its parent is a Philippine company?

Remeasurement from euros to pesos

Remeasurement from euros to Japanese Yen; translate from Yen to pesos

Remeasurement from Yen to euros; translate from euros to pesos

none of the above

Question 36 1 / 1 point
Assuming that the functional currency of a foreign subsidiary is the local currency, which of following
accounts would be translated at the current rate?

Additional Paid-in Capital

Retained Earnings

Allowance for Doubtful Accounts

Cost of Goods Sold

Question 37 1 / 1 point
Assuming that the functional currency of a foreign subsidiary is not the local currency, which of the
following accounts would be remeasured at the historical rate?

Long-term notes payable

Accounts Payable

Land

Sales Revenue

Question 38 1 / 1 point
Which of the following best describes the measurement of a gain or loss from the sale of a
depreciable asset by a foreign subsidiary whose functional currency is not the local currency?

Reconstruct the journal entry on the date of the sale using the historical rate for cash
and the depreciable asset and its accumulated depreciation.

Reconstruct the journal entry on the date of the sale using current rate for cash and the
historical rate for the depreciable asset and its accumulated depreciation.

Translate the gain using the historical rate.

Translate gains at the current rate and losses at the historical rate.
Question 39 1 / 1 point
Which of the following best describes the accounting for a foreign entity requiring translation or
remeasurement if the local economy is classified as highly inflationary?

The entity's financial statements are first adjusted for inflation and then translated into
the domestic currency.

The entity's financial statements are first adjusted for inflation and then remeasured
into the domestic currency.

The unadjusted trial balance is translated if the functional currency is the local currency.

Question 40 1 / 1 point
The adjustment resulting from the remeasurement of an entity operating in a highly inflationary
environment would appear

in the stockholders equity section of the balance sheet.

as a component of other comprehensive income.

as an ordinary income statement item.

as an extraordinary item on the income statement.

Question 41 1 / 1 point
PAS 21 requires 41 which of the following disclosures from firms involved in foreign currency
transactions?

Beginning cumulative translation adjustments.

Ending cumulative translation adjustments.

The amount of income taxes for the period allocated to translation adjustments.

All are required disclosures.

Question 42 1 / 1 point
In a company's disclosure of foreign currency transactions and hedges and translation adjustments,
all of the following items should be disclosed except

beginning and ending cumulative translation adjustments

the amount of income taxes for the period allocated to translation adjustments.

the amount transferred from cumulative translation adjustment due to changes in


foreign exchange rates.
the aggregate adjustment for the period resulting from translation adjustment.

Question 43 1 / 1 point
The reconciliation of the annual translation adjustment usually includes all of the following, EXCEPT

net assets at the beginning of the period multiplied by the change in exchange rates
during the period.

change in net assets (excluding capital transactions] multiplied by the difference


between the current rate and the average rate used to translate income.

change in net assets (excluding capital transactions) multiplied by the difference


between the historical rate and the average rate used to translate income.

change in net assets due to capital transactions multiplied by the difference between
the current rate and the rate at the time of the capital transaction.

Question 44 1 / 1 point
Exchange gains and losses resulting from translating (not remeasuring) foreign currency financial
statements into U.S. dollars should be included as a(an):

a component of other comprehensive income.

extraordinary item in the income statement for the period in which the rate changes.

ordinary gain/loss item in the income statement.

component of operating income.

Question 45 0 / 1 point
When a Philippine investor entity acquires interest in a foreign entity with the payment of foreign
currency, the determination of excess is calculated

in pesos

in the foreign currency

in pesos if remeasurement (historical rate/temporal method) is indicated

in the foreign currency if translation (current rate/functional method) is indicated

Question 46 1 / 1 point
As part of the consolidation process for a partially-held foreign subsidiary, the elimination entry to
distribute the excess of cost over book value will include a credit to Cumulative Translation
Adjustment Parent

for the amount of excess attributable to identifiable net assets times the difference
between historical and current exchange rates
for the amount of excess attributable to identifiable net assets times the difference
between average and current exchange rates

for the Parent's portion of the excess attributable to identifiable net assets times the
difference between historical and current exchange rates

for the Parent's portion of the excess attributable to identifiable net assets times the
difference between average and current exchange rates

Question 47 1 / 1 point
Consider the consolidation process for a foreign subsidiary: When the excess of cost over book
values is attributable to identifiable assets, those assets are adjusted in the "distribution" elimination
entry by an amount that is calculated as

the difference between cost and fair value as measured in the foreign currency

the difference between cost and fair value as measured in the foreign currency
multiplied by the historical exchange rate

the difference between cost and fair value as measured in the foreign currency
multiplied by the weighted-average exchange rate

the difference between cost and fair value as measured in the foreign currency
multiplied by the current exchange rate

Question 48 1 / 1 point
What is a subsidiary's functional currency?

The parent's reporting currency.

The currency in which transactions are denominated.

The currency in which the entity primarily generates and expends cash.

Always the currency of the country in which the company has its headquarters.

Question 49 1 / 1 point
In comparing the translation and the remeasurement process, which of the following is true?

The reported balance of inventory is normally the same under both methods.

The reported balance of equipment is normally the same under both methods.

The reported balance of sales is normally the same under both methods.

The reported balance of depreciation expense is normally the same under both
methods.

Question 50 1 / 1 point
Which of the following statements is true for the translation process (as opposed to
remeasurement)?

A translation adjustment can affect consolidated net income.

Equipment is translated at the historical exchange rate in effect at the date of its
purchase.

A translation adjustment is created by the change in the relative value of a subsidiary's


net assets caused by exchange rate fluctuations.

A translation adjustment is created by the change in the relative value of a subsidiary's


monetary assets and monetary liabilities caused by exchange rate fluctuations.

Question 51 1 / 1 point
A subsidiary of BB Corporation has one asset (inventory) and no liabilities. The functional currency
for this subsidiary is the foreign currency (FC). The inventory was acquired for 100,000 FC when the
exchange rate was P0.16 = 1 FC. Consolidated statements are to be produced, and the current
exchange rate is P0.19 = 1 FC. Which of the following statements is true for the consolidated
financial statements?

A remeasurement gain must be reported.

A positive translation adjustment must be reported.

A negative translation adjustment must be reported.

A remeasurement loss must be reported.

2 1 / 1 point
At what rates should the following balance sheet accounts in foreign statements be translated rather
than remeasured) into pesos? (1) Accumulated Depreciation – Equipment; (2) Equipment

(1) Current; (2) Current

(1) Current; (2) Average for year

(1) Historical; (2) Current

(1) Historical; (2) Historical

Question 53 1 / 1 point
In the translated financial statements, which method of translation maintains the underlying
valuation methods used in the foreign currency financial statements?

Current rate method; income statement translated at average exchange rate for the
year
Current rate method; income statement translated of exchange rate of the balance
sheet date

Temporal method

Monetary/nonmonetary method

Question 54 1 / 1 point
Which of the following items is not remeasured using historical exchange rates under the temporal
method?

Accumulated depreciation on equipment

Cost of goods sold.

Marketable equity securities

Retained earnings

Question 55 1 / 1 point
In accordance with Philippine generally accepted accounting principles, which translation
combination is appropriate for a foreign operation whose functional currency is the U.S. dolalres? (1)
Method; (2) Treatment of Translation adjustment

(1) Temporal; (2) Other comprehensive income

(1) Temporal; (2) Gain or loss in net income

(1) Current rate; (2) Other comprehensive income

(1) Current rate; (2) Gain or loss in net income

Question 56 1 / 1 point
A foreign subsidiary's functional currency is its local currency, which has not experienced significant
inflation. The weighted average exchange rate for the current year is the appropriate exchange rate
for translating (1) Wages Expense; (2) Wages Payable

(1) Yes; (2) Yes

(1) Yes; (2) No

(1) No; (2) Yes

(1) No; (2) No

Question 57 1 / 1 point
The functional currency of DZ. Inc.'s British subsidiary is the British pound. DZ borrowed pounds as a
partial hedge of its investment in the subsidiary. In preparing consolidated financial statements, DZ's
negative translation adjustment on its investment in the subsidiary exceeded its foreign exchange
gain on its borrowing. How should DZ's report the effects of the negative translation adjustment and
foreign exchange gain in its consolidated financial statements?

Report the translation adjustment in Other Comprehensive Income on the balance


sheet and the foreign exchange gain in the income statement.

Report the translation adjustment in the income statement and defer the foreign
exchange gain in Other Comprehensive Income on the balance sheet.

Report the translation adjustment less the foreign exchange gain in Other
Comprehensive Income on the balance sheet.

Report the translation adjustment less the foreign exchange gain in the income
statement.

Question 58 1 / 1 point
Gains from remeasuring a foreign subsidiary's financial statements from the local currency, which is
not the functional currency into the parent's currency should be reported as a(n)

Deferred foreign exchange gain.

Translation adjustment in Other Comprehensive Income.

Extraordinary item, net of income taxes.

Part of continuing operations.

Question 59 1 / 1 point
At what rates should the following balance sheet accounts in the foreign be restated into pesos? (1)
Equipment; (2) Accumulated Depreciation of Equipment

(1) Current; (2) Current

(1) Current; (2) Average for the year

(1) Historical; (2) Current

(1) Historical; (2) Historical

Question 60 1 / 1 point
A credit-balancing item resulting from the process of restating a foreign entity’s financial statements
from the local currency unit to pesos should be included as a (an):

Separate component of stockholders' equity.

Deferred credit.

Component of income from continuing operations.


Extraordinary item.

Question 61 1 / 1 point
When remeasuring foreign currency financial statements into the functional currency, which of the
following items would be remeasured using a historical exchange rate?

Inventories carried at cost.

Trading securities carried at market values.

Bonds payable.

Accrued liabilities.

Question 62 1 / 1 point
A foreign subsidiary's functional currency is its local currency, which has not experienced significant
inflation. The weighted average exchange rate for the current year would be the appropriate
exchange rate for translating: (1) Sales to Customers; (2) Wages Expenses

(1) No; (2) No

(1) Yes; (2) Yes

(1) No; (2) Yes

(1) Yes; (2) No

Question 63 1 / 1 point
The functional currency of DD Inc.'s subsidiary is the European euro. DD borrowed euros as a partial
hedge of its investment in the subsidiary. In preparing consolidated financial statements, DD's debit
balance of its translation adjustment exceeded its exchange gain on the borrowing. How should the
translation adjustment and the exchange gain be reported in DD's consolidated financial statements?

The translation adjustment should be netted against the exchange gain, and the excess
translation adjustment should be reported in the stockholders' equity section of the
balance sheet.

The translation adjustment should be netted against the exchange gain, and the excess
translation adjustment should be reported in the statement of income in computing net
income.

The translation adjustment is reported as a component of other comprehensive income


and then accumulated in the stockholders' equity section of the balance sheet, and the
exchange gain should be reported in the statement of income in computing net income.

The translation adjustment should be reported in the statement of income, and the
exchange gain should be reported separately in the stockholders' equity section of the
balance sheet.

Question 64 1 / 1 point
Which of the following accounts is a monetary item?

Cost of Sales

Inventory

Investment in Common Stock – IBM

Additional paid-in capital

None of the above

Question 65 1 / 1 point
Which of the following accounts is a monetary item?

Sales

Intercompany Bonds Payable

Investment in Common Stock – IBM

Deferred Income Tax Expense

None of the above

Question 66 1 / 1 point
Which of the following accounts is a monetary item?

Depreciation Expense

Inventory

Investment in Common Stock – Subs

Intercompany Payable – Long-term portion

None of the above

Question 67 1 / 1 point
Which of the following accounts is not a monetary item?

Accounts Receivable

Inventory
Accounts payable

Accrued liabilities

None of the above

Question 68 1 / 1 point
Which of the following accounts is not a monetary item?

Deferred Income Taxes Expense.

Additional Paid-in Capital.

Sales

Deferred charges

None of the above

Question 69 1 / 1 point
Which of the following accounts is a monetary item?

Deferred Income Taxes Expense.

Additional Paid-in Capital.

Sales

Deferred charges

None of the above

Question 70 1 / 1 point
The term current rate is defined

As the exchange rate at the balance sheet reporting date.

As the average exchange rate during the current year.

As the exchange rate in effect when a current year transaction occurred.

Differently for the balance sheet than for the income statement.

None of the above.

Question 71 1 / 1 point
Which translation procedures are followed under the current rate method of translation?

All assets and liabilities are translated at the current exchange rate.
All income statement accounts are translated at the current exchange rate.

A combination of current and historical exchange rates is used in both financial


statements.

Both a and b.

None of the above.

Question 72 1 / 1 point
What occurs in translation under the current rate method of translation?

All income statement accounts are expressed in dollars by using exchange rates in
effect when the items were recognized in the income statement.

The effects of exchange rate changes are reported currently in earnings.

All assets and liabilities are translated using exchange rates that produce the U.S dollar
equivalent at the time the transactions giving rise to the balance occurred.

The temporal method must be used.

None of the above.

Question 73 1 / 1 point
A foreign subsidiary has the foreign currency as its functional currency. The parent enters into an FX
forward to hedge its net investment. What will occur or be the accounting treatment?

There will always be an offsetting effect.

There may or may not be an offsetting effect.

Any gain or loss on the forward exchange contract must be recognized currently in
earnings

Any gain or loss on the forward exchange contract will be deferred on the parent's
books and treated as an adjustment to the Investment in Subsidiary account.

None of the above.

Question 74 1 / 1 point
A parent owns a foreign subsidiary that has as its functional currency the local currency. To avoid
reporting a possible negative effect in the U.S. dollar financial statements from an adverse change in
the exchange rate, the parent should hedge which of the following items?

The net investment (net asset) position.

The net monetary asset position.


The net monetary liability position.

The net monetary position whether it be positive or negative.

Question 75 1 / 1 point
How is the effect of an exchange rate change reported when the current rate method of translation
is used?

Report as a deferred gain or loss in the balance sheet.

Report currently in earnings.

Report in Other Comprehensive Income.

Report in the "Owner Changes in Net Assets" section of the statement of comprehensive

None of the above.

6 1 / 1 point
How is the effect of an exchange rate change for the current year reported under the temporal
method of translation?

Currently in the income statement.

Currently in the income statement as an extraordinary item if material.

As a direct charge or credit to stockholders' equity.

Deferred in the asset or liability section of the balance sheet.

None of the above.

Question 77 1 / 1 point
What is the effect of an exchange rate change called in each of the following situations?

Functional Currency
The Foreign Currency The Philippine Peso
a. Translation adjustment FX transaction Gain or Loss
b. FX transaction Gain or Loss Translation adjustment
c. FX transaction Gain or Loss FX transaction Gain or Loss
d. Translation adjustment Translation adjustment

___A___
1 / 1 point
Under the temporal method of translation, how is the effect of an exchange rate change reported?
As a deferred gain or loss in the balance sheet.

Currently in the income statement.

As a direct adjustment to equity.

In the "Other Non-owner Changes in Net Assets” section of the statement of comprehensive
income.

As an extraordinary item.

You might also like